You are on page 1of 84

CONCEALMENT AND REPRESENTATION (SECTIONS 26-47)

G.R. No. 92492. June 17, 1993.


1. THELMA VDA. DE CANILANG, petitioner, vs. HON. COURT OF APPEALS and GREAT
PACIFIC LIFE ASSURANCE CORPORATION, respondents.
Insurance Law; Concealment; The information which Jaime Canilang failed to disclose was
material to the ability of Great Pacific to estimate the probable risk he presented as a subject of
life insurance.We agree with the Court of Appeals that the information which Jaime Canilang
failed to disclose was material to the ability of Great Pacific to estimate the probable risk he
presented as a subject of life insurance. Had Canilang disclosed his visits to his doctor, the
diagnosis made and the medicines prescribed by such doctor, in the insurance application, it
may be reasonably assumed that Great Pacific would have made further inquiries and would
have probably refused to issue a non-medical insurance policy or, at the very least, required a
higher premium for the same coverage. The materiality of the information withheld by Great
Pacific did not depend upon the state of mind of Jaime Canilang. A mans state of mind or
subjective belief is not capable of proof in our judicial process, except through proof of external
acts or failure to act from which inferences as to his subjective belief may be reasonably drawn.
Neither does materiality depend upon the actual or physical events which ensue. Materiality
relates rather to the probable and reasonable influence of the facts upon the party to whom
the communication should have been made, in assessing the risk involved in making or omitting
to make further inquiries and in accepting the application for insurance; that probable and
reasonable influence of the facts concealed must, of course, be determined objectively, by the
judge ultimately.
FELICIANO, J.:
FACTS: On 18 June 1982, Jaime Canilang consulted Dr. Wilfredo B. Claudio and was diagnosed as
suffering from sinus tachycardia. The doctor prescribed the following for him: Trazepam, a
tranquilizer; and Aptin, a beta-blocker drug. Mr. Canilang consulted the same doctor again on 3
August 1982 and this time was found to have acute bronchitis.
On the next day, 4 August 1982, Jaime Canilang applied for a non-medical insurance policy
with respondent Great Pacific Life Assurance Company (Great Pacific) naming his wife,
petitioner Thelma Canilang, as his beneficiary.1 Jaime Canilang was issued ordinary life insurance
Policy No. 345163, with the face value of P19,700, effective as of 9 August 1982.
On 5 August 1983, Jaime Canilang died of congestive heart failure, anemia, and chronic
anemia.2 Petitioner, widow and beneficiary of the insured, filed a claim with Great Pacific which
the insurer denied on 5 December 1983 upon the ground that the insured had concealed
material information from it.
Petitioner then filed a complaint against Great Pacific with the Insurance Commission for
recovery of the insurance proceeds. During the hearing called by the Insurance Commissioner,
petitioner testified that she was not aware of any serious illness suffered by her late husband3
and that, as far as she knew, her husband had died because of a kidney disorder.4 A deposition
given by Dr. Wilfredo Claudio was presented by petitioner. There Dr. Claudio stated that he was
the family physician of the deceased Jaime Canilang5 and that he had previously treated him for
sinus tachycardia and acute bronchitis.6 Great Pacific for its part presented Dr. Esperanza
Quismorio, a physician and a medical underwriter working for Great Pacific.7 She testified that
the deceaseds insurance application had been approved on the basis of his medical
declaration.8 She explained that as a rule, medical examinations are required only in cases
where the applicant has indicated in his application for insurance coverage that he has
previously undergone medical consultation and hospitalization. In a decision dated 5 November
1985, Insurance Commissioner Armando Ansaldo ordered Great Pacific to pay P19,700.00 plus
legal interest and P2,000.00 as attorneys fees after holding that:

1. the ailment of Jaime Canilang was not so serious that, even if it had been disclosed, it would
not have affected Great Pacifics decision to insure him;
2. Great Pacific had waived its right to inquire into the health condition of the applicant by the
issuance of the policy despite the lack of answers to some of the pertinent questions in the
insurance application;
3. there was no intentional concealment on the part of the insured Jaime Canilang as he had
thought that he was merely suffering from a minor ailment and simple cold;10 and
4. Batas Pambansa Blg. 874 which voids an insurance contract, whether or not concealment was
intentionally made, was not applicable to Canilangs case as that law became effective only on 1
June 1985.
On appeal by Great Pacific, the Court of Appeals reversed and set aside the decision of the
Insurance Commissioner and dismissed Thelma Canilangs complaint and Great Pacifics
counterclaim. The Court of Appeals found that the use of the word intentionally by the
Insurance Commissioner in defining and resolving the issue agreed upon by the parties at pretrial before the Insurance Commissioner was not supported by the evidence; that the issue
agreed upon by the parties had been whether the deceased insured, Jaime Canilang, made a
material concealment as to the state of his health at the time of the filing of insurance
application, justifying respondents denial of the claim. The Court of Appeals also found that the
failure of Jaime Canilang to disclose previous medical consultation and treatment constituted
material information which should have been communicated to Great Pacific to enable the latter
to make proper inquiries. The Court of Appeals finally held that the Ng Gan Zee case which had
involved misrepresentation was not applicable in respect of the case at bar which involves
concealment.
G.R. No. 105135. June 22, 1995.
2. SUNLIFE ASSURANCE COMPANY OF CANADA, petitioner, vs. The Hon. COURT OF
APPEALS and Spouses ROLANDO and BERNARDA BACANI, respondents.
Insurance Law; Concealment; Words and Phrases; A neglect to communicate that
which a party knows and ought to communicate is called concealment.In weighing the
evidence presented, the trial court concluded that indeed there was concealment and
misrepresentation, however, the same was made in good faith and the facts concealed or
misrepresented were irrelevant since the policy was non-medical. We disagree. Section 26 of
The Insurance Code is explicit in requiring a party to a contract of insurance to communicate to
the other, in good faith, all facts within his knowledge which are material to the contract and as
to which he makes no warranty, and which the other has no means of ascertaining. Said Section
provides: A neglect to communicate that which a party knows and ought to communicate, is
called concealment.
Same; Same; Materiality; Matters relating to the health of the insured are material
and relevant to the approval and issuance of the life insurance policy as these
definitely affect the insurers action on the application.The terms of the contract are
clear. The insured is specifically required to disclose to the insurer matters relating to his health.
The information which the insured failed to disclose were material and relevant to the approval
and issuance of the insurance policy. The matters concealed would have definitely affected
petitioners action on his application, either by approving it with the corresponding adjustment
for a higher premium or rejecting the same. Moreover, a disclosure may have warranted a
medical examination of the insured by petitioner in order for it to reasonably assess the risk
involved in accepting the application.
Same; Same; Good faith is no defense in concealment.Thus, good faith is no defense
in concealment. The insureds failure to disclose the fact that he was hospitalized for two weeks
prior to filing his application for insurance, raises grave doubts about his bona fides. It appears
that such concealment was deliberate on his part.

Same; Same; Non-Medical Insurance Contracts; The waiver of a medical examination in a nonmedical insurance contract renders even more material the information required of the applicant
concerning previous condition of health and diseases suffered.The argument, that petitioners
waiver of the medical examination of the insured debunks the materiality of the facts concealed,
is untenable. We reiterate our ruling in Saturnino v. Philippine American Life Insurance Company,
7 SCRA 316 (1963), that x x x the waiver of a medical examination [in a non-medical insurance
contract] renders even more material the information required of the applicant concerning
previous condition of health and diseases suffered, for such information necessarily constitutes
an important factor which the insurer takes into consideration in deciding whether to issue the
policy or not x x x.
Same; Same; It is well-settled that the insured need not die of the disease he had
failed to disclose to the insurer, as it is sufficient that his non-disclosure misled the
insurer in forming his estimates of the risks of the proposed insurance policy or in
making inquiries.Anent the finding that the facts concealed had no bearing to the cause of
death of the insured, it is well settled that the insured need not die of the disease he had failed
to disclose to the insurer. It is sufficient that his non-disclosure misled the insurer in forming his
estimates of the risks of the proposed insurance policy or in making inquiries (Henson v. The
Philippine American Life Insurance Co., 56. O.G. No. 48 [1960]).
QUIASON, J.:
FACTS: On April 15, 1986, Robert John B. Bacani procured a life insurance contract for himself
from petitioner. He was issued Policy No. 3-903-766-X valued at P100,000.00, with double
indemnity in case of accidental death. The designated beneficiary was his mother, respondent
Bernarda Bacani.
On June 26, 1987, the insured died in a plane crash. Respondent Bernarda Bacani filed a claim
with petitioner, seeking the benefits of the insurance policy taken by her son. Petitioner
conducted an investigation and its findings prompted it to reject the claim.
In its letter, petitioner informed respondent Bernarda Bacani, that the insured did not disclose
material facts relevant to the issuance of the policy, thus rendering the contract of insurance
voidable. A check representing the total premiums paid in the amount of P10,172.00 was
attached to said letter.
Petitioner claimed that the insured gave false statements in his application when he answered
the following questions:
5. Within the past 5 years have you:
a) consulted any doctor or other health practitioner?
b) submitted to:
ECG?
X-rays?
blood tests?
other tests?
c) attended or been admitted to any hospital or other medical facility?
6. Have you ever had or sought advice for:
xxx
xxx
xxx
b) urine, kidney or bladder disorder?
(Rollo, p. 53).
The deceased answered question No. 5(a) in the affirmative but limited his answer to a
consultation with a certain Dr. Reinaldo D. Raymundo of the Chinese General Hospital on
February 1986, for cough and flu complications. The other questions were answered in the
negative (Rollo, p. 53).
Petitioner discovered that two weeks prior to his application for insurance, the insured was
examined and confined at the Lung Center of the Philippines, where he was diagnosed for renal

failure. During his confinement, the deceased was subjected to urinalysis, ultra-sonography and
hematology tests.
On November 17, 1988, respondent Bernarda Bacani and her husband, respondent Rolando
Bacani, filed an action for specific performance against petitioner with the Regional Trial Court,
Branch 191, Valenzuela, Metro Manila. Petitioner filed its answer with counterclaim and a list of
exhibits consisting of medical records furnished by the Lung Center of the Philippines.
On January 14, 1990, private respondents filed a Proposed Stipulation with Prayer for Summary
Judgment where they
In its letter, petitioner informed respondent Bernarda Bacani, that the insured did not disclose
material facts relevant to the issuance of the policy, thus rendering the contract of insurance
voidable. A check representing the total premiums paid in the amount of P10,172.00 was
attached to said letter.
Petitioner claimed that the insured gave false statements in his application when he answered
the questions.
The deceased answered question No. 5(a) in the affirmative but limited his answer to a
consultation with a certain Dr. Reinaldo D. Raymundo of the Chinese General Hospital on
February 1986, for cough and flu complications. The other questions were answered in the
negative (Rollo, p. 53).
Petitioner discovered that two weeks prior to his application for insurance, the insured was
examined and confined at the Lung Center of the Philippines, where he was diagnosed for renal
failure. During his confinement, the deceased was subjected to urinalysis, ultra-sonography and
hematology tests.
On November 17, 1988, respondent Bernarda Bacani and her husband, respondent Rolando
Bacani, filed an action for specific performance against petitioner with the Regional Trial Court,
Branch 191, Valenzuela, Metro Manila. Petitioner filed its answer with counterclaim and a list of
exhibits consisting of medical records furnished by the Lung Center of the Philippines.
No. L-30685. May 30, 1983.
3. NG GAN ZEE, plaintiff-appellee,
CORPORATION, defendant-appellant.

vs.

ASIAN

CRUSADER

LIFE

ASSURANCE

Mercantile Law; Insurance; Concealment; When concealment exists; Nature of


concealment.Thus, concealment exists where the assured had knowledge of a fact material
to the risk, and honesty, good faith, and fair dealing requires that he should communicate it to
the assurer, but he designedly and intentionally withholds the same. It has also been held that
the concealment must, in the absence of inquiries, be not only material, but fraudulent, or the
fact must have been intentionally withheld.
Same; Same; Same; Misrepresentation; Fraudulent intent of insured must be
established to entitle insurer to rescind insurance contract; Misrepresentation, as
defense of insurer, is an affirmative defense which must be proved.Sec. 27 of the
Insurance Law, abovequoted, nevertheless requires that fraudulent intent on the part of the
insured be established to entitle the insurer to rescind the contract. And as correctly observed by
the lower court, misrepresentation as a defense of the insurer to avoid liability is an affirmative
defense. The duty to establish such a defense by satisfactory and convincing evidence rests
upon the defendant. The evidence before the Court does not clearly and satisfactorily establish
that defense.

Same; Same; Same; Statement of insured that tumor he was operated on was
associated with ulcer of the stomach, an expression made in good faith as to the
nature of his ailment and operation and without knowledge of its incorrectness and
without any deliberate intent to mislead the insurer.It bears emphasis that Kwong Nam
had informed the appellants medical examiner that the tumor for which he was operated on was
associated with ulcer of the stomach. In the absence of evidence that the insured had
sufficient medical knowledge as to enable him to distinguish between peptic ulcer and a
tumor, his statement that said tumor was associated with ulcer of the stomach, should be
construed as an expression made in good faith of his belief as to the nature of his ailment and
operation. Indeed, such statement must be presumed to have been made by him without
knowledge of its incorrectness and without any deliberate intent on his part to mislead the
appellant.
ESCOLIN, J.:
FACTS: On May 12, 1962, Kwong Nam applied for a 20-year endowment insurance on his life for
the sum of P20,000.00, with his wife, appellee Ng Gan Zee, as beneficiary. On the same date,
appellant, upon receipt of the required premium from the insured, approved the application and
issued the corresponding policy. On December 6, 1963, Kwong Nam died of cancer of the liver
with metastasis. All premiums had been religiously paid at the time of his death.
On January 10, 1964, his widow Ng Gan Zee presented a claim in due form to appellant for
payment of the face value of the policy. On the same date, she submitted the required proof of
death of the insured. Appellant denied the claim on the ground that the answers given by the
insured to the questions appearing in his application for life insurance were untrue.
Appellee brought the matter to the attention of the Insurance Commissioner, the Hon. Francisco
Y. Mandamos, and the latter, after conducting an investigation, wrote the appellant that he had
found no material concealment on the part of the insured and that, therefore, appellee should be
paid the full face value of the policy. This opinion of the Insurance Commissioner
notwithstanding, appellant refused to settle its obligation.
Appellant alleged that the insured was guilty of misrepresentation when he answered No to the
following question appearing in the application for life insuranceHas any life insurance
company ever refused your application for insurance or for reinstatement of a lapsed policy or
offered you a policy different from that applied for? If, so, name company and date.
In its brief, appellant rationalized its thesis thus:
x x x As pointed out in the foregoing summary of the essential facts in this case, the insured
had in January, 1962, applied for reinstatement of his lapsed life insurance policy with the Insular
Life Insurance Co., Ltd. but this was declined by the insurance company, although later on
approved for reinstatement with a very high premium as a result of his medical examination.
Thus notwithstanding, the said insured answered No to the [above] question propounded to
him, x x x.
No. L-16163.February 28, 1963.
4. IGNACIO SATURNINO, in his own behalf and as the JUDICIAL GUARDIAN OF CARLOS
SATURNINO, minor, plaintiffs-appellants, vs. THE PHILIPPINE AMERICAN LIFE
INSURANCE COMPANY, defendant-appellee.
Insurance; Non-medical insurance; Medical history material to insurability of
applicant.In non-medical insurance, the waiver of medical examination renders even more
material the information required of the applicant concerning previous condition of health and
diseases suffered, for such information necessarily constitutes an important factor which the
insurer takes into consideration in deciding whether to issue the policy or not.

Same; Same; Concealment of previous operation.The concealment of the fact of the


operation itself is fraudulent, as there could not have been any mistake about it, no matter what
the ailment.
Same; Same; Concealment, whether intentional or unintentional; Ground for
rescission.In this jurisdiction, a concealment, whether intentional or unintentional, entitles the
insurer to rescind the contract of insurance, concealment being defined as negligence to
communicate that which a party knows and ought to communicate (Sections 24 and 26, Act No.
2427).
MAKALINTAL,J.:
FACTS: Plaintiffs, now appellants, filed this action in the Court of First Instance of Manila to
recover the sum of P5,000.00, corresponding to the face value of an insurance policy issued by
defendant on the life of Estefania A. Saturnino, and the sum of P1,500.00 as attorneys fees.
Defendant, now appellee, set up special defenses in its answer, with a counterclaim for damages
allegedly sustained as a result of the unwarranted presentation of this case. Both the complaint
and the counterclaim were dismissed by the trial court; but appellants were declared entitled to
the return of the premium already paid; plus interest at 6% up to January 8, 1959, when a check
for the corresponding amount P359.65 was sent to them by appellee.
The policy sued upon is one for 20-year endowment non-medical insurance. This kind of policy
dispenses with the medical examination of the applicant usually required in ordinary life policies.
However, detailed information is called for in the application concerning the applicants health
and medical history. The written application in this case was submitted by Saturnino to appellee
on November 16, 1957, witnessed by appellees agent Edward A. Santos. The policy was issued
on the same day, upon payment of the first years premium of P339.25. On September 19, 1958
Saturnino died of pneumonia, secondary to influenza. Appellants here, who are her surviving
husband and minor child, respectively, demanded payment of the face value of the policy. The
claim was rejected and this suit was subsequently instituted.
It appears that two months prior to the issuance of the policy or on September 9, 1957,
Saturnino was operated on for cancer, involving complete removal of the right breast, including
the pectoral muscles and the glands found in the right armpit. She stayed in the hospital for a
period of eight days, after which she was discharged, although according to the surgeon who
operated on her she could not be considered definitely cured, her ailment being of the malignant
type.
Notwithstanding the fact of her operation Estefania A. Saturnino did not make a disclosure
thereof in her application for insurance. On the contrary, she stated therein that she did not
have, nor had she ever had, among other ailments listed in the application, cancer or other
tumors; that she had not consulted any physician, undergone any operation or suffered any
injury within the preceding five years; and that she had never been treated for nor did she ever
have any illness or disease peculiar to her sex, particularly of the breast, ovaries, uterus, and
menstrual disorders. The application also recites that the foregoing declarations constituted a
further basis for the issuance of the policy.
The question at issue is whether or not the insured made such false representations of material
facts as to avoid the policy.
No. L-34200. September 30, 1982.
5. REGINA L. EDILLON, as assisted by her husband, MARCIAL EDILLON, petitionersappellants, vs. MANILA BANKERS LIFE INSURANCE CORPORATION and the COURT OF
FIRST INSTANCE OF RIZAL, BRANCH V, QUEZON CITY, respondents-appellees.
Commercial Law; Insurance; Concealment of age, not a case of; Estoppel; Acceptance
by insurance corporation of the premium and issuance of corresponding certificate of

insurance in favor of the insured was deemed a waiver of the exclusionary condition
of overage stated in said certificate of insurance.The age of the insured Carmen O.
Lapuz was not concealed to the insurance company. Her application for insurance coverage
which was on a printed form furnished by private respondent and which contained very few
items of information clearly indicated her age at the time of filing the same to be almost 65
years of age. Despite such information which could hardly be overlooked in the application form,
considering its prominence thereon and its materiality to the coverage applied for, the
respondent insurance corporation received her payment of premium and issued the
corresponding certificate of insurance without question. The accident which resulted in the death
of the insured, a risk covered by the policy, occurred on May 31, 1969 or FORTY-FIVE (45) DAYS
after the insurance coverage was applied for. There was sufficient time for the private
respondent to process the application and to notice that the application was over 60 years of age
and thereby cancel the policy on that ground if it was minded to do so. If the private respondent
failed to act, it is either because it was willing to waive such disqualification; or, through the
negligence or incompetence of its employees for which it has only itself to blame, it simply
overlooked such fact. Under the circumstances, the insurance corporation is already deemed in
estoppel. Its inaction to revoke the policy despite a departure from the exclusionary condition
contained in the said policy constituted a waiver of such condition.
VASQUEZ, J.:
FACTS: Sometime in April 1969, Carmen O, Lapuz applied with respondent insurance corporation
for insurance coverage against accident and injuries. She filled up the blank application form
given to her and filed the same with the respondent insurance corporation. In the said
application form which was dated April 15, 1969, she gave the date of her birth as July 11, 1904.
On the same date, she paid the sum of P20.00 representing the premium for which she was
issued the corresponding receipt signed by an authorized agent of the respondent insurance
corporation. (Rollo, p. 27.) Upon the filing of said application and the payment of the premium on
the policy applied for, the respondent insurance corporation issued to Carmen O. Lapuz its
Certificate of Insurance No. 128866. (Rollo, p. 28.) The policy was to be effective for a period of
90 days.
On May 31, 1969 or during the effectivity of Certificate of Insurance No. 12886, Carmen O. Lapuz
died in a vehicular accident in the North Diversion Road. On June 7, 1969, petitioner Regina L.
Edillon, a sister of the insured and who was the named beneficiary in the policy, filed her claim
for the proceeds of the insurance, submitting all the necessary papers and other requisites with
the private respondent. Her claim having been denied, Regina L. Edillon instituted this action in
the Court of First Instance of Rizal on August 27, 1969.
In resisting the claim of the petitioner, the respondent insurance corporation relies on a provision
contained in the Certificate of Insurance, excluding its liability to pay claims under the policy in
behalf of "persons who are under the age of sixteen (16) years of age or over the age of sixty
(60) years x x x." It is pointed out that the insured being over sixty (60) years of age when she
applied for the insurance coverage, the policy was null and void, and no risk on the part of the
respondent insurance corporation had arisen therefrom.
The trial court sustained the contention of the private respondent and dismissed the complaint;
ordered the petitioner to pay attorney's fees in the sum of ONE THOUSAND (P1,000.00) PESOS in
favor of the private respondent; and ordered the private respondent to return the sum of
TWENTY (P20.00) PESOS received by way of premium on the insurancy policy. It was reasoned
out that a policy of insurance being a contract of adhesion, it was the duty of the insured to know
the terms of the contract he or she is entering into; the insured in this case, upon learning from
its terms that she could not have been qualified under the conditions stated in said contract,
what she should have done is simply to ask for a refund of the premium that she paid. It was
further argued by the trial court that the ruling calling for a liberal interpretation of an insurance
contract in favor of the insured and strictly against the insurer may not be applied in the present
case in view of the peculiar facts and circumstances obtaining therein.

G.R. No. 186983. February 22, 2012.


6. MA. LOURDES S. FLORENDO, petitioner, vs. PHILAM PLANS, INC., PERLA ABCEDE and
MA. CELESTE ABCEDE, respondents.
Insurance Law; Concealment; Manuel had been taking medicine for his heart condition and
diabetes when he submitted his pension plan application; Pursuant to Section 27 of the
Insurance Code, Manuels concealment entitles Philam Plans to rescind its contract of insurance
with him.As already stated, Manuel had been taking medicine for his heart condition and
diabetes when he submitted his pension plan application. These clearly fell within the five-year
period. More, even if Perlas knowledge of Manuels pacemaker may be applied to Philam Plans
under the theory of imputed knowledge, it is not claimed that Perla was aware of his two other
afflictions that needed medical treatments. Pursuant to Section 27 of the Insurance Code,
Manuels concealment entitles Philam Plans to rescind its contract of insurance with him.
Same; Same; Insured persons may accept policies without reading them, and that this
is not negligence per se. But, this is not without any exception.As the Court said in
New Life Enterprises v. Court of Appeals, 207 SCRA 669 (1992): It may be true that x x x insured
persons may accept policies without reading them, and that this is not negligence per se. But,
this is not without any exception. It is and was incumbent upon petitioner Sy to read the
insurance contracts, and this can be reasonably expected of him considering that he has been a
businessman since 1965 and the contract concerns indemnity in case of loss in his moneymaking trade of which important consideration he could not have been unaware as it was
precisely the reason for his procuring the same. The same may be said of Manuel, a civil
engineer and manager of a construction company. He could be expected to know that one must
read every document, especially if it creates rights and obligations affecting him, before signing
the same. Manuel is not unschooled that the Court must come to his succor. It could reasonably
be expected that he would not trifle with something that would provide additional financial
security to him and to his wife in his twilight years.
Same; Same; Incontestability Clause; An incontestability clause precludes the insurer
from disowning liability under the policy it issued on the ground of concealment or
misrepresentation.In a final attempt to defend her claim for benefits under Manuels pension
plan, Lourdes points out that any defect or insufficiency in the information provided by his
pension plan application should be deemed waived after the same has been approved, the policy
has been issued, and the premiums have been collected. The Court cannot agree. The
comprehensive pension plan that Philam Plans issued contains a one-year incontestability period.
It states: VIII. INCONTESTABILITY After this Agreement has remained in force for one (1) year, we
can no longer contest for health reasons any claim for insurance under this Agreement, except
for the reason that installment has not been paid (lapsed), or that you are not insurable at the
time you bought this pension program by reason of age. If this Agreement lapses but is
reinstated afterwards, the one (1) year contestability period shall start again on the date of
approval of your request for reinstatement. The above incontestability clause precludes the
insurer from disowning liability under the policy it issued on the ground of concealment or
misrepresentation regarding the health of the insured after a year of its issuance. Since Manuel
died on the eleventh month following the issuance of his plan, the one year incontestability
period has not yet set in. Consequently, Philam Plans was not barred from questioning Lourdes
entitlement to the benefits of her husbands pension plan.

ABAD,J.:
FACTS: This case is about an insureds alleged concealment in his pension plan application of his
true state of health and its effect on the life insurance portion of that plan in case of death.

On October 23, 1997 Manuel Florendo filed an application for comprehensive pension plan with
respondent Philam Plans, Inc. (Philam Plans) after some convincing by respondent Perla Abcede.
The plan had a pre-need price of P997,050.00, payable in 10 years, and had a maturity value of
P2,890,000.00 after 20 years.1 Manuel signed the application and left to Perla the task of
supplying the information needed in the application.2 Respondent Ma. Celeste Abcede, Perlas
daughter, signed the application as sales counselor.
Aside from pension benefits, the comprehensive pension plan also provided life insurance
coverage to Florendo.4 This was covered by a Group Master Policy that Philippine American Life
Insurance Company (Philam Life) issued to Philam Plans.5 Under the master policy, Philam Life
was to automatically provide life insurance coverage, including accidental death, to all who
signed up for Philam Plans comprehensive pension plan.6 If the plan holder died before the
maturity of the plan, his beneficiary was to instead receive the proceeds of the life insurance,
equivalent to the pre-need price. Further, the life insurance was to take care of any unpaid
premium until the pension plan matured, entitling the beneficiary to the maturity value of the
pension plan.
On October 30, 1997 Philam Plans issued Pension Plan Agreement PP430055848 to Manuel, with
petitioner Ma. Lourdes S. Florendo, his wife, as beneficiary. In time, Manuel paid his quarterly
premiums.
Eleven months later or on September 15, 1998, Manuel died of blood poisoning. Subsequently,
Lourdes filed a claim with Philam Plans for the payment of the benefits under her husbands
plan.10 Because Manuel died before his pension plan matured and his wife was to get only the
benefits of his life insurance, Philam Plans forwarded her claim to Philam Life.
On May 3, 1999 Philam Plans wrote Lourdes a letter,12 declining her claim. Philam Life found that
Manuel was on maintenance medicine for his heart and had an implanted pacemaker. Further, he
suffered from diabetes mellitus and was taking insulin. Lourdes renewed her demand for
payment under the plan13 but Philam Plans rejected it,14 prompting her to file the present
action against the pension plan company before the Regional Trial Court (RTC) of Quezon City.
G.R. No. 113899. October 13, 1999.
7. GREAT PACIFIC LIFE ASSURANCE CORP., petitioner, vs. COURT OF APPEALS AND
MEDARDA V. LEUTERIO, respondents.
Insurance; Mortgages; Mortgage Redemption Insurance; Words and Phrases; The
rationale of a group insurance policy of mortgagors, otherwise known as the
mortgage redemption insurance, is a device for the protection of both the
mortgagee and the mortgagor; Where the mortgagor pays the insurance premium
under the group insurance policy, making the loss payable to the mortgagee, the
insurance is on the mortgagors interest, and the mortgagor continues to be a party
to the contract.We must consider the insurable interest in mortgaged properties and the
parties to this type of contract. The rationale of a group insurance policy of mortgagors,
otherwise known as the mortgage redemption insurance, is a device for the protection of both
the mortgagee and the mortgagor. On the part of the mortgagee, it has to enter into such form
of contract so that in the event of the unexpected demise of the mortgagor during the
subsistence of the mortgage contract, the proceeds from such insurance will be applied to the
payment of the mortgage debt, thereby relieving the heirs of the mortgagor from paying the
obligation. In a similar vein, ample protection is given to the mortgagor under such a concept so
that in the event of death; the mortgage obligation will be extinguished by the application of the
insurance proceeds to the mortgage indebtedness. Consequently, where the mortgagor pays the
insurance premium under the group insurance policy, making the loss payable to the mortgagee,
the insurance is on the mortgagors interest, and the mortgagor continues to be a party to the

contract. In this type of policy insurance, the mortgagee is simply an appointee of the insurance
fund, such loss-payable clause does not make the mortgagee a party to the contract.
Same; Same; Same; Parties; Real Party in Interest; The insured may be regarded as
the real party in interest, although he has assigned the policy for the purpose of
collection, or has assigned as collateral security any judgment he may obtain.The
insured private respondent did not cede to the mortgagee all his rights or interests in the
insurance, the policy stating that: In the event of the debtors death before his indebtedness
with the Creditor [DBP] shall have been fully paid, an amount to pay the outstanding
indebtedness shall first be paid to the creditor and the balance of sum assured, if there is any,
shall then be paid to the beneficiary/ies designated by the debtor. When DBP submitted the
insurance claim against petitioner, the latter denied payment thereof, interposing the defense of
concealment committed by the insured. Thereafter, DBP collected the debt from the mortgagor
and took the necessary action of foreclosure on the residential lot of private respondent. In
Gonzales La O vs. Yek Tong Lin Fire & Marine Ins. Co. we held: Insured, being the person with
whom the contract was made, is primarily the proper person to bring suit thereon. * * * Subject
to some exceptions, insured may thus sue, although the policy is taken wholly or in part for the
benefit of another person named or unnamed, and although it is expressly made payable to
another as his interest may appear or otherwise. * * * Although a policy issued to a mortgagor is
taken out for the benefit of the mortgagee and is made payable to him, yet the mortgagor may
sue thereon in his own name, especially where the mortgagees interest is less than the full
amount recoverable under the policy, * * *. And in volume 33, page 82, of the same work, we
read the following: Insured may be regarded as the real party in interest, although he has
assigned the policy for the purpose of collection, or has assigned as collateral security any
judgment he may obtain.
Same; Concealment; Words and Phrases; Concealment exists where the assured had
knowledge of a fact material to the risk, and honesty, good faith, and fair dealing
requires that he should communicate it to the assured, but he designedly and
intentionally withholds the same.The second assigned error refers to an alleged
concealment that the petitioner interposed as its defense to annul the insurance contract.
Petitioner contends that Dr. Leuterio failed to disclose that he had hypertension, which might
have caused his death. Concealment exists where the assured had knowledge of a fact material
to the risk, and honesty, good faith, and fair dealing requires that he should communicate it to
the assured, but he designedly and intentionally withholds the same.
Same; Same; The fraudulent intent on the part of the insured must be established to
entitle the insurer to rescind the contract.The fraudulent intent on the part of the insured
must be established to entitle the insurer to rescind the contract. Misrepresentation as a defense
of the insurer to avoid liability is an affirmative defense and the duty to establish such defense
by satisfactory and convincing evidence rests upon the insurer. In the case at bar, the petitioner
failed to clearly and satisfactorily establish its defense, and is therefore liable to pay the
proceeds of the insurance.
Same; Life Insurance; Unless the interest of a person insured is susceptible of exact
pecuniary measurement, the measure of indemnity under a policy of insurance upon
life or health is the sum fixed in the policy.And that brings us to the last point in the
review of the case at bar. Petitioner claims that there was no evidence as to the amount of Dr.
Leuterios outstanding indebtedness to DBP at the time of the mortgagors death. Hence, for
private respondents failure to establish the same, the action for specific performance should be
dismissed. Petitioners claim is without merit. A life insurance policy is a valued policy. Unless the
interest of a person insured is susceptible of exact pecuniary measurement, the measure of
indemnity under a policy of insurance upon life or health is the sum fixed in the policy.
Same; Mortgages; Mortgage Redemption Insurance; Where the mortgagee under a
mortgage redemption insurance has already foreclosed on the mortgage, it cannot

collect the insurance proceedsthe proceeds then rightly belong to the heirs of the
mortgagor.We noted that the Court of Appeals decision was promulgated on May 17, 1993. In
private respondents memorandum, she states that DBP foreclosed in 1995 their residential lot,
in satisfaction of mortgagors outstanding loan. Considering this supervening event, the
insurance proceeds shall inure to the benefit of the heirs of the deceased person or his
beneficiaries. Equity dictates that DBP should not unjustly enrich itself at the expense of another
(Nemo cum alterius detrimenio protest). Hence, it cannot collect the insurance proceeds, after it
already foreclosed on the mortgage. The proceeds now rightly belong to Dr. Leuterios heirs
represented by his widow, herein private respondent Medarda Leuterio.

QUISUMBING, J.:
FACTS: A contract of group life insurance was executed between petitioner Great Pacific Life
Assurance Corporation (hereinafter Grepalife) and Development Bank of the Philippines
(hereinafter DBP). Grepalife agreed to insure the lives of eligible housing loan mortgagors of DBP.
On November 11, 1983, Dr. Wilfredo Leuterio, a physician and a housing debtor of DBP applied
for membership in the group life insurance plan. In an application form, Dr. Leuterio answered
questions concerning his health condition as follows:
7. Have you ever had, or consulted, a physician for a heart condition, high blood pressure,
cancer, diabetes, lung, kidney or stomach disorder or any other physical impairment?
Answer: No. If so give details ___________.
8. Are you now, to the best of your knowledge, in good health?
Answer: [ x ] Yes [ ] No.
On November 15, 1983, Grepalife issued Certificate No. B-18558, as insurance coverage of Dr.
Leuterio, to the extent of his DBP mortgage indebtedness amounting to eighty-six thousand, two
hundred (P86,200.00) pesos.
On August 6, 1984, Dr. Leuterio died due to massive cerebral hemorrhage. Consequently, DBP
submitted a death claim to Grepalife. Grepalife denied the claim alleging that Dr. Leuterio was
not physically healthy when he applied for an insurance coverage on November 15, 1983.
Grepalife insisted that Dr. Leuterio did not disclose he had been suffering from hypertension,
which caused his death. Allegedly, such non-disclosure constituted concealment that justified the
denial of the claim.
On October 20, 1986, the widow of the late Dr. Leuterio, respondent Medarda V. Leuterio, filed a
complaint with the Regional Trial Court of Misamis Oriental, Branch 18, against Grepalife for
Specific Performance with Damages.5 During the trial, Dr. Hernando Mejia, who issued the
death certificate, was called to testify. Dr. Mejias findings, based partly from the information
given by the respondent widow, stated that Dr. Leuterio complained of headaches presumably
due to high blood pressure. The inference was not conclusive because Dr. Leuterio was not
autopsied, hence, other causes were not ruled out.
On February 22, 1988, the trial court rendered a decision in favor of respondent widow and
against Grepalife. On May 17, 1993, the Court of Appeals sustained the trial courts decision.
8. SOLIMAN V. US LIFE- RESCIND CONTRACT OF INSURANCE (104 PHIL 1046)
FACTS: US Life issued a 20 yr endowment life policy on the joint lives of Patricio Soliman and his
wife Rosario, each of them being the beneficiary of the other. In Mar. 1949, the spouses were
informed that the premium for Jan 1949 was still unpaid notwithstanding that the 31-day grace
period has already expired, and they were furnished at the same time long-form health
certificates for the reinstatement of the policies.

In Apr 1949, they submitted the certificates and paid the premiums. In Jan. 1950, Rosario died of
acute dilation of the heart, and thereafter, Patricio filed a claim for the proceeds of the insurance.
US life denied the claim and filed for the rescission of the contract on the ground that the
certificates failed to disclose that Rosario had been suffering from bronchial asthma for 3 years
prior to their submission.
ISSUE: Whether or not the contract can still be rescinded.
HELD: Yes. The insurer is once again given two years from the date of reinstatement to
investigate into the veracity of the facts represented by the insured in the application for
reinstatement.
When US life sought to rescind the contract on the ground of
concealment/misrepresentation, two years had not yet elapsed. Hence, the contract can still be
rescinded.
[No. 12707. August 10, 1918.]
9. MRS. HENRY E. HARDING and her husband, plaintiffs and
COMMERCIAL UNION ASSURANCE COMPANY, defendant and appellant.

appellees,

vs.

1.INSURANCE; INSURABLE INTEREST.A wife has an insurable interest in the property given
to her by her husband.
2.ID.; WARRANTY; PROPOSAL WRITTEN BY INSURER'S AGENT.Where it appears that the
proposal form, while signed by the insured, was made out by the person authorized to solicit the
insurance, the facts stated in the proposal, even if incorrect, will not be regarded as warranted by
the insured, in the absence of willful misstatement. Under such circumstances the proposal is to
be regarded as the act of the insurer.
3.ID.; VALUED POLICY.The valuation in a policy of fire insurance is conclusive in the absence
of fraud.
FISHER, J.:
FACTS: "It is alleged by plaintiffs and admitted by defendant that plaintiffs are husband and wife
and residents of the city of Manila; that the defendant is a foreign corporation organized and
existing under and by virtue of the laws of Great Britain and duly registered in the Philippine
Islands, and Smith, Bell & Co. (limited), a corporation organized and existing under the laws of
the' Philippine Islands, with its principal domicile in the city of Manila, is the agent in the
Philippine Islands of said defendant.
"The plaintiffs alleged that on February 16, 1916, the plaintiff Mrs. Henry E. Harding was the
owner of a Studebaker automobile, registered number 2063, in the city of Manila; that on said
date, in consideration of the payment to the defendant of the premium of P150, by said plaintiff,
Mrs. Henry E. Harding, with the consent of her husband, the defendant by its duly authorized
agent, Smith, Bell & Gompany (limited), made its policy of insurance in writing upon said
automobile in the sum of P3,000 and that the value of said automobile was set forth in said
policy (Exhibit A) to be f%000; that on March 24, 1916, said auto mobile was totally destroyed by
fire; that the loss thereby to plaintiffs was the sum of P3,000; that thereafter, within the period
mentioned in the said policy of insurance, the plaintiff, Mrs. Henry E. Harding, furnished the
defendant the proofs of her said loss and interest, and otherwise performed all the conditions of
said policy on her part, and that the defendant has not paid said loss nor any part thereof,
although due demand was made upon defendant therefor.
"The defendant, by its answer, admitted the allegations of the residence and status of the parties
and denied all the other allegation of the said complaint, and for a separate and affirmative
defense alleged (1) that on February 17, 1916, at the city of Manila, P. L, the defendant upon

request of plaintiff, Mrs. Henry E. Harding, issued to the said plaintiff the policy of insurance on
an automobile alleged by the said plaintiff to be her property; that the said request for the
issuance of said policy of insurance was made by means of a proposal in writing signed and
delivered by said plaintiff to the defendant, guaranteeing the truth of the statements contained
therein which said proposal is referred to in the said policy of insurance and made a part thereof;
(2) that certain of the statements and representations contained in said proposal and warranted
by said plaintiff to be true, to wit: (a) the price paid by the proposer for the said automobile; (6)
the value of said automobile at the time of the execution and delivery of the said proposal and
(c) the ownership of said automobile, were false and known to be false by the said plaintiff at the
time of signing and delivering the said proposal and were made for the purpose of misleading
and deceiving the defendant, and inducing the defendant to issue the said policy of insurance;
(3) that the defendant, relying upon the warranties, statements, and representations contained
in the said proposal and believing the same to be true, issued the said policy of insurance.
"The evidence in this case shows that some time in the year 1913 Levy Hermanos, the Manila
agents for the Studebaker automobile, sold the automobile No. 2063 to John Canson for P3,200
(testimony of Mr. Diehl) ; that under date of October 14, 1914, John Canson sold the said
automobile to Henry Harding for the sum of P1,500 (Exhibit 2) ; that under date of November 19,
1914, the said Henry Harding sold the said automobile No. 2063 to J. Brannigan, of Los Banos,
Province of Laguna, P. I., for the sum of P2,000 (Exhibit 3) ; that under date of December 20,
1915, J. C. Graham'of Los Banos, Province of Laguna, P. I., sold the said automobile No. 2063 to
Henry Harding of the city of Manila for the sum of P2,800 (Exhibit 4 and testimony of J. C.
Graham); that on or about January 1, 1916, the said Henry Harding gave the said automobile to
his wife, Mrs. Henry E. Harding, one of the plaintiffs, as a present; that said automobile was
repaired and repainted at the Luneta Garage at a cost of some P900 (testimony of Mr. Server);
that while the said automobile was at the Luneta Garage, the said Luneta Garage, acting as
agent for Smith, Bell & Company (limited), solicited of the plaintiff Mrs. Harding the insurance of
said automobile by the defendant Company (testimony of Mrs. Harding and Mr. Server) ; that a
proposal was filled out by the said agent and signed by the plaintiff Mrs. Henry E. Harding, and in
said proposal under the heading Triee paid by proposer' is the amount of '3,500' and, under
another heading "Present value" is the amount of'3,000'.
"The evidence tends to show that after the said proposal was made a representative of the
Manila agent of defendant went to the Luneta Garage and examined said automobile No. 2063
and Mr. Server, the General Manager of the Luneta Gai'age, an experienced automobile
mechanic, testified that at the time this automobile was insured it was worth about P3,000, and
the defendant, by and, through its said agent Smith, Bell & Company (limited), thereafter issued
a policy of insurance upon said proposal, in which policy the said automobile was described as of
the 'present value' of P3,000, and the said defendant charged the said plaintiff Mrs. Henry E.
Harding as premium or. said policy the sum of ?150, or 5 per cent of the then estimated value of
=3,000 (Exhibit A),
"The 'Schedule' in said policy of insurance describes the automobile here in question, and
provides in part as follows:
"'Now it is hereby agreed as follows:
" That during the period above set forth and during any period for which the company may agree
to renew this policy the company will subject to the exception and conditions contained herein or
endorsed hereon indemnify the insured against loss of or damage to any motor car described in
the schedule hereto' (including accessories) by whatever cause such loss or damage may be
occasioned and will further indemnify the insured up to the value of the car or P3,000 whichever
is the greater against any claim at common law made by any person (not being a person in the
said motor car nor in the insured's service) for loss of life or for accidental bodily injury or
damage to property caused by the said motor car including law costs payable in connection with
such claim when incurred with the consent of the company.'

"The evidence further shows that on March 24, 1916, the said automobile was totally destroyed
by fire, and that the iron and steel portions of said automobile which did not burn were taken into
the possession of the defendant by and through its agent Smith, Bell & Company (limited), and
sold by it for a small sum, which had never been tendered to the plaintiff prior to the trial of this
case, but in open court during the trial the sum of P10 as the proceeds of such sale was tendered
to plaintiff and refused."
[No. 47593.December 29, 1943]
10. The Insular Life Assurance Co., Ltd., petitioner, vs. Serafin D. Feliciano et al.,
respondents.
1.Life Insurance; Validity of Policy Containing False Statements Regarding Health of
the Insured.The policies were issued on the basis of the statement subscribed by the
applicant to the effect that he was and had been in good health, when as a matter of fact he was
then suffering from advanced pulmonary tuberculosis. Held: Altho the agent and the medical
examiner knew that statement to be false, no valid contract of insurance was entered into
because there was no real meeting of the minds of the parties.
2.Id.; Id.When Evaristo Feliciano, the applicant for insurance, signed the application in blank
and authorized the soliciting agent and/or the medical examiner of the Company to write the
answers for him, he made them his own agents for that purpose, and he was responsible for their
acts in that connection. If they falsified the answers for him, he could not evade the responsibility
for the falsification. He was not supposed to sign the application in blank. He knew that the
answers to the questions therein, contained would be "the basis of the policy," and for that very
reason he was required with his signature to vouch for the truth thereof.
3.Id.; Id.; Connivance with Soliciting Agent and Medical Examiner.From all the facts and
circumstances of the case, we are constrained to conclude that the insured was a coparticipant,
and coresponsible with Agent David and Medical Examiner Valdez, in the fraudulent procurement
of the policies in question and that by reason thereof said policies are void ab initio.
Ozaeta,J.:
FACTS: Evaristo Feliciano, who died on September 29, 1935, was suffering with advanced
pulmonary tuberculosis when he signed his application for insurance with the petitioner on
October 12, 1934. On that same date Doctor Trepp, who had taken X-ray pictures of his lungs,
informed the respondent Dr. Serafin D. Feliciano, brother of Evaristo, that the latter "was already
in a very serious and practically hopeless condition." Nevertheless the question contained in the
application"Have you ever suffered from any ailment or disease of the lungs, pleurisy,
pneumonia or asthma?" appears to have been answered, "No." And above the signature of the
applicant, following the answers to the various questions propounded to him, is the following
printed statement:
"I declare on behalf of myself and of any person who shall have or claim any interest in any
policy issued hereunder, that each of the above answers is full, complete and true, and that to
the best of my knowledge and belief I am a proper subject for life insurance."
Each of the policies sued upon contains the following stipulations:
"This policy and the application herefor constitute the entire contract between the parties hereto.
*
*
* Only the President, or the Manager, acting jointly with the Secre tary or Assistant
Secretary (and then only in writing signed by them) have power in behalf of the Company to
issue permits, or to modify this or any contract, or to extend the time for making any premium
payment, and the Company shall not be bound by any promise or representation heretofore or
hereafter given by any person other than the above-named officials, and by them only in writing
and signed conjointly as stated."

The application contains, among others, the following statements:


"18.I [the applicant] hereby declare that all the above statements and answers as well as all
those that I may make to the Company's Medical Examiner in continuation of this application, to
be complete, true and correct to the best of my knowledge and belief, and I hereby agree as
follows:
"1.That this declaration, with the answers to be given by me to the Medical Examiner, shall be
the basis of the policy and form part of same.
***********
"3.That the said policy shall not take effect until the first premium has been paid and the policy
has been delivered to and accepted by me, while I am in good health.
"4.That the agent taking this application has no authority to make, modify or discharge
contracts, or to waive any of the Company's rights or requirements.
"5.My acceptance of any policy issued on this application will constitute a ratification by me of
any corrections in or, additions to this application made by the Company in the space provided
'For Home Office Corrections or Additions Only.' I agree that photographic copy of this application
as corrected or added to shall constitute sufficient notice to me of the changes made."
The petitioner insists that upon the facts of the case the policies in question are null and void ab
initio and that all that the respondents are entitled to is the refund of the premiums paid thereon.
After a careful re-examination of the facts and the law, we are persuaded that petitioner's
contention is correct. To the reasons adduced in the dissenting opinion heretofore published, we
only desire to add the following considerations:
When Evaristo Feliciano, the applicant for insurance, signed the application in blank and
authorized the soliciting agent and/or the medical examiner of the Company to write the answers
for him, he made them his own agents for that purpose, and he was responsible for their acts
inxthat connection. If they falsified the answers for him, he could not evade the responsibility for
the falsification. He was not supposed to sign the application in blank. He knew that the answers
to the questions therein contained would be "the basis of the policy," and for that very reason he
was required with his signature to vouch for the truth thereof.
Moreover, from the facts of the case we cannot escape the conclusion that the insured acted in
connivance with the soliciting agent and the medical examiner of the Company in accepting the
policies in question. Above the signature of the applicant is the printed statement or
representation: "* * * I am a proper subject for life insurance." In another sheet of the same
application and above another signature of the applicant was also printed this statement: "That
the said policy shall not take effect until the first premium has been paid and the policy has been
delivered to and accepted by me, while I am in good health." When the applicant signed the
application he was "having difficulty in breathing, * * * with a very high fever." He had gone
three times to the Santol Sanatorium and had X-ray pictures taken of his lungs. He therefore
knew that he was not "a proper subject for life insurance." When he accepted the policy, he knew
that he was not in good health. Nevertheless, he not only accepted the first policy of P20,000 but
then and there applied for and later accepted another policy of P5,000.
We cannot bring ourselves to believe that the insured did not take the trouble to read the
answers contained in the photostatic copy of the application attached to and made a part of the
policy before he accepted it and paid the premium thereon. He must have noticed that the
answers to the questions therein asked concerning his clinical history were false, and yet he
accepted the first policy and applied for another. In any event, he obligated himself to read the
policy when he subscribed to this statement: "My acceptance of any policy issued on this
application will constitute a ratification by me of any corrections in or additions to this application
made by the Company * * *" By accepting the policy he became charged with knowledge of its
contents, whether he actually read it or not. He could not ostrich-like hide his head from it in
order to avoid his part of the bargain and at the same time claim the benefit thereof. He knew, or
was chargeable with knowledge, from the very terms of the two policies sued upon (one of which
is printed in English and the other in Spanish) that the soliciting agent and the medical examiner
had no power to bind the Company by any verbal promise or oral representation. The insured,

therefore, had no right to relyand we cannot believe he relied in good faithupon the oral
representation of said agent and medical examiner that he (the applicant) was a fit subject for
insurance notwithstanding that he had been and was still suffering with advanced pulmonary tu berculosis.
[No. 47593.September 13, 1941]
11. The Insular Life Assurance Co., Ltd., petitioner, vs. Serafin D. Feliciano and Angel,
Florenda, Eugenio, Herminio and Leticia, all surnamed Feliciano, represented by their
guardan ad litem Serafin D. Feliciano, respondents.
1.Insurance; Knowledge by Insurer's Agent of True State of Health of Insured;
Falsification by Insurer's Agent of Answers Given by Insured; Liability of Insurer; Case
at Bar.In the present case, the agent knew all the time the true state of health of the insured.
The insurer's medical examiner approved the application knowing full well that the applicant was
sick. The situation is one in which one of two innocent parties must bear a loss for his reliance
upon a third person. In this case, it was the insurer who gave the agent authority to deal with the
applicant. It was the one who selected the agent, thus implying that the insured could put his
trust on him. It was the one who drafted and accepted the policy and consummated the contract.
Held: That as between the two of them, the one who employed and gave character to the third
person as its agent should be the one to bear the loss.
2.Id.; Id.; Id.; Id.; Id.If an agent of the insurer, after obtaining from an applicant for insurance
a correct and truthful answer to interrogatories contained in the application for insurance,
without knowledge of the applicant fills in false answers, either fraudulently or otherwise, the
insurer cannot assert the falsity of such answers as a defense to liability on the policy, and this is
true generally without regard to the subject matter of the answers or the nature of the agent's
duties or limitations on his authority, at least if not brought to the attention of the applicant.
3.Id.; Id.; Id.; Id.; Id.; Failure of Insured to Read Application.The fact that the insured
did not read the application which he signed, is not indicative of bad faith. It has been held that it
is not negligence for the insured to sign an application without first reading it if the insurer by its
conduct in appointing the agent influenced the insured to place trust and confidence in the
agent. In the instant case, it has been proved that the insured could not read English, the
language in which the application was written, and that after the contract was signed, it was kept
by his mother. As a consequence, the insured had no opportunity to read or correct any
misstatement therein.
Laurel,J.:
FACTS: One Evaristo Feliciano filed an application for insurance with the herein petitioner upon
the solicitation of one of its agents. Two insurance policies to the aggregate amount of P25,000
were issued to him. Feliciano died on September 29, 1935. The defendant company refused to
pay on the ground that the policies were fraudulently obtained, the insured having given false
answers and statements in the application as well as in the medical report. The pre sent action
was brought to recover on said policies. The lower court rendered judgment in favor of the
plaintiffs. The lower court found that at the time Feliciano filed his application and at the time he
was subjected to physical examination by the medical examiner of the herein petitioner, he was
already suffering from tuberculosis. This fact appears in the negative both in the application and
in the medical report. The lower court, after an exhaustive examination of the conflicting
testimonies, also found that Feliciano was made to sign the application and the examiner's report
in blank, and that afterwards the blank spaces therein were filled in by the agent and the medical
examiner, who made it appear therein that Feliciano was a fit subject for insurance. The lower
court also held that neither the insured nor any member of his family concealed the real state of
health of the insured. That as a matter of fact the insured, as well as the members of his family,
told the agent and the medical examiner that the applicant had been sick and coughing for
sometime and that he had also gone three times to the Santol Sanatorium. On appeal, this

finding of facts of the lower court was sustained by the Court of Appeals. This concludes the
controversy over the facts in so far as this Court is concerned.
[No. L-4611. December 17, 1955]
12. QUA CHEE GAN, plaintiff and appellee, vs. LAW UNION AND ROCK INSURANCE Co.,
LTD., represented by its agent, WARNER, BARNES AND Co., LTD., defendant and
appellant.
1.INSURANCE; BREACH OF WARRANTY; WHEN INSURER BARRED FROM CLAIMING
POLICIES VOID AB INITIO."The insurer is barred by estoppel to claim violation of the socalled fire hydrant warranty where, knowing fully well that the number of hydrants demanded in
the warranty never existed from the very beginning, it nevertheless issued the policies subject to
such warranty, and received the corresponding premiums.
2.ID.; ID.; EVIDENCE; PAROL EVIDENCE RULE NOT APPLICABLE.The parol evidence rule
is not applicable to the present case. It is not a question here whether or not the parties may
vary a written contract by oral evidence; but whether testimony is receivable so that a party may
be, by reason of inequitable contract shown, estopped from enforcing forfeitures in its favor, in
order to forestall fraud or imposition on the insured.
3.ID.; AMBIGUITIES IN THE TERMS OF THE CONTRACT, How CONSTRUED.The contract
of insurance is one of perfect good faith (uberrimae fidei) not for the insured alone, but equally
so for the insurer; in fact, it is more so for the latter, since its dominant bargaining position
carries with it stricter responsibility. By reason of the exclusive control of the insurance company
over the terms and phraseology of the insurance contract, the ambiguity must be strictly
interpreted against the insurer and liberally in favor of the insured, specially to avoid a forfeiture
(44 C.J. S., pp. 11661175; 29 Am. Jur. 180).
4.ID.; ID.; WARRANTY AGAINST STORAGE OF GASOLINE.In the present case, gasoline is
not specifically mentioned among the prohibited articles listed in the so-called hemp warranty.
The clause relied upon by the insurer speaks of oils and is decidedly ambiguous and uncertain;
for in ordinary parlance, oils mean lubricants and not gasoline or kerosene. Besides, the
gasoline kept by the insured was only incidental to his business, being no more than a customary
2 days supply for the five or six motor vehicles used for transporting of the stored merchandise,
and it is well settled rule that the keeping of inflammable oils on the premises, though prohibited
by the policy, does not void it if such keeping is incidental to the business. (Bachrach vs. British
American Ass. Co., 17 Phil. 555, 660.)
5.ID. ; FALSE CLAIMS THAT AVOIDS THE POLICY.The rule is that to avoid a policy, the
claim filed by the insured must contain false and fraudulent statements with intent to defraud
the insurer.
6.CRIMINAL PROCEDURE; ACQUITTAL OF INSURED IN ARSON CASE; EFFECT ON CIVIL
ACTION.While the acquittal of the insured in the arson case is not res judicata on the present
civil action, the insurers evidence, to judge from the decision in the criminal case, is practically
identical in both cases and must lead to the same result, since the proof to establish the defense
of connivance at the fire in order to defraud the insurer cannot be materially less convincing
than that required in order to convict the insured of the crime of arson (Bachrach vs. British
American Assurance Co., 17 Phil. 536).

REYES, J.B. L., J.:


FACTS: Qua Chee Gan, a merchant of Albay, instituted this action in 1940, in the Court of First
Instance of said province, seeking to recover the proceeds of certain fire insurance policies

totalling P370,000, issued by the Law Union & Rock Insurance Co., Ltd., through its agent,
Warner, Barnes & Co., Ltd., upon certain bodegas and merchandise of the insured that were
burned on June 21, 1940. The records of the original case were destroyed during the liberation of
the region, and were reconstituted in 1946. After a trial that lasted several years, the Court of
First Instance rendered a decision in favor of the plaintiff.
The record shows that before the last war, plaintiff-appellee owned four warehouses or bodegas
(designated as Bodegas Nos. 1 to 4) in the municipality of Tabaco, Albay, used for the storage of
stocks of copra and of hemp, baled and loose, in which the appellee dealt extensively. They had
been, with their contents, insured with the defendant Company since 1937, and the lose made
payable to the Philippine National Bank as mortgage of the hemp and copra, to the extent of its
interest. On June, 1940, the insurance stood as follows:
Policy No.
Property Insured
Amount
2637164 (Exhibit LL") ...................................
Bodega No. 1 (Building) ............
P15,000.00
2637165 (Exhibit JJ") .....................................
Bodega No. 2 (Building) ............
10,000.00
Bodega No. 3 (Building) ............
25,000.00
Bodega No. 4 (Building) ............
10,000,00
Hemp Pressmoved by
steam engine ..........................................
5,000.00
2637345 (Exhibit X") ..................
Merchandise contents
(copra and empty
sacks of Bodega No. 1) ...........................
150,000.00
2637346 (Exhibit Y") .................
Merchandise contents
(hemp) of Bodega No. 3 .........................
150,000.00
2637067 (Exhibit GG") ...............
Merchandise contents
(loose hemp) of Bodega No. 4 ................
___5,000.00
Total .......................................................................................................
P370,000.00
Fire of undetermined origin that broke out in the early morning of July 21, 1940, and lasted
almost one week, gutted and completely destroyed Bodegas Nos. 1, 3 and\ 4, with the
merchandise stored therein. Plaintiff-appellee informed the insurer by telegram on the same
date; and on the next day, the fire adjusters engaged by appellant insurance company arrived
and proceeded to examine and photograph the premises, pored over the books of the insured
and conducted an extensive investigation. The plaintiff having submitted the corresponding fire
claims, totalling P398,562.81 (but reduced to the full amount of the insurance, P370,000), the
Insurance Company resisted payment, claiming violation of warranties and conditions, filing of

fraudulent claims, and that the fire had been deliberately caused by the insured or by other
persons in connivance with him.
With counsel for the insurance company acting as private prosecutor, Qua Chee Gan, with his
brother, Qua Chee Pao, and some employees of his, were indicted and tried in 1940 for the crime
of arson, it being claimed that they had set fire to the destroyed warehouses to collect the
insurance. They were, however, acquitted by the trial court in a final decision dated July 9, 1941.
INCONTESTABILITY (SECTION 48)
[No. 27541. November 21, 1927]
1. TAN CHAY HENG, plaintiff and appellee, vs. THE WEST COAST LIFE INSURANCE
COMPANY, defendant and appellant.
NATURE OF ACTION TO RESCIND.An action to rescind a contract is founded upon and
presupposes the existence of the contract which is sought to be rescinded.
WHEN SECTION 47 OF INSURANCE ACT is NOT A BAR.A defense to an action to recover
insurance that the policy was obtained through false representations, fraud and deceit is not in
the nature of an action to rescind and, hence, is not barred by section 47 of the Insurance Act.
NATURE OF THAT DEFENSE.A defense of that nature is founded upon the theory that,
through fraud in its execution, the policy is void ab initio, and that no valid contract was ever
made.
FACTS: Plaintiff alleges that he is of age and a resident of Bacolod, Occidental Negros; that the
defendant is a foreign insurance corporation duly organized by the laws of the Philippines to
engage in the insurance business, its main office of which is in the City of Manila; that in the
month of April, 1925, on his application the defendant accepted and approved a life insurance
policy on Tan Caeng for the sum of ?10,000 in which the plaintiff was the sole beneficiary; that
the policy was issued upon the payment by the said Tan Caeng of the first year's premium
amounting to P936; that in and by its terms, the defendant agreed to pay the plaintiff as
beneficiary the amount of the policy upon the receipt of the proofs of the death of the insured
while the policy was in force; that without any premium being due or unpaid, Tan Caeng died on
May 10, 1925; that in June, 1925, plaintiff submitted the proofs of the death of Tan Caeng with a
claim for the payment of the policy which the defendant refused to pay, for which he prays for a
corresponding judgment, with legal interest from the date of the policy, and costs.
In February, 1926, the defendant filed an answer to the complaint in which it made a general and
specific denial, and then announced its intention to file an amended answer, alleging special
defense, and on August 31, 1926.
"By way of special defense, defendant alleges:
"That the insurance policy on the life of Tan Caeng, upon which plaintiff's action is based, was
obtained by the plaintiff in confabulation with one Go Chulian, of Bacolod, Negros Occidental;
Francisco Sanchez of the same place; and Dr. V. S. Locsin, of La Carlota, Negros Occidental, thru
fraud and deceit perpetrated against this defendant in the following manner, to wit:
"1. That on or about the 22d day of February, 1925, in the municipality of Pulupandan, Occidental
Negros, the present plaintiff and the said Go Chulian, Francisco Sanchez and Dr. V. S. Locsin,
conspiring and confederating together for the purpose of defrauding and cheating the defendant
in the sum of P10,000, caused one Tan Caeng to sign an application for insurance with the
defendant in the sum of P10,000, in which application it was falsely represented to the defendant
that the said Tan Caeng was single and was a merchant, and that the plaintiff Tan Chai Heng, the
beneficiary, was his nephew, whereas in truth and in fact, and as the plaintiff and his said
coconspirators well knew, the said Tan Caeng was not single but was legally married to Marcelina
Patalita with whom he had several children; and that he was not a merchant but was a mere

employee of another Chinaman by the name of Tan Quina from whom he received only a meager
salary, and that the present plaintiff was not a nephew of the said Tan Caeng.
"2. That on said date, February 22, 1925, the said Tan Caeng was seriously ill, suffering from
pulmonary tuberculosis of about three years' duration, which illness was incurable and was well
known to the plaintiff and his said coconspirators.
"3. That on or about the same date, February 22, 1925, the said Dr. V. S. Locsin, in his capacity as
medical examiner for the defendant insurance company, pursuant to the conspiracy above
mentioned, prepared and falsified the necessary medical certificate, in which it was made to
appear, among other things, that the said Tan Caeng had never used morphine, cocaine or any
other drug; that he was then in good health and had never consulted any physician; that he had
never spit blood; and that there was no sign of either present or past disease of his lungs;
whereas in truth and in fact, as the plaintiff and his said coconspirators well knew, the said Tan
Caeng was addicted to morphine, cocaine, and opium and had been convicted and imprisoned
therefor, and was then, and for about three years prior thereto had been suffering from
pulmonary tuberculosis.
"4. That on or about the same date, to wit, February 22, 1925, the plaintiff and his said
coconspirators, pursuant to the conspiracy above mentioned, caused a confidential report to the
defendant insurance company to be signed by one V. Sy Yock Kian, who was an employee of Go
Chulian, in which confidential report, among other things, it was falsely represented to the
defendant insurance company that the said Tan Caeng was worth about P40,000, had an annual
income of from eight to ten thousand pesos net, had the appearance of good health, and never
had tuberculosis; that the plaintiff and his said coconspirators well knew that said
representations were false; and that they were made for the purpose of deceiving the defendant
and inducing it to accept the said application for insurance.
"5. That after the said application for insurance, medical certificate and confidential report had
been prepared and falsified, as aforesaid, the plaintiff and his said coconspirators caused the
same to be forwarded to the defendant at its office in Manila, the medical certificate thru the said
Dr. V. S. Locsin as medical examiner, and said application for insurance and confidential report
thru the said Francisco Sanchez in his capacity as one of the agents of the defendant insurance
company in the Province of Occidental Negros; that the defendant, believing that the
representations made in said document were true, and relying thereon, provisionally accepted
the said application for insurance on the life of Tan Caeng in the sum of P10,000 and issued a
temporary policy pending the final approval or disapproval of said application by defendant's
homeoffice in San Francisco, California, where in case of approval a permanent policy was to be
issued; that such permanent policy was never delivered to the plaintiff because defendant
discovered the fraud before its delivery.
"6. That the first agreed annual premium on the insurance in question of P936.50 not having
been paid within sixty (60) days after the date of the supposed medical examination of the
applicant as required by the regulations of the defendant insurance company, of which
regulations the said Francisco Sanchez as agent of the defendant had knowledge, the plaintiff
and his said coconspirators in order to secure the delivery to them of said temporary policy, and
in accordance with said regulations of the defendant company, caused the said Tan Caeng on
April 10, 1925 to sign the a document.
"7. That on May 10, 1925, that is to say, two months and a half after the supposed medical
examination above referred to, and exactly one month after the date of the health certificate for
reinstatement above set forth, the said Tan Caeng died in Valladolid, Occidental Negros, of
pulmonary tuberculosis, the same illness from which he was suffering at the time it is supposed
he was examined by Dr. V. S. Locsin, but that the plaintiff and his said coconspirators, pursuant
to their conspiracy, caused the said Dr. V. S. Locsin to state falsely in the certificate of death that
the said Tan Caeng had died of cerebral hemorrhage.

"That the plaintiff Tan Chai Heng, on the dates hereinabove mentioned, was, like V. Sy Yock Kian
who signed the confidential report above mentioned, an employee of the said Go Chulian; that
the latter was the ringleader of a gang of malefactors who, during, and for some years previous
to the dates above mentioned, were engaged in the illicit enterprise of procuring fraudulent life
insurances from the present defendant, similar to the one in question, and which enterprise was
capitalized by him by furnishing the funds with which to pay the premium on said fraudulent
insurance; that the said Go Chulian was the one who furnished the money with which to pay the
first and only annual premium on the insurance here in question, amounting to P936.50; that the
said Go Chulian, on August 28, 1926, was convicted by the Court of First Instance of the City of
Manila, in criminal case No. 31425 of that court, of the crime of falsification of private documents
in connection with a fraudulent insurance, similar to the present, committed against this
defendant in the month of September, 1924; that in the same case the said Francisco Sanchez
was one of the coaccused of the said Go Chulian but was discharged from the complaint,
because he offered himself and was utilized as a state's witness; that there is another civil action
now pending against Go Chulian and Sanchez in the Court of First Instance of Manila (civil case
No. 28680), in which the present defendant is the plaintiff, for the recovery of the amounts of two
insurance policies aggregating P19,000, fraudulently obtained by the said Go Chulian and
Sanchez upon the lives of one Tan Deco, who was also suffering from and died of tuberculosis,
and one Tan Anso, who was suffering from and died of beriberi.
[No. 28499. March 19; 1928]
2. BERNARDO ARGENTE, plaintiff and appellant, vs. WEST COAST LIFE INSURANCE Co.,
defendant and appellee.
1.INSURANCE; CONCEALMENT AS GROUND FOR RESCISSION OF CONTRACT OF
INSURANCE; SECTION 25 OF THE INSURANCE ACT APPLIED AND CONSTRUED.One
ground for the rescission of a contract of insurance under the Insurance Act is "a concealment/'
which in section 25 is defined as "A neglect to communicate that which a party knows and ought
to communicate." Applied to the facts, it is held that the concealment was material and sufficient
to avoid the policy. It can fairly be assumed that had the true facts been disclosed by the
assured, the insurance would never have been granted.
2.ID. ; ID. ; ID.The basis of the rule vitiating the contract in cases of concealment is that it
misleads or deceives the insurer into accepting the risk, or accepting it at the rate of premium
agreed upon. The insurer, relying upon the belief that the assured will disclose every material f
act within his actual or presumed knowledge, is misled into a belief that the circumstance
withheld does not exist, and he is thereby induced to estimate the risk upon a false basis that it
does not exist. The principal question, therefore, must be, Was the assurer misled or deceived
into entering a contract obligation or in fixing the premium of insurance by a withholding of
material information or facts within the assured's knowledge or presumed knowledge?
3.ID. ; ID. ; SECTION 47 OF THE INSURANCE ACT APPLIED AND CONSTRUED.Section 47
of the Insurance Act providing "Whenever a right to rescind a contract of insurance is given to
the insurer by any provision of this chapter, such right must be exercised previous to the
commencement of an action on the contract" was derived from section 2583 of the California
Civil Code, but in contrast thereto, makes use of the imperative "must" instead of the permissive
"may."
4.ID. ; ID. ; ID.A failure to exercise the right of rescission cannot prejudice any defense to the
action which the concealment may furnish.
5.ID. ; ID. ; ID.Where any of the material representations are false, the insurer's tender of the
premium and notice that the policy is canceled, before the commencement of suit thereon,
operate to rescind the contract of insurance, and are a sufficient compliance with the law.
MALCOLM, J.:

FACTS: This is an action upon a joint life insurance policy for P15,000 issued by the defendant,
the West Coast Life Insurance Co., on May 15, 1925, in favor of the plaintiff, Bernardo Argente,
and his wife, Vicenta de Ocampo, the latter having died on November 18, 1925. Fraud in
obtaining the policy was pleaded by way of special defense. On the issue thus suggested, the
court adopted the theory of the defendant, and held the insurance policy null and void, with the
result that the complaint was dismissed, with costs.
On February 9, 1925, Bernardo Argente signed an application for joint insurance with his wife in
the sum of P2,000. The wife, Vicenta de Ocampo, signed a like application for the same policy.
Both applications, with the exception of the names and the signatures of the applicants, were
written by Jose Geronimo del Rosario, an agent for the West Coast Life Insurance Co. But all the
information contained in the applications was furnished the agent by Bernardo Argente.
Pursuant to his application, Bernardo Argente was examined by Dr. Cesareo Sta. Ana, a medical
examiner for the West Coast Life Insurance Co., on February 10, 1925, in the office of the
Customs House. The result of such examination was recorded in the Medical Examiner's Report,
and with the exception of the signature of Bernardo Argente, was in the hand-writing of Doctor
Sta. Ana. But the information or answers to the questions contained on the face of the Medical
Examiner's Report were furnished the doctor by the applicant, Bernardo Argente.
Pursuant to her application, Vicenta de Ocampo, wife of the plaintiff, was examined by Dr.
Cesareo Sta. Ana on February 10, 1925, at her residence in Manila. The result of the medical
examination, including, among other things, the answers given by Vicenta de Ocampo to the
questions propounded, to her by the physician, appears in the Medical Examiner's Report.
On May 9, 1925, Bernardo Argente and his wife submitted to the West Coast Life Insurance Co.
an amended application for insurance, increasing the amount thereof to P15,000, and asked that
the policy be dated May 15, 1925. The amended application was accompanied by the documents
entitled "Short Form Medical Report." In both of these documents appear certain questions and
answers.
A temporary policy for P15,000 was issued to Bernardo Argente and his wife as of May 15, 1925,
but it was not delivered to Bernardo Argente until July 2, 1925, when the first quarterly premium
on the policy was paid. In view of the fact that more than thirty days had elapsed since the
applicants were examined by the company's physician, each of them was required to file a
certificate of health before the policy was delivered to them.
On November 18, 1925, Vicenta de Ocampo died of cerebral apoplexy. Thereafter Bernardo
Argente presented a claim in due form to the West Coast Life Insurance Co. for the payment of
the sum of P15,000 the amount of the joint life insurance policy. Following investigation
conducted by the Manager of the Manila office of the insurance company, it was apparently
disclosed that the answers given by the insured in their medical examinations with regard to
their health and previous illnesses and medical attendance were untrue. For that reason, the
West Coast Life Insurance Co. refused to pay the claim of Bernardo Argente, and 021 May 25,
1926, wrote him to the effect that the claim was rejected because the insurance was obtained
through fraud and misrepresentation.
G.R. No. 125678. March 18, 2002.
3. PHILAMCARE HEALTH SYSTEMS, INC., petitioner, vs. COURT OF APPEALS and JULITA
TRINOS, respondents.
Insurance; Elements; Words and Phrases; A contract of insurance is an agreement
whereby one undertakes for a consideration to indemnify another against loss,
damage or liability arising from an unknown or contingent event.Section 2 (1) of the
Insurance Code defines a contract of insurance as an agreement whereby one undertakes for a
consideration to indemnify another against loss, damage or liability arising from an unknown or

contingent event. An insurance contract exists where the following elements concur: 1. The
insured has an insurable interest; 2. The insured is subject to a risk of loss by the happening of
the designated peril; 3. The insurer assumes the risk; 4. Such assumption of risk is part of a
general scheme to distribute actual losses among a large group of persons bearing a similar risk;
and 5. In consideration of the insurers promise, the insured pays a premium.
Same; Every person has an insurable interest in the life and health of himself.Section
3 of the Insurance Code states that any contingent or unknown event, whether past or future,
which may damnify a person having an insurable interest against him, may be insured against.
Every person has an insurable interest in the life and health of himself. Section 10 provides:
Every person has an insurable interest in the life and health: (1) of himself, of his spouse and of
his children; (2) of any person on whom he depends wholly or in part for education or support, or
in whom he has a pecuniary interest; (3) of any person under a legal obligation to him for the
payment of money, respecting property or service, of which death or illness might delay or
prevent the performance; and (4) of any person upon whose life any estate or interest vested in
him depends.
Same; Health Care Agreements; A health care agreement is in the nature of non-life
insurance, which is primarily a contract of indemnity.In the case at bar, the insurable
interest of respondents husband in obtaining the health care agreement was his own health. The
health care agreement was in the nature of non-life insurance, which is primarily a contract of
indemnity. Once the member incurs hospital, medical or any other expense arising from sickness,
injury or other stipulated contingent, the health care provider must pay for the same to the
extent agreed upon under the contract.
Same; Same; Misrepresentation; Where matters of opinion are called for, answers
made in good faith and without intent to deceive will not avoid a policy even though
they are untrue.The answer assailed by petitioner was in response to the question relating to
the medical history of the applicant. This largely depends on opinion rather than fact, especially
coming from respondents husband who was not a medical doctor. Where matters of opinion or
judgment are called for, answers made in good faith and without intent to deceive will not avoid
a policy even though they are untrue. Thus, (A)lthough false, a representation of the expectation,
intention, belief, opinion, or judgment of the insured will not avoid the policy if there is no actual
fraud in inducing the acceptance of the risk, or its acceptance at a lower rate of premium, and
this is likewise the rule although the statement is material to the risk, if the statement is
obviously of the foregoing character, since in such case the insurer is not justified in relying upon
such statement, but is obligated to make further inquiry. There is a clear distinction between
such a case and one in which the insured is fraudulently and intentionally states to be true, as a
matter of expectation or belief, that which he then knows, to be actually untrue, or the
impossibility of which is shown by the facts within his knowledge, since in such case the intent to
deceive the insurer is obvious and amounts to actual fraud. (Italics ours)
Same; Same; Concealment; Concealment as a defense for the health care provider or
insurer to avoid liability is an affirmative defense and the duty to establish such
defense by satisfactory and convincing evidence rests upon the provider or insurer;
The liability of the health care provider attaches once the member is hospitalized for
the disease or injury covered by the agreement or whenever he avails of the covered
benefits which he has prepaid.The fraudulent intent on the part of the insured must be
established to warrant rescission of the insurance contract. Concealment as a defense for the
health care provider or insurer to avoid liability is an affirmative defense and the duty to
establish such defense by satisfactory and convincing evidence rests upon the provider or
insurer. In any case, with or without the authority to investigate, petitioner is liable for claims
made under the contract. Having assumed a responsibility under the agreement, petitioner is
bound to answer the same to the extent agreed upon. In the end, the liability of the health care
provider attaches once the member is hospitalized for the disease or injury covered by the
agreement or whenever he avails of the covered benefits which he has prepaid.

Same; Same; Same; Rescission; The right to rescind should be exercised previous to
the commencement of an action on the contract.Under Section 27 of the Insurance Code,
a concealment entitles the injured party to rescind a contract of insurance. The right to rescind
should be exercised previous to the commencement of an action on the contract. In this case, no
rescission was made. Besides, the cancellation of health care agreements as in insurance policies
require the concurrence of the following conditions: 1. Prior notice of cancellation to insured; 2.
Notice must be based on the occurrence after effective date of the policy of one or more of the
grounds mentioned; 3. Must be in writing, mailed or delivered to the insured at the address
shown in the policy; 4. Must state the grounds relied upon provided in Section 64 of the
Insurance Code and upon request of insured, to furnish facts on which cancellation is based.
Same; Same; Contracts; The rule that by reason of the exclusive control of the
insurance company over the terms and phraseology of the insurance contract,
ambiguity must be strictly interpreted against the insurer and liberally in favor of the
insured, especially to avoid forfeiture, is equally applicable to Health Care
Agreements.None of the above preconditions was fulfilled in this case. When the terms of
insurance contract contain limitations on liability, courts should construe them in such a way as
to preclude the insurer from non-compliance with his obligation. Being a contract of adhesion,
the terms of an insurance contract are to be construed strictly against the party which prepared
the contractthe insurer. By reason of the exclusive control of the insurance company over the
terms and phraseology of the insurance contract, ambiguity must be strictly interpreted against
the insurer and liberally in favor of the insured, especially to avoid forfeiture. This is equally
applicable to Health Care Agreements. The phraseology used in medical or hospital service
contracts, such as the one at bar, must be liberally construed in favor of the subscriber, and if
doubtful or reasonably susceptible of two interpretations the construction conferring coverage is
to be adopted, and exclusionary clauses of doubtful import should be strictly construed against
the provider.
Same; Same; Since a health care agreement is in the nature of a contract of
indemnity, payment should be made to the party who incurred the expenses.
Petitioner alleges that respondent was not the legal wife of the deceased member considering
that at the time of their marriage, the deceased was previously married to another woman who
was still alive. The health care agreement is in the nature of a contract of indemnity. Hence,
payment should be made to the party who incurred the expenses. It is not controverted that
respondent paid all the hospital and medical expenses. She is therefore entitled to
reimbursement. The records adequately prove the expenses incurred by respondent for the
deceaseds hospitalization, medication and the professional fees of the attending physicians.
YNARES-SANTIAGO, J.:
FACTS: Ernani Trinos, deceased husband of respondent Julita Trinos, applied for a health care
coverage with petitioner Philamcare Health Systems, Inc. In the standard application form, he
answered no to the following question:
Have you or any of your family members ever consulted or been treated for high blood pressure,
heart trouble, diabetes, cancer, liver disease, asthma or peptic ulcer? (If Yes, give details).
The application was approved for a period of one year from March 1, 1988 to March 1, 1989.
Accordingly, he was issued Health Care Agreement No. P010194. Under the agreement,
respondents husband was entitled to avail of hospitalization benefits, whether ordinary or
emergency, listed therein. He was also entitled to avail of out-patient benefits such as annual
physical examinations, preventive health care and other out-patient services.
Upon the termination of the agreement, the same was extended for another year from March 1,
1989 to March 1, 1990, then from March 1, 1990 to June 1, 1990. The amount of coverage was
increased to a maximum sum of P75,000.00 per disability.

During the period of his coverage, Ernani suffered a heart attack and was confined at the Manila
Medical Center (MMC) for one month beginning March 9, 1990. While her husband was in the
hospital, respondent tried to claim the benefits under the health care agreement. However,
petitioner denied her claim saying that the Health Care Agreement was void. According to
petitioner, there was a concealment regarding Ernanis medical history. Doctors at the MMC
allegedly discovered at the time of Ernanis confinement that he was hypertensive, diabetic and
asthmatic, contrary to his answer in the application form. Thus, respondent paid the
hospitalization expenses herself, amounting to about P76,000.00.
After her husband was discharged from the MMC, he was attended by a physical therapist at
home. Later, he was admitted at the Chinese General Hospital. Due to financial difficulties,
however, respondent brought her husband home again. In the morning of April 13, 1990, Ernani
had fever and was feeling very weak. Respondent was constrained to bring him back to the
Chinese General Hospital where he died on the same day.
On July 24, 1990, respondent instituted with the Regional Trial Court of Manila, Branch 44, an
action for damages against petitioner and its president, Dr. Benito Reverente, which was
docketed as Civil Case No. 90-53795. She asked for reimbursement of her expenses plus moral
damages and attorneys fees.
4. SOLIMAN V. US LIFE (104 PHIL 1046)
-Refer to Case No. 8 in Concealment and Representation
G.R. No. 48049. June 29, 1989.
5. EMILIO TAN, JUANITO TAN, ALBERTO TAN and ARTURO TAN, petitioners, vs. THE
COURT OF APPEALS and THE PHILIPPINE AMERICAN LIFE INSURANCE COMPANY,
respondents.
Commercial Law; Insurance; Essence of the phrase Incontestability clause.The socalled incontestability clause precludes the insurer from raising the defenses of false
representations or concealment of material facts insofar as health and previous diseases are
concerned if the insurance has been in force for at least two years during the insureds lifetime.
The phrase during the lifetime found in Section 48 simply means that the policy is no longer
considered in force after the insured has died. The key phrase in the second paragraph of Section
48 is for a period of two years.
Same; Same; Same; Respondent company not barred from proving that the policy is
void ab initio by reason of the insureds fraudulent concealment or misrepresentation.
As noted by the Court of Appeals, to wit: The policy was issued on November 6, 1973 and the
insured died on April 26, 1975. The policy was thus in force for a period of only one year and five
months. Considering that the insured died before the two-year period had lapsed, respondent
company is not, therefore, barred from proving that the policy is void ab initio by reason of the
insureds fraudulent concealment or misrepresentation. Moreover, respondent company
rescinded the contract of insurance and refunded the premiums paid on September 11, 1975,
previous to the commencement of this action on November 27, 1975.
Same; Same; Same; Incontestability clause is a sufficient answer to the various tactics
employed by insurance companies to avoid liability.The insurer has two years from the
date of issuance of the insurance contract or of its last reinstatement within which to contest the
policy, whether or not, the insured still lives within such period. After two years, the defenses of
concealment or misrepresentation, no matter how patent or well founded, no longer lie. Congress
felt this was a sufficient answer to the various tactics employed by insurance companies to avoid
liability. The petitioners interpretation would give rise to the incongruous situation where the
beneficiaries of an insured who dies right after taking out and paying for a life insurance policy,
would be allowed to collect on the policy even if the insured fraudulently concealed material
facts.

GUTIERREZ, JR., J.:


FACTS: This is a petition for review on certiorari of the Court of Appeals decision affirming the
decision of the Insurance Commissioner which dismissed the petitioners complaint against
respondent Philippine American Life Insurance Company for the recovery of the proceeds from
their late fathers policy.
The facts of the case as found by the Court of Appeals are:
Petitioners appeal from the Decision of the Insurance Commissioner dismissing herein
petitioners complaint against respondent Philippine American Life Insurance Company for the
recovery of the proceeds of Policy No. 1082467 in the amount of P80,000.00.
On September 23, 1973, Tan Lee Siong, father of herein petitioners, applied for life insurance in
the amount of P80,000.00 with respondent company. Said application was approved and Policy
No. 1082467 was issued effective November 6, 1973, with petitioners the beneficiaries thereof
(Exhibit A).
On April 26, 1975, Tan Lee Siong died of hepatoma (Exhibit B). Petitioners then filed with
respondent company their claim for the proceeds of the life insurance policy. However, in a letter
dated September 11, 1975, respondent company denied petitioners claim and rescinded the
policy by reason of the alleged misrepresentation and concealment of material facts made by the
deceased Tan Lee Siong in his application for insurance (Exhibit 3). The premiums paid on the
policy were thereupon refunded.
Alleging that respondent companys refusal to pay them the proceeds of the policy was
unjustified and unreasonable, petitioners filed on November 27, 1975, a complaint against the
former with the Office of the Insurance Commissioner, docketed as I.C. Case No. 218.
After hearing the evidence of both parties, the Insurance Commissioner rendered judgment on
August 9, 1977, dismissing petitionerscomplaint.
G.R. No. 175666.July 29, 2013.
6. MANILA BANKERS LIFE INSURANCE CORPORATION, petitioner, vs. CRESENCIA P.
ABAN, respondent.
Insurance Law; Fraud; Fraudulent intent on the part of the insured must be
established to entitle the insurer to rescind the contract.Allegations of fraud, which are
predicated on respondents alleged posing as Sotero and forgery of her signature in the
insurance application, are at once belied by the trial and appellate courts finding that Sotero
herself took out the insurance for herself. [F]raudulent intent on the part of the insured must be
established to entitle the insurer to rescind the contract. In the absence of proof of such
fraudulent intent, no right to rescind arises.
Same; Incontestability Clause; An insurer is given two years from the effectivity of
a life insurance contract and while the insured is alive to discover or prove that the
policy is void ab initio or is rescindible by reason of the fraudulent concealment or
misrepresentation of the insured or his agent.Section 48 serves a noble purpose, as it regulates
the actions of both the insurer and the insured. Under the provision, an insurer is given two years
from the effectivity of a life insurance contract and while the insured is alive to discover or
prove that the policy is void ab initio or is rescindible by reason of the fraudulent concealment or
misrepresentation of the insured or his agent. After the two-year period lapses, or when the
insured dies within the period, the insurer must make good on the policy, even though the policy
was obtained by fraud, concealment, or misrepresentation. This is not to say that insurance fraud
must be rewarded, but that insurers who recklessly and indiscriminately solicit and obtain
business must be penalized, for such recklessness and lack of discrimination ultimately work to
the detriment of bona fide takers of insurance and the public in general.

Same; Insurance Business; Insurers cannot be allowed to collect premiums on


insurance policies, use these amounts collected and invest the same through the
years, generating profits and returns therefrom for their own benefit, and thereafter
conveniently deny insurance claims by questioning the authority or integrity of their
own agents or the insurance policies they issued to their premium-paying clients.If
insurers cannot vouch for the integrity and honesty of their insurance agents/salesmen and the
insurance policies they issue, then they should cease doing business. If they could not properly
screen their agents or salesmen before taking them in to market their products, or if they do not
thoroughly investigate the insurance contracts they enter into with their clients, then they have
only themselves to blame. Otherwise said, insurers cannot be allowed to collect premiums on
insurance policies, use these amounts collected and invest the same through the years,
generating profits and returns therefrom for their own benefit, and thereafter conveniently deny
insurance claims by questioning the authority or integrity of their own agents or the insurance
policies they issued to their premium-paying clients. This is exactly one of the schemes which
Section 48 aims to prevent.
Same; Same; Contract of Adhesion; An insurance contract is a contract of adhesion
which must be construed liberally in favor of the insured and strictly against the
insurer in order to safeguard the formers interest.Insurers may not be allowed to delay
the payment of claims by filing frivolous cases in court, hoping that the inevitable may be put off
for years or even decades by the pendency of these unnecessary court cases. In the
meantime, they benefit from collecting the interest and/or returns on both the premiums
previously paid by the insured and the insurance proceeds which should otherwise go to their
beneficiaries. The business of insurance is a highly regulated commercial activity in the country,
and is imbued with public interest. [A]n insurance contract is a contract of adhesion which must
be construed liberally in favor of the insured and strictly against the insurer in order to safeguard
the [formers] interest.
DEL CASTILLO,J.:
FACTS: The ultimate aim of Section 48 of the Insurance Code is to compel insurers to solicit
business from or provide insurance coverage only to legitimate and bona fide clients, by
requiring them to thoroughly investigate those they insure within two years from effectivity of
the policy and while the insured is still alive. If they do not, they will be obligated to honor claims
on the policies they issue, regardless of fraud, concealment or misrepresentation. The law
assumes that they will do just that and not sit on their laurels, indiscriminately soliciting and
accepting insurance business from any Tom, Dick and Harry.
On July 3, 1993, Delia Sotero (Sotero) took out a life insurance policy from Manila Bankers Life
Insurance Corporation (Bankers Life), designating respondent Cresencia P. Aban (Aban), her
niece,5 as her beneficiary.
Petitioner issued Insurance Policy No. 747411 (the policy), with a face value of P100,000.00, in
Soteros favor on August 30, 1993, after the requisite medical examination and payment of the
insurance premium.
On April 10, 1996,7 when the insurance policy had been in force for more than two years and
seven months, Sotero died. Respondent filed a claim for the insurance proceeds on July 9, 1996.
Petitioner conducted an investigation into the claim,8 and came out with the following findings:
1.Sotero did not personally apply for insurance coverage, as she was illiterate;
2.Sotero was sickly since 1990;
3.Sotero did not have the financial capability to pay the insurance premiums on Insurance
Policy No. 747411;
4.Sotero did not sign the July 3, 1993 application for insurance; [and]
5.Respondent was the one who filed the insurance application, and x x x designated herself as
the beneficiary.10

For the above reasons, petitioner denied respondents claim on April 16, 1997 and refunded the
premiums paid on the policy.
On April 24, 1997, petitioner filed a civil case for rescission and/or annulment of the policy, which
was docketed as Civil Case No. 97-867 and assigned to Branch 134 of the Makati Regional Trial
Court. The main thesis of the Complaint was that the policy was obtained by fraud, concealment
and/or misrepresentation under the Insurance Code,12 which thus renders it voidable under
Article 139013 of the Civil Code.
Respondent filed a Motion to Dismiss14 claiming that petitioners cause of action was barred by
prescription pursuant to Section 48 of the Insurance Code, which provides as follows:
Whenever a right to rescind a contract of insurance is given to the insurer by any provision of
this chapter, such right must be exercised previous to the commencement of an action on the
contract.
After a policy of life insurance made payable on the death of the insured shall have been in force
during the lifetime of the insured for a period of two years from the date of its issue or of its last
reinstatement, the insurer cannot prove that the policy is void ab initio or is rescindible by reason
of the fraudulent concealment or misrepresentation of the insured or his agent.
During the proceedings on the Motion to Dismiss, petitioners investigator testified in court,
stating among others that the insurance underwriter who solicited the insurance is a cousin of
respondents husband, Dindo Aban,15 and that it was the respondent who paid the annual
premiums on the policy.
THE POLICY (SECTIONS 49-66 AND SECTIONS 232 AND 233)
[No. 15895. November 29, 1920.]
1. RAFAEL ENRIQUEZ, as administrator of the estate of the late Joaquin 'Ma. Herrer,
plaintiff and appellant, vs. SUN LIFE ASSURANCE COMPANY OF CANADA, defendant
and appellee.
1.INSURANCE; PHILIPPINE LAW.The law of insurance is now found in the Insurance Act and
the Civil Code.
2.ID. ; OFFER AND ACCEPTANCE.The Civil Code rule, that an acceptance made by letter
shall bind the person making the offer only f rom the date, it came to his knowledge, is
controlling.
3.ID. ; ID.On September 24, 1917, H made application to an insurance company through its
office in Manila for a life annuity. Two days later he paid the sum of P6,000 to the manager of the
company's Manila office and was given a receipt therefor. On November 26, 1917, the head office
gave notice of acceptance by cable to Manila. On the same date the Manila office prepared a
letter notifying H that his application had been accepted and this was placed in the ordinary
channels for transmission, but as far as known, was never actually mailed and was never
received by the applicant. H died on December 20, 1917. Held: That the contract for a life
annuity was not perfected because it had not been proved satisfactorily that the acceptance of
the , application ever came to the knowledge of the applicant.
4.ID. ; ID.An acceptance of an offer of insurance not actually or constructively communicated
to the proposer does not make a contract. Only the mailing of acceptance completes the contract
of insurance, as the locus pnitenti is ended when the acceptance has passed beyond the
control of the party.
5.ID. ; ID. ; MAILING AND DELIVERY OF MAIL MATTER, PRESUMPTION.When a letter or
other mail matter is addressed and mailed with postage prepaid there is a rebuttable

presumption of fact that it was received by the addressee as soon as it could have been
transmitted to him in the ordinary course of the mails. But if any one of these elemental facts
fails to appear, it is fatal to the presumption.
MALCOLM, J.:
FACTS: On September 24, 1917, Joaquin Herrer made application to the Sun Life Assurance
Company of Canada through its office in Manila for a life annuity. Two days later he paid the sum
of P6,000 to the manager of the company's Manila office and was given a receipt.
The application was immediately forwarded to the head office of the company at Montreal,
Canada. On November 26, 1917, the head office gave notice of acceptance by cable to Manila.
(Whether on the same day the cable was received notice was sent by the Manila office to Herrer
that the application had been accepted, is a disputed point, which will be discussed later.) On
December 4, 1917, the policy was issued at Montreal. On December 18, 1917, attorney Aurelio A.
Torres wrote to the Manila office of the company stating that Herrer desired to withdraw his
application. The following day the local office replied to Mr. Torres, stating that the policy had
been issued, and called attention to the notification of November 26, 1917. This letter was
received by Mr. Torres on the morning of December 21, 1917. Mr. Herrer died on December 20,
1917.
As above suggested, the issue of fact raised by the evidence is whether Herrer received notice of
acceptance of his application. To resolve this question, we propose to go directly to the evidence
of record.
The chief clerk of the Manila office of the Sun Life Assurance Company of Canada at the time of
the trial testified that he prepared the letter introduced in evidence as Exhibit 3, of date
November 26, 1917, and handed it to the local manager, Mr. E. E. White, for signature. The
witness admitted on cross-examination that after preparing the letter and giving it to the
manager, he knew nothing of what became of it. The local manager, Mr. White, testified to
having received the cablegram accepting the application of Mr. Herrer from the home office on
November 26, 1917. He said that on the same day he signed a letter notifying Mr. Herrer of this
acceptance. The witness further said that letters, after being signed, were sent to the chief clerk
and placed on the mailing desk f or transmission. The witness could not tell if the letter had ever
actually been placed in the mails. Mr. Tuason, who was the chief clerk, on November 26, 1917,
was not called as a witness.
G.R. No. 112329. January 28, 2000.
2. VIRGINIA A. PEREZ, petitioner, vs. COURT OF APPEALS and BF LIFEMAN INSURANCE
CORPORATION, respondents.
Actions; Contracts; Obligations; A potestative condition depends upon the exclusive
will of one of the parties. For that reason, it is considered void.A potestative condition
depends upon the exclusive will of one of the parties. For this reason, it is considered void. Article
1182 of the New Civil Code states: When the fulfillment of the condition depends upon the sole
will of the debtor, the conditional obligation shall be void. In the case at bar, the following
conditions were imposed by the respondent company for the perfection of the contract of
insurance: (a) a policy must have been issued; (b) the premiums paid; and (c) the policy must
have been delivered to and accepted by the applicant while he is in good health.
Same; Same; Same; The condition is a suspensive one whereby the acquisition of
rights depends upon the happening of an event which constitutes the condition.The
condition imposed by the corporation that the policy must have been delivered to and accepted
by the applicant while he is in good health can hardly be considered as a potestative or
facultative condition. On the contrary, the health of the applicant at the time of the delivery of
the policy is beyond the control or will of the insurance company. Rather, the condition is a
suspensive one whereby the acquisition of rights depends upon the happening of an event which

constitutes the condition. In this case, the suspensive condition was the policy must have been
delivered and accepted by the applicant while he is in good health. There was non-fulfillment of
the condition, however, inasmuch as the applicant was already dead at the time the policy was
issued. Hence, the nonfulfillment of the condition resulted in the non-perfection of the contract.
Same; Same; Same; A contract of insurance must be assented to by both parties
either in person or by their agents.A contract of insurance, like other contracts, must be
assented to by both parties either in person or by their agents. So long as an application for
insurance has not been either accepted or rejected, it is merely an offer or proposal to make a
contract. The contract, to be binding from the date of application, must have been a completed
contract, one that leaves nothing to be done, nothing to be completed, nothing to be passed
upon, or determined, before it shall take effect. There can be no contract of insurance unless the
minds of the parties have met in agreement.
Same; Same; Same; A contract which is null and void is no contract at all and hence
could not be the subject of rescission.A final note. It has not escaped our notice that the
Court of Appeals declared Insurance Policy 056300 for P50,000.00 null and void and rescinded.
The Court of Appeals corrected this in its Resolution of the motion for reconsideration filed by
petitioner, thus: Anent the appearance of the word rescinded in the dispositive portion of the
decision, to which defendant-appellee attaches undue significance and makes capital of, it is
clear that the use of the words and rescinded is, as it is hereby declared, a superfluity. It is
apparent from the context of the decision that the insurance policy in question was found null
and void, and did not have to be rescinded. True, rescission presupposes the existence of a
valid contract. A contract which is null and void is no contract at all and hence could not be the
subject of rescission.
YNARES-SANTIAGO, J.:
FACTS: A contract of insurance, like all other contracts, must be assented to by both parties,
either in person or through their agents and so long as an application for insurance has not been
either accepted or rejected, it is merely a proposal or an offer to make a contract.
Primitivo B. Perez had been insured with the BF Lifeman Insurance Corporation since 1980 for
P20,000.00. Sometime in October 1987, an agent of the insurance corporation, Rodolfo Lalog,
visited Perez in Guinayangan, Quezon and convinced him to apply for additional insurance
coverage of P50,000.00, to avail of the ongoing promotional discount of P400.00 if the premium
were paid annually.
On October 20, 1987, Primitivo B. Perez accomplished an application form for the additional
insurance coverage of P50,000.00. On the same day, petitioner Virginia A. Perez, Primitivos wife,
paid P2,075.00 to Lalog. The receipt issued by Lalog indicated the amount received was a
deposit.1 Unfortunately, Lalog lost the application form accomplished by Perez and so on
October 28, 1987, he asked the latter to fill up another application form.2 On November 1, 1987,
Perez was made to undergo the required medical examination, which he passed.
Pursuant to the established procedure of the company, Lalog forwarded the application for
additional insurance of Perez, together with all its supporting papers, to the office of BF Lifeman
Insurance Corporation at Gumaca, Quezon which office was supposed to forward the papers to
the Manila office.
On November 25, 1987, Perez died in an accident. He was riding in a banca which capsized
during a storm. At the time of his death, his application papers for the additional insurance of
P50,000.00 were still with the Gumaca office. Lalog testified that when he went to follow up the
papers, he found them still in the Gumaca office and so he personally brought the papers to the
Manila office of BF Lifeman Insurance Corporation. It was only on November 27, 1987 that said
papers were received in Manila.

Without knowing that Perez died on November 25, 1987, BF Lifeman Insurance Corporation
approved the application and issued the corresponding policy for the P50,000.00 on December 2,
1987.
Petitioner Virginia Perez went to Manila to claim the benefits under the insurance policies of the
deceased. She was paid P40,000.00 under the first insurance policy for P20,000.00 (double
indemnity in case of accident) but the insurance company refused to pay the claim under the
additional policy coverage of P50,000.00, the proceeds of which amount to P150,000.00 in view
of a triple indemnity rider on the insurance policy. In its letter of January 29, 1988 to Virginia A.
Perez, the insurance company maintained that the insurance for P50,000.00 had not been
perfected at the time of the death of Primitivo Perez. Consequently, the insurance comptshiy
refunded the amount of P2,075.00 which Virginia Perez had paid.
On September 21, 1990, private respondent BF Lifeman Insurance Corporation filed a complaint
against Virginia A. Perez seeking the rescission and declaration of nullity of the insurance
contract in question.
Petitioner Virginia A. Perez, on the other hand, averred that the deceased had fulfilled all his
prestations under the contract and all the elements of a valid contract are present. She then filed
a counterclaim against private respondent for the collection of P150,000.00 as actual damages,
P100,000.00 as exemplary damages, P30,000.00 as attorneys fees and P10,000.00 as expenses
for litigation.
G.R. No. 119176. March 19, 2002.
3. COMMISSIONER OF INTERNAL REVENUE, petitioner, vs. LINCOLN PHILIPPINE LIFE
INSURANCE COMPANY, INC. (now JARDINE-CMA LIFE INSURANCE COMPANY, INC.) and
THE COURT OF APPEALS, respondents.
Insurance; Any rider, clause, warranty or endorsement pasted or attached to the
policy is considered part of such policy or contract of insurance.Section 49, Title VI of
the Insurance Code defines an insurance policy as the written instrument in which a contract of
insurance is set forth. Section 50 of the same Code provides that the policy, which is required to
be in printed form, may contain any word, phrase, clause, mark, sign, symbol, signature,
number, or word necessary to complete the contract of insurance. It is thus clear that any rider,
clause, warranty or endorsement pasted or attached to the policy is considered part of such
policy or contract of insurance.
Same; Same; Same; The amount insured by the policy at the time of its issuance
necessarily includes the additional sum covered by the automatic increase clause
because it is already determinate at the time the transaction is entered into and
forms part of the policy.Here, although the automatic increase in the amount of life
insurance coverage was to take effect later on, the date of its effectivity, as well as the amount
of the increase, was already definite at the time of the issuance of the policy. Thus, the amount
insured by the policy at the time of its issuance necessarily included the additional sum covered
by the automatic increase clause because it was already determinable at the time the
transaction was entered into and formed part of the policy. The automatic increase clause in
the policy is in the nature of a conditional obligation under Article 1181, by which the increase of
the insurance coverage shall depend upon the happening of the event which constitutes the
obligation. In the instant case, the additional insurance that took effect in 1984 was an obligation
subject to a suspensive obligation, but still a part of the insurance sold to which private
respondent was liable for the payment of the documentary stamp tax.
KAPUNAN, J.:
FACTS: Private respondent Lincoln Philippine Life Insurance Co., Inc., (now Jardine-CMA Life
Insurance Company, Inc.) is a domestic corporation registered with the Securities and Exchange

Commission and engaged in life insurance business. In the years prior to 1984, private
respondent issued a special kind of life insurance policy known as the Junior Estate Builder
Policy, the distinguishing feature of which is a clause providing for an automatic increase in the
amount of life insurance coverage upon attainment of a certain age by the insured without the
need of issuing a new policy. The clause was to take effect in the year 1984. Documentary stamp
taxes due on the policy were paid by petitioner only on the initial sum assured.
In 1984, private respondent also issued 50,000 shares of stock dividends with a par value of
P100.00 per share or a total par value of P5,000,000.00. The actual value of said shares,
represented by its book value, was P19,307,500.00. Documentary stamp taxes were paid based
only on the par value of P5,000,000.00 and not on the book value.
Subsequently, petitioner issued deficiency documentary stamps tax assessment for the year
1984 in the amounts of (a) P464,898.75, corresponding to the amount of automatic increase of
the sum assured on the policy issued by respondent, and (b) P78,991.25 corresponding to the
book value in excess of the par value of the stock dividends.
Private respondent questioned the deficiency assessments and sought their cancellation in a
petition filed in the Court of Tax Appeals, docketed as CTA Case No. 4583.
On March 30, 1993, the Court of Tax Appeals found no valid basis for the deficiency tax
assessment on the stock dividends, as well as on the insurance policy.
Petitioner appealed the CTAs decision to the Court of Appeals. On November 18, 1994, the Court
of Appeals promulgated a decision affirming the CTAs decision insofar as it nullified the
deficiency assessment on the insurance policy, but reversing the same with regard to the
deficiency assessment on the stock dividends. The CTA ruled that the correct basis of the
documentary stamp tax due on the stock dividends is the actual value or book value represented
by the shares.
No. 15774. November 29, 1920.]
4. PILAR C. DE LIM, plaintiff and appellant, vs. SUN LIFE ASSURANCE COMPANY OF
CANADA, defendant and appellee.
1.INSURANCE CONTRACTS; PROVISIONAL POLICIES.A so-called "provisional policy" was
issued to the applicant reading as follows: "Received (subject to the following stipulations and
agreements) the sum of four hundred and thirty-three pesos, being the amount of the first year's
premium for a Life Assurance Policy on the life of Mr. Luis D. Lim y Garcia of Zamboanga for
P5,000 for which an application, dated the 6th day of July, 1917, has been made to the Sun Life
Assurance Company of Canada. The above mentioned life is to be assured in accordance with the
terms and conditions contained or inserted by the Company in the policy which may be granted
by it in this particular case for four months only from the date of the application, provided that
the Company shall confirm this agreement by issuing a policy on said application -when the
same shall be submitted to the Head Office in Montreal. Should the Company not issue such a
policy, then this agreement shall be null and void ab initio, and the Company shall be held not to
have been on the risk at all, but in such case the amount herein acknowledged shall be returned.
(Seal.) (Sgd.) T. B. MACAULAY, President. (Sgd.) A. F. PETERS, Agent." Held: That a contract of
insurance was not here consummated by the parties and that, consequently, the widow of the
deceased cannot recover the amount of the insurance from the insurance company.
2.ID. ; ID.A contract of insurance, like other contracts, must be assented to by both parties
either in person or by their agents, So long as an application for insurance has not been either
accepted or rejected, it is merely an offer or proposal to make a contract. The contract, to be
binding from the date of the application, must have been a completed contract, one that leaves
nothing to be done, nothing to be completed, nothing to be passed upon, or determined, before

it shall take effect. There can be no contract of insurance unless the minds of the parties have
met in agreement.
3.ID.; ID.Where an agreement is made between the applicant and the agent whether by
signing an application containing such condition, or otherwise, that no liability shall attach until
the principal approves the risk and a receipt is given by the agent, such acceptance is merely
conditional, and is subordinated to the act of the company in approving or rejecting; so in life
insurance a "binding slip" or "binding receipt" does not insure of itself.
MALCOLM, J.:
FACTS: On July 6, 1917, Luis Lim y Garcia of Zamboanga made application to the Sun Life
Assurance Company of Canada for a policy of insurance on his life in the sum of P5,000. In his
application Lim designated his wife, Pilar C. de Lim, the plaintiff herein, as the beneficiary. The
first premium of P433 was paid by Lim, and upon such payment the company issued what was
called a "provisional policy." Luis Lim y Garcia died on August 23, 1917, after the issuance of the
provisional policy but before approval of the application by the home office of the insurance
company. The instant action is brought by the beneficiary, Pilar C. de Lim, to recover from the
Sun Life Assurance Company of Canada the sum of P5,000, the amount named in the provisional
policy.
The "provisional policy" upon which this action rests reads as follows:
"Received (subject to the f ollowing stipulations and agreements) the sum of four hundred and
thirty-three pesos, being the amount of the first year's premium for a Life Assurance Policy on
the life of Mr. Luis D. Lim y Garcia of Zamboanga for P5,000, for which an application dated the
6th day of July, 1917, has been made to the Sun Life Assurance Company of Canada.
"The above-mentioned life is to be assured in accordance with the terms and conditions
contained or inserted by the Company in the policy which may be granted by it in this particular
case for four months only from the date of the application, provided that the Company shall
confirm this agreement by issuing a policy on said application when the. same shall be submitted
to the Head Office in Montreal. Should the Company not issue such a policy, then this agreement
shall be null and void ab initio, and the Company shall be held not to have been on the risk at all,
but in such case the amount herein acknowledged shall be returned.
[SEAL.] (Sgd.) "T. B. MACAULAY, President.
(Sgd.) "A. F. PETERS, Agent"
Our duty in this case is to ascertain the correct meaning of the document above quoted. A
perusal of the same many times by the writer and by other members of the court leaves a
decided impression of vagueness in the mind. Apparently it is to be a provisional policy "for four
months only from the date of this application." We use the term "apparently" advisedly, because
immediately following the words fixing the four months period comes the word "provided" which
has the meaning of "if." Otherwise stated, the policy for four months is expressely made subject
to the the affirmative condition that "the company shall confirm this agreement by issuing a
policy on said application when the same shall be submitted to the head office in Montreal." To
re-enforce the same there follows the negative condition"Should the company not issue such a
policy, then this agreement shall be null and void ab initio, and the company shall be held not to
have been on the risk." Certainly, language could hardly be used which would more clearly
stipulate that-the agreement should not go into effect until the home office of the company
should confirm it by issuing a policy. As we read and understand the so-called provisional policy it
amounts to nothing but an acknowledgment on behalf of the company, that it has received from
the person named therein the sum of money agreed upon as the first year's premium upon a
policy to be issued upon the application, if the application is accepted by the company.
It is of course a primary rule that a contract of insurance, like other contracts, must be assented
to' by both parties either in person or by their agents. So long as an application for insurance has

not been either accepted or rejected, it is merely an offer or proposal to make a contract. The
contract, to be binding from the date of the application, must have been a completed contract,
one that leaves nothing to be done, nothing to be completed, nothing to be passed upon, or
determined, before it shall take effect. There can be no contract of insurance unless the minds of
the parties have met in agreement. Our view is, that a contract of insurance was not here
consummated by the parties.
No. L-31845. April 30, 1979.
5. GREAT PACIFIC LIFE ASSURANCE COMPANY, petitioner, vs. HONORABLE COURT OF
APPEALS, respondents.
Insurance; Binding deposit receipt; Concept and Nature; When binding deposit receipt
not effective.Clearly implied from the aforesaid conditions is that the binding deposit receipt
in question is merely an acknowledgment, on behalf of the company, that the latters branch
office had received from the applicant the insurance premium and had accepted the application
subject for processing by the insurance company; and that the latter will either approve or reject
the same on the basis of whether or not the applicant is insurable on standard rates. Since
petitioner Pacific Life disapproved the insurance application of respondent Ngo Hing, the binding
deposit receipt in question had never become in force at any time. Upon this premise, the
binding deposit receipt (Exhibit E) is, manifestly, merely conditional and does not insure outright.
As held by this Court, where an agreement is made between the applicant and the agent, no
liability shall attach until the principal approves the risk and a receipt is given by the agent. The
acceptance is merely conditional, and is subordinated to the act of the company in approving or
rejecting the application. Thus, in life insurance, a binding slip or binding receipt does not
insure by itself.
Same; Same; No insurance contract between private person and insurance company
for non-acceptance of alternative insurance plan of the company and non-compliance
of conditions in binding deposit receipt; Refund of deposit proper.It bears repeating
that through the intra-company communication of April 30, 1957 (Exhibit 3-M), Pacific Life
disapproved the insurance application in question on the ground that it is not offering the twentyyear endowment insurance policy to children less than seven years of age. What it offered
instead is another plan known as the Juvenile Triple Action, which private respondent failed to
accept. In the absence of a meeting of the minds between petitioner Pacific Life and private
respondent Ngo Hing over the 20-year endowment life insurance in the amount of P50,000.00 in
favor of the latters one-year old daughter, and with the non-compliance of the abovequoted
conditions stated in the disputed binding deposit receipt, there could have been no insurance
contract duly perfected between them. Accordingly, the deposit paid by private respondent shall
have to be refunded by Pacific Life.
Same; Same; Completed Contract; Concept Of; Contract of insurance must be
completed contract to be binding.As held in De Lim vs. Sun Life Assurance Company of
Canada, supra, a contract of insurance, like otter contracts, must be asserted to by both parties
either in parson or by their agents. x x x. The contract, to be binding from the date of the
application, must have been a completed contract, one that leaves nothing to be done, nothing
to be completed, nothing to be passed upon, or determined, before it shall take effect. There can
be no contract of insurance unless the minds of the parties have met in agreement.
Same; Concealment; Nature and kind of concealment which renders ineffective
application for insurance coverage; Duties required of insurance agents.Relative to
the second issue of alleged concealment, this Court is of the firm belief that private respondent
had deliberately concealed the state of health and physical condition of his daughter Helen Go.
When private respondent supplied the required essential data for the insurance application form,
he was fully aware that his one-year old daughter is typically a mongoloid child. Such a
congenital physical defect could never be ensconced nor disguised. Nonetheless, private

respondent, in apparent bad faith, withheld the fact material to the risk to be assumed by the
insurance company. As an insurance agent of Pacific Life, he ought to know, as he surely must
have known, his duty and responsibility to supply such a material fact. Had he divulged said
significant fact in the insurance application form. Pacific Life would have verified the same and
would have had no choice but to disapprove the application outright.
Same; Same; Nature and effect of concealment on insurance contract.The contract of
insurance is one of perfect good faith (uberrima fides meaning good faith; absolute and perfect
candor or openness and honesty; the absence of any concealment or deception, however slight
[Blacks Law Dictionary, 2nd Edition], not for the insured alone but equally so for the insurer
Fieldmans Insurance Co., Inc. vs. Vda. de Songco, 25 SCRA 70). Concealment is a neglect to
communicate that which a party known and ought to communicate (Section 25, Act No. 2427).
Whether intentional or unintentional, the concealment entitles the insurer to rescind the contract
of insurance (Section 26, Id.; Yu Pang Cheng vs. Court of Appeals, et al., 105 Phil. 930; Saturnino
vs. Philippine American Life Insurance Company, 7 SCRA 316). Private respondent appears guilty
thereof.
DE CASTRO, J.:
FACTS: It appears that on March 14, 1957, private respondent Ngo Hing filed an application with
the Great Pacific Life Assurance Company (hereinafter referred to as Pacific Life) for a twentyyear endowment policy in the amount of P50,000.00 on the life of his one-year old daughter
Helen Go. Said respondent supplied the essential data which petitioner Lapulapu D. Mondragon,
Branch Manager of the Pacific Life in Cebu City wrote on the corresponding form in his own
handwriting (Exhibit I-M). Mondragon finally type-wrote the data on the application form which
was signed by private respondent Ngo Hing. The latter paid the annual premium, the sum of
P1,077.75 going over to the Company, but he retained the amount of P1,317.00 as his
commission for being a duly authorized agent of Pacific life. Upon the payment of the insurance
premium, the binding deposit receipt (Exhibit E) was issued to private respondent Ngo Hing.
Likewise, petitioner Mondragon handwrote at the bottom of the back page of the application form
his strong recommendation for the approval of the insurance application. Then on April 30, 1957,
Mondragon received a letter from Pacific Life disapproving the insurance application (Exhibit 3M). The letter stated that the said life insurance application for 20-year endowment plan is not
available for minors below seven years old, but Pacific life can consider the same under the
Juvenile Triple Action Plan, and advised that if the offer is acceptable, the Juvenile NonMedical
Declaration be sent to the Company.
The non-acceptance of the insurance plan by Pacific Life was allegedly not communicated by
petitioner Mondragon to private respondent Ngo Hing. Instead, on May 6, 1957, Mondragon
wrote back Pacific life again strongly recommending the approval of the 20-year endowment life
insurance on the ground that Pacific Life is the only insurance company not selling the 20-year
endowment insurance plan to children, pointing out that since 1954 the customers, especially
the Chinese, were asking for such coverage.
It was when things were in such state that as May 28, 1957 Helen Go died of influenza with
complication of bronchopneumonia. Thereupon, private respondent sought the payment of the
proceeds of the insurance, but having failed in his effort, he filed the action for the recovery of
the same before the Court of First Instance of Cebu, which rendered the adverse decision as
earlier referred to against both petitioners.

No. L-38613. February 25, 1982.


6. PACIFIC TIMBER EXPORT CORPORATION, petitioner, vs. THE HONORABLE COURT OF
APPEALS and WORKMENS INSURANCE COMPANY, INC., respondents.

Insurance Law; A Cover Note issued in advance of the issuance of a marine policy is
binding as an insurance contract although no separate premium was paid therefor.
The fact that no separate premium was paid on the Cover Note before the loss insured against
occurred, does not militate against the validity of petitioners contention, for no such premium
could have been paid, since by the nature of the Cover Note, it did not contain, as all Cover
Notes do not contain particulars of the shipment that would serve as basis for the computation of
the premiums. As a logical consequence, no separate premiums are intended or required to be
paid on a Cover Note. This is a fact admitted by an official of respondent company, Juan Jose
Camacho, in charge of issuing cover notes of the respondent company (p. 33, tsn, September 24,
1965).
Same; Same.At any rate, it is not disputed that petitioner paid in full all the premiums as
called for by the statement issued by private respondent after the issuance of the two regular
marine insurance policies, thereby leaving no account unpaid by petitioner due on the insurance
coverage, which must be deemed to include the Cover Note. If the Note is to be treated as a
separate policy instead of integrating it to the regular policies subsequently issued, the purpose
and function of the Cover Note would be set at naught or rendered meaningless, for it is in a real
sense a contract, not a mere application for insurance which is a mere offer.
Same; Same.The non-payment of premium on the Cover Note is, therefore, no cause for the
petitioner to lose what is due it as if there had been payment of premium, for non-payment by it
was not chargeable against its fault. Had all the logs been lost during the loading operations, but
after the issuance of the Cover Note, liability on the note would have already arisen even before
payment of premium. This is how the cover note as a binder should legally operate; otherwise,
it would serve no practical purpose in the realm of commerce, and is supported by the doctrine
that where a policy is delivered without requiring payment of the premium, the presumption is
that a credit was intended and policy is valid.
Same; Delay of insured in reporting the loss must be objected to promptly by insurer.
Sending of insurance adjuster to assess the loss amounts to waiver of delay in giving
notice of loss.The defense of delay as raised by private respondent in resisting the claim
cannot be sustained. The law requires this ground of delay to be promptly and specifically
asserted when a claim on the insurance agreement is made. The undisputed facts show that
instead of invoking the ground of delay in objecting to petitioners claim of recovery on the cover
note, it took steps clearly indicative that this particular ground for objection to the claim was
never in its mind. The nature of this specific ground for resisting a claim places the insurer on
duty to inquire when the loss took place, so that it could determine whether delay would be a
valid ground upon which to object to a claim against it.
Same; Same.In the proceedings that took place later in the Office of the Insurance
Commissioner, private respondent should then have raised this ground of delay to avoid liability.
It did not do so. It must be because it did not find any delay, as this Court fails to find a real and
substantial sign thereof. But even on the assumption that there was delay, this Court is satisfied
and convinced that as expressly provided by law, waiver can successfully be raised against
private respondent.
DE CASTRO, J.:
FACTS: On March 19, 1963, the plaintiff secured temporary insurance from the defendant for its
exportation of 1,250,000 board feet of Philippine Lauan and Apitong logs to be shipped from the
Diapitan Bay, Quezon Province to Okinawa and Tokyo, Japan. The defendant issued on said date
Cover Note No. 1010, insuring the said cargo of the plaintiff Subject to the Terms and Conditions
of the WORKMENS INSURANCE COMPANY, INC. printed Marine Policy form as filed with and
approved by the Office of the Insurance Commissioner.
The regular marine cargo policies were issued by the defendant in favor of the plaintiff on April
2, 1963. The two marine policies bore the numbers 53 HO 1032 and 53 HO 1033 (Exhibits B and

C, respectively). Policy No. 53 HO 1032 (Exhibit B) was for 542 pieces of logs equivalent to
499,950 board feet. Policy No. 53 HO 1033 was for 853 pieces of logs equivalent to 695,548
board feet (Exhibit C). The total cargo insured under the two marine policies accordingly
consisted of 1,395 logs, or the equivalent of 1,195,498 bd. ft.
After the issuance of Cover Note No. 1010 (Exhibit A), but before the issuance of the two marine
policies Nos. 53 HO 1032 and 53 HO 1033, some of the logs intended to be exported were lost
during loading operations in the Diapitan Bay. The logs were to be loaded on the SS Woodlock
which docked about 500 meters from the shoreline of the Diapitan Bay. The logs were taken from
the log pond of the plaintiff and from which they were towed in rafts to the vessel.
At about 10:00 oclock a. m. on March 29, 1963, while the logs were alongside the vessel, bad
weather developed resulting in 75 pieces of logs which were rafted together to break loose from
each other. 45 pieces of logs were salvaged, but 30 pieces were verified to have been lost or
washed away as a result of the accident.
In a letter dated April 4, 1963, the plaintiff informed the defendant about the loss of
approximately 32 pieces of logs during loading of the SS Woodlock.
Although dated April 4, 1963, the letter was received in the office of the defendant only on April
15, 1963, as shown by the stamp impression appearing on the left bottom corner of said letter.
The plaintiff subsequently submitted a Claim Statement demanding payment of the loss under
Policies Nos. 53 HO 1032 and 53 HO 1033, in the total amount of P19,286.79. On July 17, 1963,
the defendant requested the First Philippine Adjustment Corporation to inspect the loss and
assess the damage. The adjustment company submitted its Report on August 23, 1963 (Exhibit
H). in said report, the adjuster found that the loss of 30 pieces of logs is not covered by Policies
Nos. 53 HO 1032 and 1033 inasmuch as said policies covered the actual number of logs loaded
on board the SS Woodlock. However, the loss of 30 pieces of logs is within the 1,250,000 bd. ft.
covered by Cover Note No. 1010 insured for $70,000.00.
On September 14, 1963, the adjustment company submitted a computation of the defendants
probable liability on the loss sustained by the shipment, in the total amount of P11,042.04.
On January 13, 1964, the defendant wrote the plaintiff denying the latters claim, on the ground
that defendants investigation revealed that the entire shipment of logs covered by the two
marines policies No. 53 HO 1032 and 53 HO 1033 were received in good order at their point of
destination. It was further stated that the said loss may not be considered as covered under
Cover Note No. 1010 because the said Note had become null and void by virtue of the issuance
of Marine Policy Nos. 53 HO 1032 and 1033 (Exhibit J-1). The denial of the claim by the
defendant was brought by the plaintiff to the attention of the Insurance Commissioner by means
of a letter dated March 21, 1964 (Exhibit K). In a reply letter dated March 30, 1964, Insurance
Commissioner Francisco Y. Mandanas observed that it is only fair and equitable to indemnify the
insured under Cover Note No. 1010, and advised early settlement of the said marine loss and
salvage claim.
On June 26, 1964, the defendant informed the Insurance Commissioner that, on advice of their
attorneys, the claim of the plaintiff is being denied on the ground that the cover note is null and
void for lack of valuable consideration.
G.R. No. 109937. March 21, 1994.
7. DEVELOPMENT BANK OF THE PHILIPPINES, petitioner, vs. COURT OF APPEALS and
the ESTATE OF THE LATE JUAN B. DANS, represented by CANDIDA G. DANS, and the
DBP MORTGAGE REDEMPTION INSURANCE POOL, respondents.

Civil Law; Contracts; Insurance; Where there was no perfected contract of insurance,
DBP MRI Pool cannot be held liable on the contract that does not exist.Undisputably,
the power to approve MRI applications is lodged with the DBP MRI Pool. The pool, however, did
not approve the application of Dans. There is also no showing that it accepted the sum of
P1,476.00, which DBP credited to its account with full knowledge that it was payment for Danss
premium. There was, as a result, no perfected contract of insurance; hence, the DBP MRI Pool
cannot be held liable on a contract that does not exist.
Same; Agency; Obligation of the Agent; Agent acting as such is not personally liable
unless he expressly binds himself or exceeds his authority.Under Article 1897 of the
Civil Code of the Philippines, the agent who acts as such is not personally liable to the party
with whom he contracts, unless he expressly binds himself or exceeds the limits of his authority
without giving such party sufficient notice of his powers.
Same; Same; Same; Liability of the agent who exceeds the scope of his authority
depends upon whether the 3rd person is aware of the limits of agents powers.The
liability of an agent who exceeds the scope of his authority depends upon whether the third
person is aware of the limits of the agents powers. There is no showing that Dans knew of the
limitation on DBPs authority to solicit applications for MRI.
Same; Same; Same; If the third person dealing with an agent is unaware of the limits
of the authority conferred by the principal on the agent and the third person has been
deceived by the non-disclosure by the agent, then the latter is liable for damages to
him.If the third person dealing with an agent is unaware of the limits of the authority conferred
by the principal on the agent and he (third person) has been deceived by the non-disclosure
thereof by the agent, then the latter is liable for damages to him (V Tolentino, Commentaries and
Jurisprudence on the Civil Code of the Philippines, p. 422 [1992], citing Sentencia [Cuba] of
September 25, 1907). The rule that the agent is liable when he acts without authority is founded
upon the supposition that there has been some wrong or omission on his part either in
misrepresenting, or in affirming, or concealing the authority under which he assumes to act
(Francisco, V., Agency 307 [1952], citing Hall v. Lauderdale, 46 N.Y. 70, 75). Inasmuch as the nondisclosure of the limits of the agency carries with it the implication that a deception was
perpetrated on the unsuspecting client, the provisions of Articles 19, 20 and 21 of the Civil Code
of the Philippines come into play.
QUIASON, J.:
FACTS: This is a petition for review on certiorari under Rule 45 of the Revised Rules of Court to
reverse and set aside the decision of the Court of Appeals in CA-G.R. CV No. 26434 and its
resolution denying reconsideration thereof.
We affirm the decision of the Court of Appeals with modification.
In May 1987, Juan B. Dans, together with his wife Candida, his son and daughter-in-law, applied
for a loan of P500,000.00 with the Development Bank of the Philippines (DBP), Basilan Branch. As
the principal mortgagor, Dans, then 76 years of age, was advised by DBP to obtain a mortgage
redemption insurance (MRI) with the DBP Mortgage Redemption Insurance Pool (DBP MRI Pool).
A loan, in the reduced amount of P300,000.00, was approved by DBP on August 4, 1987 and
released on August 11, 1987. From the proceeds of the loan, DBP deducted the amount of
P1,476.00 as payment for the MRI premium. On August 15, 1987, Dans accomplished and
submitted the MRI Application for Insurance and the Health Statement for DBP MRI Pool.
On August 20, 1987, the MRI premium of Dans, less the DBP service fee of 10 percent, was
credited by DBP to the savings account of the DBP MRI Pool. Accordingly, the DBP MRI Pool was
advised of the credit.

On September 3, 1987, Dans died of cardiac arrest. The DBP, upon notice, relayed this
information to the DBP MRI Pool. On September 23, 1987, the DBP MRI Pool notified DBP that
Dans was not eligible for MRI coverage, being over the acceptance age limit of 60 years at the
time of application.
On October 21, 1987, DBP apprised Candida Dans of the disapproval of her late husbands MRI
application. The DBP offered to refund the premium of P1,476.00 which the deceased had paid,
but Candida Dans refused to accept the same, demanding payment of the face value of the MRI
or an amount equivalent to the loan. She, likewise, refused to accept an ex gratia settlement of
P30,000.00, which the DBP later offered.
On February 10, 1989, respondent Estate, through Candida Dans as administratrix, filed a
complaint with the Regional Trial Court, Branch I, Basilan, against DBP and the insurance pool for
Collection of Sum of Money with Damages. Respondent Estate alleged that Dans became
insured by the DBP MRI Pool when DBP, with full knowledge of Dans age at the time of
application, required him to apply for MRI, and later collected the insurance premium thereon.
Respondent Estate therefore prayed: (1) that the sum P139,500.00, which it paid under protest
for the loan, be reimbursed; (2) that the mortgage debt of the deceased be declared fully paid;
and (3) that damages be awarded.
No. L-20853. May 29, 1967.
8. BONIFACIO BROS., INC., ET AL., plaintiffs-appellants, vs. ENRIQUE MORA, ET AL.,
defendants-appellees.
Contracts; Contracts take effect only between the parties thereto; Exception.
Contracts take effect only between the parties thereto, except in some specific instances
provided by law where the contract contains some stipulation in favor of a third person which is
known as a stipulation pour autrui or a provision in favor of a third person not a party to the
contract. Under this doctrine, a third person is allowed to avail himself of a benef it granted to
him by the terms of the contract, provided that the contracting parties have clearly and
deliberately conferred a favor upon such person. Consequently, a third person, not a party to the
contract, has no action against the parties thereto, and cannot generally demand the
enforcement of the same.
Same; Stipulation pour autrui; When a third person has an enforceable interest in the
contract.The question of whether a third person has an enforceable interest in a contract
must be settled by determining whether the contracting parties intended to tender him such an
interest by deliberately inserting terms in their agreement with the avowed purpose of conferring
a favor upon such third person. The fairest test to determine whether the interest of a third
person in a contract is a stipulation pour autrui or merely an incidental interest, is to rely upon
the intention of the parties as disclosed by their contract.
Same; Insurance; Nature of insurance policy.A policy of insurance is a distinct and
independent contract between the insured and insurer. A third person has no right in law or
equity to the proceeds of an insurance unless there is a contract or trust, expressed or implied,
between the insured and third person.
Same; Interpretation of clause in insurance contract regarding repair of damaged vehicle.The
clause in an insurance policy, authorizing the owner of the damaged vehicle to contract for its
repair does not mean that the repairman is entitled to collect the cost of repair out of the
proceeds of the insurance. It merely establishes the procedure that the insured has to follow in
order to be entitled to indemnity for repair.
Same; Meaning of loss in insurance.The word "loss" in insurance law embraces injury or
damage. A loss may be total or partial.

Same; When mortgagee of damaged car, as beneficiary, is preferred to the repairman


with respect to insurance proceeds.Where the mortgagee is the beneficiary in a car
insurance, it has a better right than the repairman to the insurance proceeds.
CASTRO, J.:
FACTS: Enrique Mora, owner of an Oldsmobile sedan model 1956, bearing plate No. QC-8088,
mortgaged the same to the H.S. Reyes, Inc., with the condition that the former would insure the
automobile with the latter as beneficiary. The automobile was thereafter insured on June 23,
1959 with the State Bonding & Insurance Co,, Inc., and motor car insurance policy A-0615 was
issued to Enrique Mora, the pertinent provisions of which read:
"1. The Company (referring to the State Bonding & Insurance Co., Inc.) will, subject to the Limits
of Liability, indemnify the Insured against loss of or damages to the Motor Vehicle and its
accessories and spare parts whilst thereon; (a) by accidental collision or overturning or collision
or overturning consequent upon mechanical breakdown or consequent upon wear and tear,
xxx
xxx
xxx
2. At its own option the Company may pay in cash the amount of the loss or damage or may
repair, reinstate, or replace the Motor Vehicle or any part thereof or its accessories or spare
parts. The liability of the Company shall not exceed the value of the parts whichever is the less.
The Insured's estimate of value stated in the schedule will be the maximum amount payable by
the Company in respect of any claim for loss or damage.
xxx
xxx
xxx
4. The Insured may authorize the repair of the Motor Vehicle necessitated by damage for which
the Company may be liable under this Policy provided that:(a) The estimated cost of such
repair does not exceed the Authorized Repair Limit, (b) A detailed estimate of the cost is
forwarded to the Company without delay, subject to the condition that 'Loss, if any, is payable to
H.S. Reyes, Inc..', by virtue of the fact that said Oldsmobile sedan was mortgaged in favor of the
said H.S. Reyes, Inc. and that under a clause in said insurance policy, any loss was made payable
to the H.S, Reyes, Inc. as Mortgagee;
xxx
xxx
xxx
During the effectivity of the insurance contract, the car met with an accident. The insurance
company then assigned the accident to the H.H. Bayne Adjustment Co. for investigation and
appraisal of the damage. Enrique Mora, without the knowledge and consent of the H.S. Reyes,
Inc., authorized the Bonifacio Bros. Inc. to furnish the labor and materials, some of which were
supplied by the Ayala Auto Parts Co. For the cost of labor and materials, Enrique Mora was billed
at P2,102.73 through the H.H. Bayne Adjustment Co. The insurance company, after claiming a
franchise in the amount of ?100, drew a check in the amount of P2,002.73, as proceeds of the
insurance policy, payable to the order of Enrique Mora or H.S. Reyes, Inc., and entrusted the
check to the H.H. Bayne Adjustment Co. for disposition and delivery to the proper party. In the
meantime, the car was delivered to Enrique Mora without the consent of the H.S, Reyes, Inc., and
without payment to the Bonifacio Bros. Inc. and the Ayala Auto Parts Co. of the cost of repairs
and materials,
Upon the theory that the insurance proceeds should be paid directly to them, the Bonifacio Bros.
Inc. and the Ayala Auto Parts Co. filed on May 8, 1961 a complaint with the Municipal Court of
Manila against Enrique Mora and the State Bonding & Insurance Co., Inc. for the collection of the
sum of P2,002.73.
No. L-23276. November 29, 1968.
9. MELECIO COQUIA, MARIA ESPANUEVA and MANILA YELLOW TAXICAB Co., INC.,
plaintiffs-appellees, vs. FIELDMEN'S INSURANCE Co., INC., defendant-appellant.

Pleading and practice; Parties; Cause of action; Only parties to a contract may bring
an action thereon; Exception; Contracts pour autrui.Although, in general, only parties to
a contract may bring an action based thereon, this rule is subject to exceptions, one of which is
.found in the second paragraph of Article 1311 of the Civil Code of the Philippines, reading: "a
contract should contain some stipulation in favor of a third person, he may demand its fulfillment
provided he communicated his acceptance to the obligor before its revocation. A mere incidental
benefit or interest of a person is not sufficient. The contracting parties must have clearly and
deliberately conferred a favor upon a third person."
This is but the restatement of a well-known principle concerning contracts pour autrui, the
enforcement of which may be demanded by a third party for whose benefit it was made,
although not a party to the contract, before the stipulation in his favor has been revoked by the
contracting parties.
Insurance law; Where an insurance policy is typical of a contract pour autrui; Case at
bar.In the case at bar, the insurance policy contains stipulations pursuant to which the
insurance company "will indemnify any authorized Driver who is driving the Motor Vehicle" of the
Insured and, in the event of death of said driver, the Company shall, likewise, "indemnify his
personal representatives," and the Company "may, at its option, make indemnity payable
directly to the claimants or heirs of claimants x x x it being the true intention of this Policy to
protect x x x the liabilities of the Insured towards the passengers of the Motor Vehicle and the
Public" in other words, third parties.
Thus, the policy under consideration is typical of contracts pour autrui, this character being made
more manifest by the fact that the deceased driver paid fifty percent (50%) of the corresponding
premiums, which were deducted from his weekly commissions. Under these conditions, it is clear
that the Coquiaswho, admittedly, are the sole heirs of the deceasedhave a direct cause of
action against the Company (Uy Tam v. Leonard, 30 Phil. 471, 485-486; Kauffman v. Philippine
National Bank, 42 Phil. 182, 187, 189), and, since they could have maintained this action by
themselves, without the assistance of the insured it goes without saying that they could and did
properly join the latter in filing the complaint herein (Guingon v. Capital Insurance & Surety Co.,
Inc., L-22042, Aug. 17, 1967).
Same; Where a condition in the policy concerning reference of dispute to an
arbitrator, as a condition precedent to a right of action or suit upon a policy, was
deemed waived; Case at bar.Section 17 of the policy under consideration reads: "If any
difference or dispute shall arise with respect to the amount of the Company's liability under this
Policy, the same shall be referred to the decision of a single arbitrator to be agreed upon by both
parties or failing such agreement of a single arbitrator, to the decision of two arbitrators, one to
be appointed in writing by each of the parties within one calendar month after having been
required in writing so to do by either of the parties and in case of disagreement between the
arbitrators, to the decision of an umpire who shall have been appointed in writing by the
arbitrators. before entering on the reference and the costs of and incidental ,to the reference
shall be dealt with in the Award. And it is hereby expressly stipulated and declared that it shall be
a condition precedent to any right of action or suit upon this Policy that the award by such
arbitrator, arbitrators or umpire of the amount of the Company's liability hereunder if disputed
shall be first obtained."
The record shows, however, that none of the parties to the contract invoked this section, or
made any reference to arbitration, during the negotiations preceding the institution of the
present case. In fact, counsel for both parties stipulated, in the trial court, that none of them had,
at any time during said negotiations, even suggested the settlement of the issue between them
by arbitration, as provided in said section. Their aforementioned acts or omissions had the effect
of a waiver of their respective right to demand an arbitration (Kahnweiler v. Phenix Insurance Co.
of Brooklyn, 67 Fed. 483; Independent School District No. 35, St. Louis County v. A. Hedenberg &
Co., Inc., 7 NW 2nd, 511).

CONCEPCION, C.J.:
FACTS: This is an appeal from a decision of the Court of First Instance of Manila, certified to us
by the Court of Appeals, only questions of law being involved therein. Indeed, the pertinent facts
have been stipulated and/or, admitted by the parties at the hearing of the case in the trial court,
to dispense with the presentation of evidence therein.
It appears that on December 1, 1961, appellant Fieldmen's Insurance Company, Inc.hereinafter
referred to as the Companyissued, in favor of the Manila Yellow Taxicab Co., Inc.hereinafter
referred to as the Insureda common carrier accident insurance policy, covering the period from
December 1, 1961 to December 1, 1962. It was stipulated in said policy that:
"The Company will, subject to the Limits of Liability and under the Terms. of this Policy, indemnify
the Insured in the event of accident caused by or arising out of the use of Motor Vehicle against
all sums which the Insured will become legally liable to pay in respect of: Death or bodily injury
to any farepaying passenger including the Driver. Conductor and/or Inspector who is riding in the
Motor Vehicle insured at the time of accident or injury."
While the policy was in force, or on February 10, 1962, a taxicab of the Insured, driven by Carlito
Coquia, met a vehicular accident at Magaldan, Pangasinan, in consequence of which Carlito
died. The Insured filed therefor a claim for P5,000.00 to which the Company replied with an offer
to pay P2,000.00, by way of compromise. The Insured rejected the same and made a counteroffer for P4,000.00, but the Company did not accept it. Hence, on September 18, 1962, the
Insured and Carlito's parents, namely, Melecio Coquia and Maria Espanuevahereinafter referred
to as the Coquiasfiled a complaint against the Company to collect the proceeds of the
aforementioned policy. In its answer, the Company admitted the existence thereof, but pleaded
lack of cause of action on the part of the plaintiffs.
No. L-22042. August 17, 1967.
10. DIONISIA, EULOGIO, MARINA, GUILLERMO and NORBERTO all surnamed GUINGON,
plaintiffs-appellees, vs. ILUMINADO DEL MONTE, JULIO AGUILAR and CAPITAL
INSURANCE & SURETY Co., INC., defendants. CAPITAL INSURANCE & SURETY Co., INC.,
defendant-appellant.
Insurance; Right of injured person to sue insurer of party at fault; Condition.The right
of the person injured to sue the insurer of the party at fault (insured) depends on whether the
contract of insurance is intended to benefit third persons also or only the insured. The test
applied is this: Where the contract provides for indemnity against liability to third persons, then
third persons to whom the insured is liable. can sue the insurer. Where the contract is for
indemnity against actual loss or payment, then third persons cannot proceed against the insurer,
the contract being solely to reimburse the insured for liability actually discharged by him through
payment to third persons, said third persons' recourse being thus limited to the insured alone.
Same; Pleading and Practice; Joinder of parties; When "no action" clause in insurance
policy cannot prevail over procedural rule as to joinder of parties.Where the insurer
agreed to indemnify the insured "against all sums x x x which the Insured shall become legally
liable to pay in respect of: a death of or bodily injury to any person", the insurance is one for
indemnity against liability, From the f act then that the insured is liable to the third-person, such
third-person is entitled to sue the insurer. The "no action" clause in the policy of insurance cannot
prevail over the Rules of Court provision aimed at avoiding multiplicity of suits.
BENGZON, J.P., J.:
FACTS: Julio Aguilar owned and operated several jeepneys in the City of Manila among which
was one with plate number PUJ-206-Manila, 1961. He entered into a contract with the Capital
Insurance & Surety Co., Inc. insuring the operation of his jeepneys against accidents with thirdparty liability. As a consequence thereof an insurance policy was executed by the Capital

Insurance & Surety Co., Inc., the pertinent provisions of which in so far as this case is concerned
contains the following:
"Section IILIABILITY TO THE PUBLIC
"1. The Company, will, subject to the limits of liability, indemnify the Insured in the event of
accident caused by or arising out of the use of the Motor Vehicle/s or in connection with the
loading or unloading of the Motor Vehicle/s, against all sums including claimant's costs and
expenses which the Insured shall become legally liable to pay in respect of:
"a. death of or bodily injury to any person
"b. damage to property"
During the effectivity of such insurance policy on February 20, 1961 Iluminado del Monte, one of
the drivers of the jeepneys operated by Aguilar, while driving along the intersection of Juan Luna
and Moro streets, City of Manila, bumped with the jeepney abovementioned one Gervacio
Guingon who had just alighted from another jeepney and as a consequence the latter died some
days thereafter.
A corresponding information for homicide thru reckless imprudence was filed against Iluminado
del Monte, who pleaded guilty. A penalty of four months imprisonment was imposed on him.
As a corollary to such action, the heirs of Gervacio Guingon filed an action for damages praying
that the sum of P82,771.80 be paid to them jointly and severally by the defendants, driver
Iluminado del Monte, owner and operator Julio Aguilar, and the Capital Insurance & Surety Co.,
Inc. For failure to answer the complaint, Del Monte and Aguilar were declared in default. Capital
Insurance & Surety Co., Inc. answered, alleging that the plaintiff has no cause of action against it.
[No. 9374. February 16, 1915.]
11. FRANCISCO DEL VAL ET AL., plaintiffs and appellants, vs. ANDRES DEL, VAL,
defendant and appellee
7.LIFE INSURANCE; PROCEEDS; HEIR AS BENEFICIARY.Where a life-insurance policy is
made payable to one of the heirs of the person whose life is insured, the proceeds of the policy
on the death of the insured belong exclusively to the beneficiary and not to the estate of the
person whose life was insured; and such proceeds are his individual property and not the
property of the heirs of the person whose life was insured.
8.ID.; ID.; ID.; ARTICLE 1035, CIVIL CODE.Article 1035 of the Civil Code, providing that "an
heir by force of law surviving with others of the same character to a succession must bring into
the hereditary estate the property or securities he may have received from the deceased during
the life of the same, by way of dowry, gift, or for any good consideration, in order to compute it
in fixing the legal portions and in the account of the division," is not applicable to the proceeds of
an insurance policy made payable to one of the heirs of the insured by name, nor can the
proceeds of such a policy be considered a gift under article 819 of the Civil Code.
9.ID.; ID.; ID.; CODE OF COMMERCE.The contract of life insurance is a special contract and
the destination of the proceeds thereof is determined by special laws which deal exclusively with
that subject. The Civil Code has no provisions which relate directly and specifically to lifeinsurance contracts or to the destination of life-insurance proceeds. That subject is regulated
exclusively by the Code of Commerce, which provides for the terms of the contract, the relations
of the parties and the destination of the proceeds of the policy.
MORELAND, J.:
FACTS: The pleadings set forth that the plaintiffs and defendant are brothers and sisters; that
they are the only heirs at law and next of kin of Gregorio Nacianceno del Val, who died in Manila
on August 4, 1910, intestate; that an administrator was appointed for the estate of the deceased,

and, after a partial administration, it was closed and the administrator discharged by order of the
Court of First Instance dated December 9, 1911; that during the lifetime of the deceased he took
out insurance on his life for the sum of P40,000 and made it payable to the defendant as sole
beneficiary; that after his death the defendant collected the face of the policy; that of said policy
he paid the sum of P18,36.5.20 to redeem certain real estate which the decedent had sold to
third persons with a right to repurchase; that the redemption of said premises was made by the
attorney of the defendant in the name of the plaintiffs and the defendant as heirs of the
deceased vendor; that the redemption in the name of the plaintiffs was, so defendant declares,
without his knowledge or consent; that since the redemption of said premises they have been in
the possession of the plaintiffs, who have had the use and benefit thereof; that during that time
the plaintiffs paid no taxes and made no repairs.
It further appears from the pleadings that the defendant, on the death of the deceased, took
possession of most of his personal property, which he still has in his possession, and that he has
also the balance on said insurance policy amounting to P21,634.80.
Plaintiffs contend that the amount of the insurance policy belonged to the estate of the deceased
and not to the defendant personally; that, therefore, they are entitled to a partition not only of
the real and personal property, but also of the P40,000 life insurance. The complaint prays a
partition of all the property, both real and personal, left by the deceased; that the defendant
account for P21,634.80, and that that sum be divided equally among the plaintiffs and defendant
along with the other property of deceased.
The defendant denies the material allegations of the complaint and sets up as special defense
and counterclaim that the redemption of the real estate sold by his father was made in the name
of the plaintiffs and himself instead of in his name alone without his knowledge or consent; and
that it was not his intention to use the proceeds of the insurance policy for the benefit of any
person but himself, he alleging that he was and is the sole owner thereof and that it is his
individual property. He, therefore, asks that he be declared the owner of the real estate
redeemed by the payment of the P18,365.20, the owner .of the remaining P21,634.80, the
balance of the insurance policy, and that the plaintiffs account for the use and occupation of the
premises so redeemed since the date of the redemption.
No. L-44059. October 28, 1977
12. THE INSULAR LIFE ASSURANCE COMPANY, LTD., plaintiff-appellee, vs. CARPONIA T.
EBRADO and PASCUALA VDA. DE EBRADO, defendants-appellants.
Commercial Law; Insurance; Insurance Code; Word Interest in Sec. 50 of Insurance
Act which provides that insurance shall be applied exclusively to the proper interest
of the person in whose name it is made refers only to the insured and not to the
beneficiary; contract of insurance personal in character.Section 50 of the Insurance Act
which provides that (t)he insurance shall be applied exclusively to the proper interest of the
person in whose name it is made cannot be validly seized upon to hold that the same includes
the beneficiary. The word interest highly suggests that the provision refers only to the
insured and not the beneficiary, since a contract of insurance is personal in character.
Otherwise, the prohibitory laws against illicit relationships especially on property and descent will
be rendered nugatory, as the same could easily be circumvented by modes of insurance.
Same; Same; On matters not otherwise specifically provided for by the Insurance Law,
the contract of life insurance is governed by general rules of civil law.Rather the
general rules of civil law should be applied to resolve this void in the Insurance Law. Article 2011
of the New Civil Code states: The contract of insurance is governed by special laws. Matters not
expressly provided for in such special laws shall be regulated by this Code. When not otherwise
specifically provided for by the Insurance Law, the contract of life insurance is governed by the
general rules of the civil law regulating contracts. And under Article 2012 of the same Code, any

person who is forbidden from receiving any donation under Article 739 cannot be named
beneficiary of a life insurance policy by the person who cannot make a donation to him.
Common-law spouses are, definitely, barred from receiving donations from each other.
Same; Same; Life Insurance policy no different from civil donation as far as
beneficiary is concerned; Both are founded on liberality; Common-law spouses
designated as beneficiary barred from receiving life insurance proceeds from a legally
married person; Reasons therefor.In essence, a life insurance policy is no different from a
civil donation insofar as the beneficiary is concerned. Both are founded upon the same
consideration: liberality. A beneficiary is like a donee, because from the premiums of the policy
which the insured pays out of liberality, the beneficiary will receive the proceeds or profits of said
insurance. As a consequence, the proscription in Article 739 of the new Civil Code should equally
operate in life insurance contracts. The mandate of Article 2012 cannot be laid aside: any person
who cannot receive a donation cannot be named as beneficiary in the life insurance policy of the
person who cannot make the donation. Under American law, a policy of life insurance is
considered as a testament and in construing it, the courts will, so far as possible treat it as a will
and determine the effect of a clause designating the beneficiary by rules under which wills are
interpreted.
Same; Same; Conviction for adultery or concubinage for those barred from receiving
donations or life insurance not required as only preponderance of evidence is
necessary.We do not think that a conviction for adultery or concubinage is exacted before the
disabilities mentioned in Article 739 may effectuate. More specifically, with regard to the
disability on persons who were guilty of adultery or concubinage at the time of the donation, x
x x The underscored clause neatly conveys that no criminal conviction for the disqualifying
offense is a condition precedent. In fact, it cannot even be gleaned from the aforequoted
provision that a criminal prosecution is needed. On the contrary, the law plainly states that the
guilt of the party may be proved in the same action for declaration of nullity of donation. And,
it would be sufficient if evidence preponderates upon the guilt of the consort for the offense
indicated. The quantum of proof in criminal cases is not demanded.
MARTIN, J.:
FACTS: This is a novel question in insurance law: Can a common-law wife named as beneficiary
in the life insurance policy of a legally married man claim the proceeds thereof in case of death
of the latter?
On September 1, 1968, Buenaventura Cristor Ebrado was issued by The Insular Life Assurance
Co., Ltd., Policy No. 009929 on a whole-life plan for P5,882.00 with a rider for Accidental Death
Benefits for the same amount. Buenaventura C. Ebrado designated Carponia T. Ebrado as the
revocable beneficiary in his policy. He referred to her as his wife.
On October 21, 1969, Buenaventura C. Ebrado died as a result of an accident when he was hit by
a falling branch of a tree. As the insurance policy was in force, The Insular Life Assurance Co.,
Ltd. stands liable to pay the coverage in the total amount of P11,745.73, representing the face
value of the policy in the amount of P5,882.00 plus the additional benefits for accidental death
also in the amount of P5,882.00 and the refund of P18.00 paid for the premium due November,
1969, minus the unpaid premiums and interest thereon due for January and February, 1969, in
the sum of P36.27.
Carponia T. Ebrado filed with the insurer a claim for the proceeds of the policy as the designated
beneficiary therein, although she admits that she and the insured Buenaventura C. Ebrado were
merely living as husband and wife without the benefit of marriage.
Pascuala Vda. de Ebrado also filed her claim as the widow of the deceased insured. She asserts
that she is the one entitled to the insurance proceeds, not the common-law wife, Carponia T.
Ebrado.

In doubt as to whom the insurance proceeds shall be paid, the insurer, The Insular Life Assurance
Co., Ltd. commenced an action for Interpleader before the Court of First Instance of Rizal on April
29, 1970.
No. L-15862. July 31, 1961.
13. PAULO ANG and SALLY C. ANG, plaintiffs-appellees, vs. FULTON FIRE INSURANCE
CO., ET AL., defendants. FULTON FIRE INSURANCE CO., defendant-appellant.
Insurance; Fire Insurance Policies; Condition limiting period for filing claim after
rejection not merely procedural but a condition precedent.The condition contained in
an insurance policy that claims must be presented within one year after rejection is not merely a
procedural requirement but an important matter essential to a prompt settlement of claims
against insurance companies as it demands that insurance suits be brought by the insured while
the evidence as to the origin and cause of destruction have not yet disappeared. It is in the
nature of a condition precedent to the liability of the insurer, or in other terms, a resolution
clause, the purpose of which is to terminate all liabilities in case the action is not filed by the
insured within the period stipulated.
Same; Contractual limitations in policies prevail over statutory limitations.
Contractual limitations in insurance policies prevail over the statutory limitations, as well as over
the exceptions to the latter, because the rights of the parties flow from the contract of insurance.
Their contract is the law between the parties, and their agreement that an action on a claim
denied by the insurer must be brought within one year from the denial, governs, not the rules on
the prescription of actions.
Same; When filing of action by insured against agent in effective.If there is no
condition in the policy that an action should be filed by the insured against the agent for his
claim, the filing of such action has no legal effect and serves no other purpose except that of
notifying the agent of the claim. There is no law giving any effect to such action upon the
principal, and courts cannot by interpretation extend the clear scope of the agreement beyond
what is agreed upon by the parties.
LABRADOR, J.:
FACTS: The present action was instituted by the spouses Paulo Ang and Sally C. Ang against the
Fulton Fire Insurance Company and the Paramount Surety and Insurance Company, Inc. to
recover from them the face value of a fire insurance policy issued in plaintiffs favor covering a
store owned and operated by them in Laoag, Ilocos Norte. From a judgment of the court ordering
the defendant Fulton Fire Insurance Co. to pay the plaintiffs the sum of P10,000.00, with interest,
and an additional sum of P2,000.00 as attorneys fees, and costs, the defendants have appealed
directly to this Court.
On September 9, 1953, defendant Fulton Fire Insurance Company issued a policy No. F-4730340,
in favor of P. & S. Department Store (Sally C. Ang) over stocks of general merchandise, consisting
principally of dry goods, contained in a building occupied by the plaintiffs at Laoag, Ilocos Norte.
The premium is P500.00 annually. The insurance was issued for one year, but the same was
renewed for another year on September 31, 1954. On December 27, 1954, the store containing
the goods insured was destroyed by fire. On December 30, following, plaintiffs executed the first
claim form. The claim together with all the necessary papers relating thereto, were forwarded to
the Manila Adjustment Company, the defendants adjusters, and received by the latter on June 8,
1955. On January 12, 1955, the Manila Adjustment Company accepted receipt of the claim and
requested the submission of the books of accounts of the insured for the year 1953-1954 and a
clearance from the Philippine Constabulary and the police. On April 6, 1956, the Fulton Fire
Insurance Company wrote the plaintiffs that their claim was denied. This denial of the claim was
received by the plaintiffs on April 19, 1956.

On January 13, 1955, plaintiff Paulo Ang and ten others were charged for arson in Criminal Case
No. 1429 in the Justice of the Peace Court of Laoag, Ilocos Norte. The case was remanded for trial
to the Court of First Instance of Ilocos Norte and there docketed as Criminal Case No. 2017. The
said court in a decision dated December 9, 1957, acquitted plaintiff Paulo Ang of the crime of
arson.
The present action was instituted on May 5, 1958. The action was originally instituted against
both the Fulton Fire Insurance Company and the Paramount Surety and Insurance Company, Inc.,
but on June 16, 1958, upon motion of the Paramount Surety, the latter was dropped from the
complaint.
On May 26, 1958, the defendant Fulton Fire Insurance Company filed an answer to the complaint,
admitting the existence of the contract of insurance, its renewal and the loss by fire of the
department store and the merchandise contained therein, but denying that the loss by the fire
was accidental, alleging that it was occasioned by the wilful act of the plaintiff Paulo Ang himself.
It claims that under paragraph 13 of the policy, if the loss or damage is occasioned by the wilful
act of the insured, or if the claim is made and rejected but no action is commenced within 12
months after such rejection, all benefits under the policy would be forfeited, and that since the
claim of the plaintiffs was denied and plaintiffs received notice of denial on April 18, 1956, and
they brought the action only on May 5, 1958, all the benefits under the policy have been
forfeited.
On February 12, 1959, plaintiffs filed a reply to the above answer of the Fulton Fire Insurance,
alleging that on May 11, 1956, plaintiffs had instituted Civil Case No. 2949 in the Court of First
Instance of Manila, to assert the claim; that this case was dismissed without prejudice on
September 3, 1957 and that deducting the period with in which said action was pending, the
present action was still within the 12 month period from April 12, 1956. The court below held that
the bringing of the action in the Court of First Instance of Manila on May 11, 1956, tolled the
running of the 12 month period within which the action must be filed.
G.R. No. 85141. November 28, 1989
14. FILIPINO MERCHANTS INSURANCE CO., INC., petitioner, vs. COURT OF APPEALS and
CHOA TIEK SENG, respondents.
Insurance; An all risks policy covers all losses other than those caused by the wilful
and fraudulent act of insured.The very nature of the term all risks must be given a broad
and comprehensive meaning as covering any loss other than a wilful and fraudulent act of the
insured. This is pursuant to the very purpose of an all risks insurance to give protection to the
insured in those cases where difficulties of logical explanation or some mystery surround the loss
or damage to property. An all risks policy has been evolved to grant greater protection than
that afforded by the perils clause, in order to assure that no loss can happen through the
incidence of a cause neither insured against nor creating liability in the ship; it is written against
all losses, that is, attributable to external causes.
Same; Same; Insurer has burden of proof to show that loss is caused by an excepted
risk.Generally, the burden of proof is upon the insured to show that a loss arose from a
covered peril, but under an all risks, policy the burden is not on the insured to prove the
precise cause of loss or damage for which it seeks compensation. The insured under an all risks
insurance policy has the initial burden of proving that the cargo was in good condition when the
policy attached and that the cargo was damaged when unloaded from the vessel; thereafter, the
burden then shifts to the insurer to show the exception to the coverage. As we held in ParisManila Perfumery Co. vs. Phoenix Assurance Co., Ltd. the basic rule is that the insurance
company has the burden of proving that the loss is caused by the risks excepted and for want of
such proof, the company is liable.

Same; Insurable Interest; Perfected contract of sale even without delivery vests in the
vendee, an equitable title, an existing interest over the goods sufficient to be subject
of insurance.Herein private respondent, as vendee/consignee of the goods in transit has such
existing interest therein as may be the subject of a valid contract of insurance. His interest over
the goods is based on the perfected contract of sale. The perfected contract of sale between him
and the shipper of the goods operates to vest in him an equitable title even before delivery or
before he performed the conditions of the sale. The contract of shipment, whether under F.O.B.,
C.I.F., or C. & F. as in this case, is immaterial in the determination of whether the vendee has an
insurable interest or not in the goods in transit. The perfected contract of sale even without
delivery vests in the vendee an equitable title, an existing interest over the goods sufficient to be
the subject of insurance.
Same; Marine Insurance; Obligations and Contracts; Delivery; Delivery of goods on
board the carrying vessels partake of the nature of actual delivery.Further, Article
1523 of the Civil Code provides that where, in pursuance of a contract of sale, the seller is
authorized or required to send the goods to the buyer, delivery of the goods to a carrier, whether
named by the buyer or not, for, the purpose of transmission to the buyer is deemed to be a
delivery of the goods to the buyer, the exceptions to said rule not obtaining in the present case.
The Court has heretofore ruled that the delivery of the goods on board the carrying vessels
partake of the nature of actual delivery since, from that time, the foreign buyers assumed the
risks of loss of the goods and paid the insurance premium covering them.
REGALADO, J.:
FACTS: This is an action brought by the consignee of the shipment of fishmeal loaded on board
the vessel SS Bougainville and unloaded at the Port of Manila on or about December 11, 1976
and seeks to recover from the defendant insurance company the amount of P51,568.62
representing damages to said shipment which has been insured by the defendant insurance
company under Policy No. M-2678. The defendant brought a third party complaint against third
party defendants Compagnie Maritime Des Chargeurs Reunis and/or E. Razon, Inc. seeking
judgment against the third (sic) defendants in case judgment is rendered against the third party
plaintiff. It appears from the evidence presented that in December 1976, plaintiff insured said
shipment with defendant insurance company under said cargo Policy No. M-2678 for the sum of
P267,653.59 for the goods described as 600 metric tons of fishmeal in new gunny bags of 90
kilos each from Bangkok, Thailand to Manila against all risks under warehouse to warehouse
terms.
Actually, what was imported was 59.940 metric tons not 600 tons at $395.42 a ton CNF Manila.
The fishmeal in 666 new gunny bags were unloaded from the ship on December 11, 1976 at
Manila unto the arrastre contractor E. Razon, Inc. and defendants surveyor ascertained and
certified that in such discharge 105 bags were in bad order condition as jointly surveyed by the
ships agent and the arrastre contractor. The condition of the bad order was reflected in the turn
over survey report of Bad Order cargoes Nos. 120320 to 120322, as Exhibit C-4 consisting of
three (3) pages which are also Exhibits 4, 5 and 6-Razon. The cargo was also surveyed by the
arrastre contractor before delivery of the cargo to the consignee and the condition of the cargo
on such delivery was reflected in E. Razons Bad Order Certificate No. 14859, 14863 and 14869
covering a total of 227 bags in bad order condition. Defendants surveyor has conducted a final
and detailed survey of the cargo in the warehouse for which he prepared a survey report Exhibit
F with the findings on the extent of shortage or loss on the bad order bags totalling 227 bags
amounting to 12,148 kilos, Exhibit F-1. Based on said computation the plaintiff made a formal
claim against the defendant Filipino Merchants Insurance Company for P51,568.62 (Exhibit C) the
computation of which claim is contained therein.
A formal claim statement was also presented by the plaintiff against the vessel dated December
21, 1976, Exhibit B, but the defendant Filipino Merchants Insurance Company refused to pay the
claim. Consequently, the plaintiff brought an action against said defendant as adverted to above
and defendant presented a third party complaint against the vessel and the arrastre contractor.

G.R. No. 124050. June 19, 1997


15. MAYER STEEL PIPE CORPORATION and HONGKONG GOVERNMENT SUPPLIES
DEPARTMENT, petitioners, vs. COURT OF APPEALS, SOUTH SEA SURETY AND
INSURANCE CO., INC. and the CHARTER INSURANCE CORPORATION, respondents.
Insurance; Carriage of Goods by Sea Act; Prescription; Under Section 3(6) of the
Carriage of Goods by Sea Act, only the carriers liability is extinguished if no suit is
brought within one year.Section 3(6) of the Carriage of Goods by Sea Act states that the
carrier and the ship shall be discharged from all liability for loss or damage to the goods if no suit
is filed within one year after delivery of the goods or the date when they should have been
delivered. Under this provision, only the carriers liability is extinguished if no suit is brought
within one year. But the liability of the insurer is not extinguished because the insurers liability is
based not on the contract of carriage but on the contract of insurance. A close reading of the law
reveals that the Carriage of Goods by Sea Act governs the relationship between the carrier on
the one hand and the shipper, the consignee and/or the insurer on the other hand. It defines the
obligations of the carrier under the contract of carriage. It does not, however, affect the
relationship between the shipper and the insurer. The latter case is governed by the Insurance
Code.
Same; Same; Same; Ruling in Filipino Merchants should apply only to suits against the
carrier filed either by the shipper, the consignee or the insurer.The ruling in Filipino
Merchants should apply only to suits against the carrier filed either by the shipper, the consignee
or the insurer. When the court said in Filipino Merchants that Section 3(6) of the Carriage of
Goods by Sea Act applies to the insurer, it meant that the insurer, like the shipper, may no longer
file a claim against the carrier beyond the one-year period provided in the law. But it does not
mean that the shipper may no longer file a claim against the insurer because the basis of the
insurers liability is the insurance contract.
Same; Same; An all risks insurance policy covers all kinds of loss other than those
due to willful and fraudulent act of the insured.An insurance contract is a contract
whereby one party, for a consideration known as the premium, agrees to indemnify another for
loss or damage which he may suffer from a specified peril. An all risks insurance policy covers
all kinds of loss other than those due to willful and fraudulent act of the insured. Thus, when
private respondents issued the all risks policies to petitioner Mayer, they bound themselves to
indemnify the latter in case of loss or damage to the goods insured. Such obligation prescribes in
ten years, in accordance with Article 1144 of the New Civil Code.
PUNO, J.:
FACTS: This is a petition for review on certiorari to annul and set aside the Decision of
respondent Court of Appeals dated December 14, 19951 and its Resolution dated February 22,
19962 in CA-G.R. CV No. 45805 entitled Mayer Steel Pipe Corporation and Hongkong Government
Supplies Department v. South Sea Surety Insurance Co., Inc. and The Charter Insurance
Corporation.
In 1983, petitioner Hongkong Government Supplies Department (Hongkong) contracted
petitioner Mayer Steel Pipe Corporation (Mayer) to manufacture and supply various steel pipes
and fittings. From August to October, 1983, Mayer shipped the pipes and fittings to Hongkong as
evidenced by Invoice Nos. MSPC-1014, MSPC-1015, MSPC-1025, MSPC-1020, MSPC-1017 and
MSPC-1022.
Prior to the shipping, petitioner Mayer insured the pipes and fittings against all risks with private
respondents South Sea Surety and Insurance Co., Inc. (South Sea) and Charter Insurance Corp.
(Charter). The pipes and fittings covered by Invoice Nos. MSPC-1014, 1015 and 1025 with a total
amount of US$212,772.09 were insured with respondent South Sea, while those covered by
Invoice Nos. 1020, 1017 and 1022 with a total amount of US$149,470.00 were insured with
respondent Charter.

Petitioners Mayer and Hongkong jointly appointed Industrial Inspection (International) Inc. as
third-party inspector to examine whether the pipes and fittings are manufactured in accordance
with the specifications in the contract. Industrial Inspection certified all the pipes and fittings to
be in good order condition before they were loaded in the vessel. Nonetheless, when the goods
reached Hongkong, it was discovered that a substantial portion thereof was damaged.
Petitioners filed a claim against private respondents for indemnity under the insurance contract.
Respondent Charter paid petitioner Hongkong the amount of HK$64,904.75. Petitioners
demanded payment of the balance of HK$299,345.30 representing the cost of repair of the
damaged pipes. Private respondents refused to pay because the insurance surveyors report
allegedly showed that the damage is a factory defect.
On April 17, 1986, petitioners filed an action against private respondents to recover the sum of
HK$299,345.30. For their defense, private respondents averred that they have no obligation to
pay the amount claimed by petitioners because the damage to the goods is due to factory
defects which are not covered by the insurance policies.
The trial court ruled in favor of petitioners. It found that the damage to the goods is not due to
manufacturing defects. It also noted that the insurance contracts executed by petitioner Mayer
and private respondents are all risks policies which insure against all causes of conceivable
loss or damage. The only exceptions are those excluded in the policy, or those sustained due to
fraud or intentional misconduct on the part of the insured.
No. 33637. December 31, 1931]
16. ANG GIOK CHIP, doing business under the name and style of Hua Bee Kong Si,
plaintiff and appellee, vs. SPRINGFIELD FIRE & MARINE INSURANCE COMPANY,
defendant and appellant.
1.INSURANCE; SECTION 65, INSURANCE ACT, ACT No. 2427, AS AMENDED,
CONSTRUED; VALIDITY OF A WARRANTY IN THE FORM OF A RIDER TO AN INSURANCE
POLICY.A warranty referred to in the policy as forming part of the contract of insurance and in
the form of a rider to the insurance policy is valid and sufficient under section 65 of the
Insurance Act.
2.ID.; ID.; ID.A rider attached to the policy of insurance is a part of the contract, to the same
extent and with like effect as if actually embodied therein.
3.ID.; ID.; ID.An express warranty must appear upon the face of the policy of insurance, or be
clearly incorporated therein and made a part thereof by explicit reference, or by words clearly
evidencing such intention.
4.ID.; ID.; ID.; ACCEPTANCE OF POLICY.The receipt of a policy of insurance by the insured
without objection binds the acceptor and the insured to the terms thereof.
5.STATUTES; CONSTRUCTION OF STATUTES ADOPTED FROM OTHER STATES.The
Philippine law on insurance was taken verbatim from the law of California. Accordingly, the courts
of the Philippines should follow in fundamental points at least, the construction placed by
California courts on a California law.
MALCOLM, J.:
FACTS: Ang Giok Chip doing business under the name and style of Hua Bee Kong Si was formerly
the owner of a warehouse situated at No. 643 Calle Reina Regente, City of Manila. The contents
of the warehouse were insured with three insurance companies for the total sum of P60,000. One
insurance policy, in the amount of P10,000, was taken out with the Springfield Fire & Marine

Insurance Company. The warehouse was destroyed by fire on January 11, 1928; while the policy
issued by the latter company was in force.
Predicated on this policy the plaintiff instituted action in the Court of First Instance of Manila
against the defendant to recover a proportional part of the loss coming to P8,170.59. Four special
defenses were interposed on behalf of the insurance company, one being planted on a violation
of warranty F fixing the amount of hazardous goods which might be stored in the insured
building. The trial judge in his decision found against the insurance company on all points, and
gave judgment in favor of the plaintiff for the sum of P8,188.74. From this judgment the
insurance company has appealed, and it is to the first and fourth errors assigned that we would
address particular attention.
Considering the result at which we arrive, it is unnecessary for us to discuss three of the four
special defenses which were made by the insurance company. We think, however, that it would
be a reasonable deduction to conclude that more than 3 per cent of the total value of the
merchandise contained in the warehouse constituted hazardous goods, and that this per cent
reached as high as 39. We place reliance on the consular invoices and on the testimony of the
adjuster, Herridge. Having thus swept to one side all intervening obstacles, the legal question
recurs, as stated in the beginning of this decision, of whether or not warranty F was null and
void.
To place this question in its proper light, we turn to the policy issued by the Springfield Fire &
Marine Insurance Company in favor of the plaintiff. The description of the risk in this policy is as
follows:
"Ten thousand pesos Philippine Currency.On general non-hazardous merchandise, chiefly
consisting of chucherias, also produce, Cacao, Flour, all the property of the Insured, or held by
them in trust, on commission or on joint account with others, or for which he is responsible, while
contained during the currency of this policy in the godown, situate No. 643 Calle Reina Regente.
*
*
*
"This policy is subject to the hereon attached 'Ordinary Short Period Rate Scale' Warranties A & F,
Co-insurances Clause 'and Three Fourths Loss Clause,' which are forming part of same. Coinsurance declared:
"P20,000.Sun Insurance Office Ltd. (K & S)." (Italics inserted.) Securely pasted on the left hand
margin of the face of the policy are five warranties and special clauses. One of them is warranty
F, specifically referred to on the f ace of the policy, reading in part as follows:
"WARRANTY F
"It is hereby declared and agreed that during the currency of this policy no hazardous goods be
stored in the Building to which this insurance applies or in any building communicating therewith,
provided, always, however, that the Insured be permitted to store a small quantity of the
hazardous goods specified below, but not exceeding in all 3 per cent of the total value of the
whole of the goods or merchandise contained in said warehouse, viz; *
*
*."
The applicable law is found in the Insurance Act, Act No. 2427, as amended, section 65 reading:
"Every express warranty, made at or before the execution of a policy, must be contained in the
policy itself, or in another instrument signed by the insured and referred to in the policy, as
making a part of it." As the Philippine law was taken verbatim from the law of California, in
accordance with well settled canons of statutory construction, the court should follow in
fundamental points, at least, the construction placed by California courts on a California law.
Unfortunately the researches of counsel reveal no authority coming from the courts of California
which is exactly on all fours with the case before us. However, there are certain considerations
lying at the basis of California law and certain indications in the California decisions which point
the way for the decision in this case.

Section 65 of the Philippine Insurance Act corresponds to section 2605 of the Civil Code of
California. The comments of the Code Examiners of California disclose that the language of
section 2605 was quite diff rent from that under the Code as adopted in 1872. That language
was f ound too harsh as to insurance companies. The Code Examiners' notes state: "The
amendment restores the law as it existed previous to the Code: See Parsons on Maritime Law,
106, and Phillips on Insurance, sec. 756." The passage referred to in Phillips on Insurance, was
worded by the author as follows:
"Any express warranty or condition is always a part of the policy, but, like any other part of an
express contract, may be written in the margin, or contained in proposals or documents
expressly referred to in the policy, and so made a part of it." The annotator of the Civil Code of
Calif ornia, after setting forth these facts, adds:
"*
*
* The section as it now reads is in harmony with the rule that a warranty may be
contained in another instrument than the policy when expressly referred to in the policy as
forming a part thereof: *
*
*."
What we have above stated has been paraphrased from the decision of the California Court of
Appeals in the case of Isaac Upham Co. vs. United States Fidelity & Guaranty Co. ([1922], 211
Pac., 809), and thus discloses the attitude of the California courts. Likewise in the Federal courts,
in the case of Conner vs. Manchester Assur. Co. ([1904], 130 Fed., 743), section 2605 of the Civil
Code of California came under observation, and it was said that it "is in effect an affirmance of
the generally accepted doctrine applicable to such contracts."
We, therefore, think it wrong to hold that the California law represents a radical departure from
the basic principles governing the law of insurance. We are more inclined to believe that the
codification of the law of California had exactly the opposite purpose, and that in the language of
the Federal court it was but an affirmance of the generally accepted doctrine applicable to such
contracts.
No. L-67835. October 12, 1987.
17. MALAYAN INSURANCE CO., INC. (MICO), petitioner, us. GREGORIA CRUZ ARNALDO,
in her capacity as the INSURANCE COMMISSIONER, and CORONACION PINCA,
respondents.
Mercantile Law; Insurance; Fire Insurance Policy; A valid cancellation of policy
requires the concurrence of the following conditions 1) Prior notice of cancellation to
insured; 2) Notice must be based on the occurrence after effective date of policy of
one or more of the grounds mentioned; 3) Must be in writing, mailed or delivered to
the insured at the address shown in the policy; 4) Must state the grounds relied upon
provided in Sec. 64 of the Insurance Code and upon request of insured, to furnish
facts on which cancellation is based.MICO claims it cancelled the policy in question on
October 15, 1981, for non-payment of premium. To support this assertion, it presented one of its
employees, who testified that "the original of the endorsement and credit memo"presumably
meaning the alleged cancellation"were sent the assured by mail through our mailing section."
However, there is no proof that the notice, assuming it complied with the other requisites
mentioned above, was actually mailed to and received by Pinca. All MICO offers to show that the
cancellation was communicated to the insured is its employee's testimony that the said
cancellation was sent "by mail through our mailing section," without more. The petitioner then
says that its "stand is enervated (sic) by the legal presumption of regularity and due
performance of duty," (not realizing perhaps that "enervated" means "debilitated," not
"strengthened").
Same; Same, Same; A valuation fixed in the fire insurance policy is conclusive in case
of total loss in the absence of fraudThe last point raised by the petitioner should not pose
much difficulty. The valuation fixed in fire insurance policy is conclusive in case of total loss in the
absence of fraud, which is not shown here Loss and its amount may be determined on the basis

of such proof as may be offered by the insured, which need not be of such persuasiveness as is
required in judicial proceedings. If, as in this case, the insured files notice and preliminary proof
of loss and the insurer fails to specify to the former all the defects thereof and without
unnecessary delay, all objections to notice and proof of loss are deemed waived under Section
90 of the Insurance Code.
CRUZ, J.:
FACTS: On June 7, 1981, the petitioner (hereinafter called (MICO) issued to the private
respondent, Coronacion Pinca, Fire Insurance Policy No. F-001-17212 on her property for the
amount of P100,000.00, effective July 22, 1981, until July 22, 1982.
On October 15, 1981, MICO allegedly cancelled the policy for non-payment, of the premium and
sent the corresponding notice to Pinca.
On December 24, 1981, payment of the premium for Pinca was received by Domingo Adora,
agent of MICO.
On January 15, 1982, Adora remitted this payment to MICO, together with other payments.
On January 18, 1982, Pinca's property was completely burned.
On February 5, 1982, Pinca's payment was returned by MICO to Adora on the ground that her
policy had been cancelled earlier. But Adora refused to accept it.
In due time, Pinca made the requisite demands for payment, which MICO rejected. She then went
to the Insurance Commission. It is because she was ultimately sustained by the public
respondent that the petitioner has come to us for relief.
From the procedural viewpoint alone, the petition must be rejected. It is stillborn.
The records show that notice of the decision of the public respondent dated April 5, 1982, was
received by MICO on April 10, 1982.8 On April 25, 1982, it filed a motion for reconsideration,
which was denied on June 4, 1982.9 Notice of this denial was received by MICO on June 13, 1982,
as evidenced by Annex "1," duly authenticated by the Insurance Commission.10 The instant
petition was filed with this Court on July 2, 1982.
The position of the petitioner is that the petition is governed by Section 416 of the Insurance
Code giving it thirty days within which to appeal by certiorari to this Court. Alternatively, it also
invokes Rule 45 of the Rules of Court, For their part, the public and private respondents insist
that the applicable law is B.P. 129, which they say governs not only courts of justice but also
quasi-judicial bodies like the Insurance Commission. The period for appeal under this law is also
fifteen days, as under Rule
The pivotal date is the date the notice of the denial of the motion for reconsideration was
received by MICO.
MICO avers this was June 18, 1982, and offers in evidence its Annex "B,"12 which is a copy of the
Order of June 14,1982, with a signed rubber-stamped notation on the upper left-hand corner that
it was received on June 18, 1982, by its legal department. It does not indicate from whom. At the
bottom, significantly, there is another signature under which are the ciphers "6-13-82," for which
no explanation has been given.
Against this document, the private respondent points in her Annex "1,"13 the authenticated copy
of the same Order with a rubber-stamped notation at the bottom thereof indicating that it was
received for the Malayan Insurance Co., Inc. by J. Gotladera on "6-13-82." The signature may or

may not have been written by the same person who signed at the bottom of the petitioner's
Annex "B."
Between the two dates, the court chooses to believe June 13, 1982, not only because the
numbers "6-13-82" appear on both annexes but also because it is the date authenticated by the
administrative division of the Insurance Commission. Annex "B" is at worst self-serving; at best, it
might only indicate that it was received on June 18, 1982, by the legal department of MICO, after
it had been received earlier by some other of its personnel on June 13, 1982. Whatever the
reason for the delay in transmitting it to the legal department need not detain us here.
Under Section 416 of the Insurance Code, the period for appeal is thirty days from notice of the
decision of the Insurance Commission. The petitioner filed its motion for reconsideration on April
25, 1981, or fifteen days from such notice, and the reglementary period began to run again after
June 13, 1981, date of its receipt of notice of the denial of the said motion for reconsideration. As
the herein petition was filed on July 2, 1981, or nineteen days later, there is no question that it is
tardy by four days.
Counted from June 13, the fifteen-day period prescribed under Rule 45, assuming it is applicable,
would end on June 28, 1982, or also four days from July 2, when the petition was filed.
If it was filed under B.P. 129, then, considering that the motion for reconsideration was filed on
the fifteenth day after MICO received notice of the decision, only one more day would have
remained for it to appeal to wit, June 14, 1982. That would make the petition eighteen days late
by July 2.
Indeed, even if the applicable law were still R.A. 5434, governing appeals from administrative
bodies, the petition would still be tardy. The law provides for a fixed period of ten days from
notice of the denial of a seasonable motion for reconsideration within which to appeal from the
decision. Accordingly, that ten-day period, counted from June 13, 1982, would have ended on
June 23,1982, making the petition filed on July 2, 1982 nine days late.
Whichever law is applicable, therefore, the petition can and should be dismissed for late filing.
On the merits, it must also fail. MICO's arguments that there was no payment of premium and
that the policy had been cancelled before the occurrence of the loss are not acceptable. Its
contention that the claim was allowed without proof of loss is also untenable.
The petitioner relies heavily on Section 77 of the Insurance Code providing that:
"SEC. 77, An insurer is entitled to payment of the premium as soon as the thing is exposed to the
peril insured against. Notwithstanding any agreement to the contrary, no policy or contract of
insurance issued by an insurance company is valid and binding unless and until the premium
thereof has been paid, except in the case of a life or an industrial life policy whenever the grace
period pro vision applies."
The above provision is not applicable because payment of the premium was in fact eventually
made in this case. Notably, the premium invoice issued to Pinca at the time of the delivery of the
policy on June 7, 1981 was stamped "Payment Received" of the amount of P930.60 on "12-24-81"
by Domingo Adora.14 This is important because it suggests an understanding between MICO and
the insured that such payment could be made later, as agent Adora had assured Pinca. In any
event, it is not denied that this payment was actually made by Pinca to Adora, who remitted the
same to MICO. It is not disputed that the premium was actually paid by Pinca to Adora on
December 24, 1981, who received it on behalf of MICO, to which it was remitted on January 15,
1982. What is questioned is the validity of Pinca's payment and of Adora's authority to receive it.
WARRANTIES (SECTIONS 67-68)

1. PRUDENTIAL GUARANTEE and ASSURANCE, INC., petitioner, vs. TRANS-ASIA


SHIPPING LINES, INC., respondent.
[491 SCRA 411(2006)]
Insurance Law; Maritime Law; Bureau Veritas is a classification society recognized in
the marine industry.As found by the Court of Appeals and as supported by the records,
Bureau Veritas is a classification society recognized in the marine industry. As it is undisputed
that TRANS-ASIA was properly classed at the time the contract of insurance was entered into,
thus, it becomes incumbent upon PRUDENTIAL to show evidence that the status of TRANS-ASIA
as being properly CLASSED by Bureau Veritas had shifted in violation of the warranty.
Unfortunately, PRUDENTIAL failed to support the allegation.
Same; Same; Warranties; It is generally accepted that a warranty is a statement or
promise set forth in the policy, or by reference incorporated therein, the untruth or
non-fulfillment of which in any respect, and without reference to whether the insurer
was in fact prejudiced by such untruth or non-fulfillment, renders the policy voidable
by the insurer; For the breach of warranty to avoid a policy, the same must be duly
shown by the party alleging the same.We are not unmindful of the clear language of Sec.
74 of the Insurance Code which provides that, the violation of a material warranty, or other
material provision of a policy on the part of either party thereto, entitles the other to rescind. It
is generally accepted that [a] warranty is a statement or promise set forth in the policy, or by
reference incorporated therein, the untruth or non-fulfillment of which in any respect, and
without reference to whether the insurer was in fact prejudiced by such untruth or nonfulfillment, renders the policy voidable by the insurer. However, it is similarly indubitable that for
the breach of a warranty to avoid a policy, the same must be duly shown by the party alleging
the same. We cannot sustain an allegation that is unfounded. Consequently, PRUDENTIAL, not
having shown that TRANS-ASIA breached the warranty condition, CLASSED AND CLASS
MAINTAINED, it remains that TRANSASIA must be allowed to recover its rightful claims on the
policy.
Same; Same; Same; Waivers; Breach of warranty or of a condition renders the
contract defeasible at the option of the insurer, but if he so elects, he may waive his
privilege and power to rescind by the mere expression of an intention to do so in
which event his liability under the policy continues as before.We do not find that the
Court of Appeals was in error when it held that PRUDENTIAL, in renewing TRANS-ASIAs insurance
policy for two consecutive years after the loss covered by Policy No. MH93/1363, was considered
to have waived TRANS-ASIAs breach of the subject warranty, if any. Breach of a warranty or of a
condition renders the contract defeasible at the option of the insurer; but if he so elects, he may
waive his privilege and power to rescind by the mere expression of an intention so to do. In that
event his liability under the policy continues as before. There can be no clearer intention of the
waiver of the alleged breach than the renewal of the policy insurance granted by PRUDENTIAL to
TRANS-ASIA in MH94/1595 and MH95/1788, issued in the years 1994 and 1995, respectively. To
our mind, the argument is made even more credulous by PRUDENTIALs lack of proof to support
its allegation that the renewals of the policies were taken only after a request was made to
TRANS-ASIA to furnish them a copy of the certificate attesting that M/V Asia Korea was
CLASSED AND CLASS MAINTAINED. Notwithstanding PRUDENTIALs claim that no certification was
issued to that effect, it renewed the policy, thereby, evidencing an intention to waive TRANSASIAs alleged breach. Clearly, by granting the renewal policies twice and successively after the
loss, the intent was to benefit the insured, TRANSASIA, as well as to waive compliance of the
warranty.
Same; Same; Loan and Trust Receipts; Notwithstanding its designation, the tenor of
the Loan and Trust Receipt evidences that the real nature of the transaction
between the parties was that the amount indicated therein was not intended as a
loan whereby the insured is obligated to pay the insurer, but rather, the same was a
partial payment or an advance on the policy of the claims due the former.The Court of

Appeals held that the real character of the transaction between the parties as evidenced by the
Loan and Trust Receipt is that of an advance payment by PRUDENTIAL of TRANSASIAs claim on
the insurance, thus: x x x We agree. Notwithstanding its designation, the tenor of the Loan and
Trust Receipt evidences that the real nature of the transaction between the parties was that the
amount of P3,000,000.00 was not intended as a loan whereby TRANS-ASIA is obligated to pay
PRUDENTIAL, but rather, the same was a partial payment or an advance on the policy of the
claims due to TRANS-ASIA.
Same; Same; Same; Words and Phrases; The clear import of the phrase at the
expense of and under the exclusive direction and control as used in the Loan and
Trust Receipt grants solely to the insurer the power to prosecute, even as the same
is carried in the name of the insured, thereby making the latter merely an agent of
the former, the principal, in the prosecution of the suit against parties who may have
occasioned the loss.We find that per the Loan and Trust Receipt, even as TRANS-ASIA
agreed to promptly prosecute suit against such persons, corporation or corporations through
whose negligence the aforesaid loss was caused or who may otherwise be responsible therefore,
with all due diligence in its name, the prosecution of the claims against such third persons are
to be carried on at the expense of and under the exclusive direction and control of PRUDENTIAL
GUARANTEE AND ASSURANCE INC. The clear port its allegation that the renewals of the policies
were taken only after a request was made to TRANS-ASIA to furnish them a copy of the certificate
attesting that M/V Asia Korea was CLASSED AND CLASS MAINTAINED. Notwithstanding
PRUDENTIALs claim that no certification was issued to that effect, it renewed the policy, thereby,
evidencing an intention to waive TRANS-ASIAs alleged breach. Clearly, by granting the renewal
policies twice and successively after the loss, the intent was to benefit the insured, TRANSASIA,
as well as to waive compliance of the warranty.
Same; Same; Loan and Trust Receipts; Notwithstanding its designation, the tenor of
the Loan and Trust Receipt evidences that the real nature of the transaction
between the parties was that the amount indicated therein was not intended as a
loan whereby the insured is obligated to pay the insurer, but rather, the same was a
partial payment or an advance on the policy of the claims due the former.The Court of
Appeals held that the real character of the transaction between the parties as evidenced by the
Loan and Trust Receipt is that of an advance payment by PRUDENTIAL of TRANSASIAs claim on
the insurance, thus: x x x We agree. Notwithstanding its designation, the tenor of the Loan and
Trust Receipt evidences that the real nature of the transaction between the parties was that the
amount of P3,000,000.00 was not intended as a loan whereby TRANS-ASIA is obligated to pay
PRUDENTIAL, but rather, the same was a partial payment or an advance on the policy of the
claims due to TRANS-ASIA.
Same; Same; Same; Words and Phrases; The clear import of the phrase at the
expense of and under the exclusive direction and control as used in the Loan and
Trust Receipt grants solely to the insurer the power to prosecute, even as the same
is carried in the name of the insured, thereby making the latter merely an agent of
the former, the principal, in the prosecution of the suit against parties who may have
occasioned the loss.We find that per the Loan and Trust Receipt, even as TRANS-ASIA
agreed to promptly prosecute suit against such persons, corporation or corporations through
whose negligence the aforesaid loss was caused or who may otherwise be responsible therefore,
with all due diligence in its name, the prosecution of the claims against such third persons are
to be carried on at the expense of and under the exclusive direction and control of PRUDENTIAL
GUARANTEE AND ASSURANCE INC. The clear scribed by the Monetary Board on the amount of
the claim due the insured from the date following the time prescribed in Section 242 or in Section
243, as the case may be, until the claim is fully satisfied. Finally, Section 244 considers the
failure to pay the claims within the time prescribed in Sections 242 or 243, when applicable, as
prima facie evidence of unreasonable delay in payment. To the mind of this Court, Section 244
does not require a showing of bad faith in order that attorneys fees be granted. As earlier stated,
under Section 244, a prima facie evidence of unreasonable delay in payment of the claim is
created by failure of the insurer to pay the claim within the time fixed in both Sections 242 and

243 of the Insurance Code. As established in Section 244, by reason of the delay and the
consequent filing of the suit by the insured, the insurers shall be adjudged to pay damages which
shall consist of attorneys fees and other expenses incurred by the insured.
Same; Same; Same; Interests; Marine Insurance; Section 244 of the Insurance Code is
categorical in imposing an interest twice the ceiling prescribed by the Monetary Board
due the insured, from the date following the time prescribed in Section 242 or in
Section 243, as the case may be, until the claim is fully satisfied.Section 244 of the
Insurance Code is categorical in imposing an interest twice the ceiling prescribed by the
Monetary Board due the insured, from the date following the time prescribed in Section 242 or in
Section 243, as the case may be, until the claim is fully satisfied. In the case at bar, we find
Section 243 to be applicable as what is involved herein is a marine insurance, clearly, a policy
other than life insurance. Section 243 is hereunder reproduced: SEC. 243. The amount of any loss
or damage for which an insurer may be liable, under any policy other than life insurance policy,
shall be paid within thirty days after proof of loss is received by the insurer and ascertainment of
the loss or damage is made either by agreement between the insured and the insurer or by
arbitration; but if such ascertainment is not had or made within sixty days after such receipt by
the insurer of the proof of loss, then the loss or damage shall be paid within ninety days after
such receipt. Refusal or failure to pay the loss or damage within the time prescribed herein will
entitle the assured to collect interest on the proceeds of the policy for the duration of the delay
at the rate of twice the ceiling prescribed by the Monetary Board, unless such failure or refusal to
pay is based on the ground that the claim is fraudulent.
CHICO-NAZARIO, J.:
FACTS: Plaintiff [TRANS-ASIA] is the owner of the vessel M/V Asia Korea. In consideration of
payment of premiums, defendant [PRUDENTIAL] insured M/V Asia Korea for loss/damage of the
hull and machinery arising from perils, inter alia, of fire and explosion for the sum of P40 Million,
beginning [from] the period [of] July 1, 1993 up to July 1, 1994. This is evidenced by Marine
Policy No. MH93/1363 (Exhibits A to A-11). On October 25, 1993, while the policy was in force,
a fire broke out while [M/V Asia Korea was] undergoing repairs at the port of Cebu. On October
26, 1993 plaintiff [TRANS-ASIA] filed its notice of claim for damage sustained by the vessel. This
is evidenced by a letter/formal claim of even date (Exhibit B). Plaintiff [TRANS-ASIA] reserved
its right to subsequently notify defendant [PRUDENTIAL] as to the full amount of the claim upon
final survey and determination by average adjuster Richard Hogg International (Phils.) of the
damage sustained by reason of fire. An adjusters report on the fire in question was submitted by
Richard Hogg International together with the U-Marine Surveyor Report (Exhibits 4 to 4-115).
On May 29, 1995[,] plaintiff [TRANS-ASIA] executed a document denominated Loan and Trust
receipt, a portion of which read (sic):
Received from Prudential Guarantee and Assurance, Inc., the sum of PESOS THREE MILLION
ONLY (P3,000,000.00) as a loan without interest under Policy No. MH 93/1353 [sic], repayable
only in the event and to the extent that any net recovery is made by Trans-Asia Shipping
Corporation, from any person or persons, corporation or corporations, or other parties, on
account of loss by any casualty for which they may be liable occasioned by the 25 October 1993:
Fire on Board. (Exhibit 4)
In a letter dated 21 April 1997 defendant [PRUDENTIAL] denied plaintiffs claim (Exhibit 5). The
letter reads:
After a careful review and evaluation of your claim arising from the above-captioned incident, it
has been ascertained that you are in breach of policy conditions, among them WARRANTED
VESSEL CLASSED AND CLASS MAINTAINED. Accordingly, we regret to advise that your claim is
not compensable and hereby DENIED.
This was followed by defendants letter dated 21 July 1997 requesting the return or payment of
the P3,000,000.00 within a period of ten (10) days from receipt of the letter (Exhibit 6).

Following this development, on 13 August 1997, TRANS-ASIA filed a Complaint5 for Sum of
Money against PRUDENTIAL with the RTC of Cebu City, docketed as Civil Case No. CEB-20709,
wherein TRANS-ASIA sought the amount of P8,395,072.26 from PRUDENTIAL, alleging that the
same represents the balance of the indemnity due upon the insurance policy in the total amount
of P11,395,072.26. TRANS-ASIA similarly sought interest at 42% per annum citing Section 2436
of Presidential Decreee No. 1460, otherwise known as the Insurance Code, as amended.
[No. 9370. March 31, 1915.]
2. K. S. YOUNG, plaintiff and appellee, vs. THE MIDLAND TEXTILE INSURANCE
COMPANY, defendant and appellant.
1.INSURANCE; EFFECT OF VIOLATION OF CONTRACT OF.Contracts of insurance are
contracts of indemnity, upon the terms and conditions specified therein. Parties have a right to
impose such reasonable conditions at the time of the making of the contract as they deem wise
and necessary. The rate of premium is measured by the character of the risk assumed. The
insurer, for a comparatively small consideration, undertakes to guarantee the insured against
loss or damage, upon the terms and conditions agreed upon, and upon no other. When the
insurer is called upon to pay, in case of loss, he may justly insist upon a fulfillment of the terms
of the contract. If the insured cannot bring- himself within the terms and conditions of the
contract, he is not entitled to recover for any loss suffered. The terms of the contract constitute
the measure of the insurer's liability. If the contract has been terminated, by a violation of its
terms on the part of the insured, there can be no recovery. Compliance with the terms of the
contract is a condition precedent to the right of recovery.
Courts cannot make contracts for the parties. While contracts of insurance are construed most
favorably to the insured, yet they must be construed according to the sense and meaning of the
terms which the parties themselves have used. Astute and subtle distinctions should not be
permitted, when the language of the contract is plain and unambiguous. Such distinctions tend
to bring the law itself into disrepute.
2.ID. ; "STORED"; STORING.The word "stored" has been defined to be a deposit in a store or
warehouse for preservation or safe keeping; to put away for future use, especially for future
consumption; to place in a warehouse or other place of deposit for safe keeping. Said definition
does not include a deposit in a store, in small quantities, for daily use. "Daily use" precludes the
idea of deposit for preservation or safe keeping, as well as a deposit for future consumption or
safe keeping.
3.ID.; VIOLATION OF TERMS OF CONTRACT WHICH DOES NOT CONTRIBUTE TO LOSS OR
INJURY.A violation of the terms of a contract of insurance, by either party, will constitute the
basis for a termination of the contractual relations, at the election of the other. The right to
terminate the contractual relations exists, even though the violation was not the direct cause of
the loss. In the present case, the deposit of the "hazardous goods," in the building insured, was a
violation of the terms of the contract. Although the hazardous goods did not contribute to the
loss, the insurer, at his election, was relieved from liability. Said deposit created a new risk, not
included in the terms of the contract. The insurer had neither been paid, nor had he entered into
a contract, to cover the increased risk.
JOHNSON, J.:
FACTS: 1. The plaintiff conducted a candy and fruit store on the Escolta, in the city of Manila,
and occupied a building at 321 Calle Claveria, as a residence and bodega (storehouse).
2. On the 29th of May, 1912, the defendant, in consideration of the payment of a premium of
P60, entered into a contract of insurance with the plaintiff (policy No. 509105) by the terms of
which the defendant company, upon certain conditions, promised to pay to the plaintiff the sum
of P3,000, in case said residence and bodega and contents should be destroyed by fire.

3. One of the conditions of said contract of insurance is found in "warranty B" and is as follows:
"Warranty B.It is hereby declared and agreed that during the pendency of this policy no
hazardous goods be stored or kept for sale, and no hazardous trade or process be carried on, in
the building to which this insurance applies, or in any building connected therewith."
4. On the 4th or 5th of February, 1913, the plaintiff placed in said residence and bodega, three
boxes, 18 by 18 by 20 inches measurement, which belonged to him and which were filled with
fireworks.
5. On the 18th day of March, 1913, said residence and bodega and the contents thereof were
partially destroyed by fire.
6. Said fireworks had been given to the plaintiff by the former owner of the Luneta Candy Store;
that the plaintiff intended to use the same in the celebration of the Chinese new year; that the
authorities of the city of Manila had prohibited the use of fireworks on said occasion, and that the
plaintiff then placed the same in said bodega, where they remained from the 4th or 5th of
February, 1913, until after the fire of the 18th of March, 1913.
7. Both of the parties agree that said fireworks come within the phrase "hazardous goods,"
mentioned in said "warranty B" of the policy.
8. That said fireworks were found in a part of the building not destroyed by the fire; that they in
no way contributed to the fire, or to the loss occasioned thereby.
The only question presented by the parties is whether or not the placing of said fireworks in the
building insured, under the conditions above enumerated, they being "hazardous goods," is a
violation of the terms of the contract of insurance and especially of "warranty B."
G.R. No. 138941. October 8, 2001.
AMERICAN HOME ASSURANCE COMPANY, petitioner, vs. TANTUCO ENTERPRISES, INC.,
respondent.
Insurance; In construing the words used descriptive of a building insured, the
greatest liberality is shown by the courts in giving effect to the insurance.In
construing the words used descriptive of a building insured, the greatest liberality is shown by
the courts in giving effect to the insurance. In view of the custom of insurance agents to examine
buildings before writing policies upon them, and since a mistake as to the identity and character
of the building is extremely unlikely, the courts are inclined to consider that the policy of
insurance covers any building which the parties manifestly intended to insure, however
inaccurate the description may be. Notwithstanding, therefore, the misdescription in the policy, it
is beyond dispute, to our mind, that what the parties manifestly intended to insure was the new
oil mill.
Same; Pleadings and Practice; Parole Evidence Rule; A party may present evidence to
modify, explain or add to the terms of the written agreement if he puts in issue in his
pleading, among others, its failure to express the true intent and agreement of the
parties thereto.These facts lead us to hold that the present case falls within one of the
recognized exceptions to the parole evidence rule. Under the Rules of Court, a party may present
evidence to modify, explain or add to the terms of the written agreement if he puts in issue in his
pleading, among others, its failure to express the true intent and agreement of the parties
thereto. Here, the contractual intention of the parties cannot be understood from a mere reading
of the instrument. Thus, while the contract explicitly stipulated that it was for the insurance of
the new oil mill, the boundary description written on the policy concededly pertains to the first oil
mill. This irreconcilable difference can only be clarified by admitting evidence aliunde, which will
explain the imperfection and clarify the intent of the parties.

Same; In determining what the parties intended, the courts will read and construe the
policy as a whole and if possible, give effect to all the parts of the contract, keeping in
mind always, however, the prime rule that in the event of doubt, this doubt is to be
resolved against the insurer.We again stress that the object of the court in construing a
contract is to ascertain the intent of the parties to the contract and to enforce the agreement
which the parties have entered into. In determining what the parties intended, the courts will
read and construe the policy as a whole and if possible, give effect to all the parts of the
contract, keeping in mind always, however, the prime rule that in the event of doubt, this doubt
is to be resolved against the insurer. In determining the intent of the parties to the contract, the
courts will consider the purpose and object of the contract.
Same; Not only are warranties strictly construed against the insurer, but they should,
likewise, by themselves be reasonably interpreted.It ought to be remembered that not
only are warranties strictly construed against the insurer, but they should, likewise, by
themselves be reasonably interpreted. That reasonableness is to be ascertained in light of the
factual conditions prevailing in each case. Here, we find that there is no more need for an
internal hydrant considering that inside the burned building were: (1) numerous portable fire
extinguishers, (2) an emergency fire engine, and (3) a fire hose which has a connection to one of
the external hydrants.
PUNO, J.:
FACTS: Respondent Tantuco Enterprises, Inc. is engaged in the coconut oil milling and refining
industry. It owns two oil mills. Both are located at its factory compound at Iyam, Lucena City. It
appears that respondent commenced its business operations with only one oil mill. In 1988, it
started operating its second oil mill. The latter came to be commonly referred to as the new oil
mill.
The two oil mills were separately covered by fire insurance policies issued by petitioner American
Home Assurance Co., Philippine Branch.1 The first oil mill was insured for three million pesos
(P3,000,000.00) under Policy No. 306-7432324-3 for the period March 1, 1991 to 1992.2 The new
oil mill was insured for six million pesos (P6,000,000.00) under Policy No. 306-7432321-9 for the
same term. Official receipts indicating payment for the full amount of the premium were issued
by the petitioners agent.
A fire that broke out in the early morning of September 30, 1991 gutted and consumed the new
oil mill. Respondent immediately notified the petitioner of the incident. The latter then sent its
appraisers who inspected the burned premises and the properties destroyed. Thereafter, in a
letter dated October 15, 1991, petitioner rejected respondents claim for the insurance proceeds
on the ground that no policy was issued by it covering the burned oil mill. It stated that the
description of the insured establishment referred to another building thus: Our policy Nos. 3067432321-9 (Ps 6M) and 306-7432324-4 (Ps 3M) extend insurance coverage to your oil mill under
Building No. 5, whilst the affected oil mill was under Building No. 14.
A complaint for specific performance and damages was consequently instituted by the
respondent with the RTC, Branch 53 of Lucena City.
4. ANG GIOK CHIP V. SPRINGFIELD FIRE AND MARITIME INSURANCE
(56 PHIL 375)
5. QUA CHE GAN V. LAW UNION AND ROCKS INSURANCE
(GR NO. L-4611)

6. E. M. BACHRACH, plaintiff and appellee, vs. BRITISH AMERICAN ASSURANCE


COMPANY, a corporation, defendant and appellant (17 PHIL 555)
1.FIRE INSURANCE; CONDITIONS RELIED UPON MUST BE EXPRESSED IN POLICY.When
property is insured any condition upon which the insurer wishes to rely, in order to avoid liability
in case of a loss, must be expressed in the policy.
2.ID.; ALIENATION; EXECUTION OF A CHATTEL MORTGAGE UPON INSURED PROPERTY.
Interest in property insured does not pass by the mere execution of a chattel mortgage, and,
while the chattel mortgage is a conditional sale, there is no alienation, within the meaning of the
insurance law, until the mortgagee acquires a right to take possession by default under the
terms of the mortgage.
3.ID.; SUFFICIENCY OF EVIDENCE IN A CIVIL SUIT FOLLOWING A CRIMINAL
PROSECUTION.The evidence in a civil suit, following an unsuccessful criminal prosecution
involving the same subject matter, should not be materially less convincing than that required to
convict the accused of the alleged crime.
4.ID.; NOTICE OF LOSS; WAIVER OF NOTICE BY INSURERS.Where the terms of an insurance
policy require that notice of loss be given, a denial of liability by the insurers under the policy
operates as a waiver of notice of loss because if the policy is null and void the furnishing of such
notice would be vain and useless. Immediate notice means within a reasonable time.
JOHNSON, J.:
FACTS: On the 13th of July, 1908, the plaintiff commenced an action against the defendant to
recover the sum of P9,841.50, the amount due, deducting the salvage, upon the following fire
insurance policy issued by the defendant to the plaintiff:
"[Fire policy No. 3007499.]
"This policy of insurance witnesseth, that E. M. Bachrach, esq., Manila (hereinafter called the
insured), having paid to the undersigned, as authorized agent of the British American Assurance
Company (hereinafter called the company), the sum of two thousand pesos Philippine currency,
for insuring against loss or damage by fire, as hereinafter mentioned, the property hereinafter
described, in the sum of several sums f ollowing, viz:
"Ten thousand pesos Philippine currency, on goods, belonging to a general furniture store, such
as iron and brass bedsteads, toilet tables, chairs, ice boxes, bureaus, washstands, mirrors, and
sea-grass furniture (in accordance with warranty 'D' of the tariff attached hereto) the property of
the assured, in trust, on commission or for which he is responsible, whilst stored in the ground
floor and first story of house and dwelling No. 16 Calle Martinez, district 3, block 70, Manila, built,
ground floor of stone and or brick, first story of hard wood and roofed with galvanized iron
bounded in the front by the said calle, on one side by Calle David and on the other two sides by
buildings of similar construction and occupation.
"Co-insurances allowed, particulars of which to be declared in the event of loss or claim.
"The company hereby agrees with the insured (but subject to the conditions on the back hereof,
which are to be taken as a part of this policy) that -if the property above described, or any part
thereof, shall be destroyed or damaged by fire, at any time between the 21st day of February,
1908, and 4 o'clock in the afternoon of the 21st day of February, 1909, or (in case of the renewal
of .this policy) at any time afterwards, so long as, and during the period in respect of which the
insured shall have paid to the company, and they shall have accepted, the sum required for the
renewal of this policy, the company will, out of their capital stock, and f unds, pay or make good
to the insured the value of the property so destroyed, or the amount of such damage thereto, to
any amount not exceeding, in respect of each or any of the several matters above specified, the
sum set opposite thereto, respectively, and not exceeding in the whole the sum of ten thousand
pesos, and also not exceeding, in any case, the amount of the insurable interest therein of the
insured at the time of the happening of such fire.

"In witness whereof, the British American Assurance Company has caused these presents to be
signed this 21st day of February, in the year of our Lord 1908.
"For the company.
"W. F. STEVENSON & Co., LTD.,
"By ....................................... ,
"Manager Agents"
And indorsed on the back the following:
"The within policy covers and includes a 'Calalac' automobile to the extent of (P1,250) twelve
hundred and fifty pesos Philippine currency.
"Memo: Permission is hereby granted for the use of gasoline not to exceed 10 gallons for the
above automobile, but only whilst contained in the reservoir of the car. It is further warranted
that the car be neither filled nor emptied in the within-described building or this policy be null
and void,
"Manila, 27th February, 1908.
"W. F. STEVENSON & Co., LTD.,
"By ............................................ ,
"Manager Agents."

G.R. No. 114427. February 6, 1995.


7. ARMANDO GEAGONIA, petitioner, vs. COURT OF APPEALS and COUNTRY BANKERS
INSURANCE CORPORATION, respondents.
Insurance; The incorporation of Condition 3 in the policy is allowed by Section 75 of
the Insurance Code.Condition 3 of the private respondent's Policy No. F-14622 is a condition
which is not proscribed by law. Its incorporation in the policy is allowed by Section 75 of the
Insurance Code which provides that "[a] policy may declare that a violation of specified
provisions thereof shall avoid it, otherwise the breach of an immaterial provision does not avoid
the policy." Such a condition is a provision which invariably appears in fire insurance policies and
is intended to prevent an increase in the moral hazard. It is commonly known as the additional or
"other insurance" clause and has been upheld as valid and as a warranty that no other insurance
exists. Its violation would thus avoid the policy. However, in order to constitute a violation, the
other insurance must be upon the same subject matter, the same interest therein, and the same
risk.
Same; Same; Separate insurances covering different insurable interests may be
obtained by the mortgagor and the mortgagee.As to a mortgaged property, the
mortgagor and the mortgagee have each an independent insurable interest therein and both
interests may be covered by one policy, or each may take out a separate policy covering his
interest, either at the same or at separate times. The mortgagor's insurable interest covers the
full value of the mortgaged property, even though the mortgage debt is equivalent to the full
value of the property. The mortgagee's insurable interest is to the extent of the debt, since the
property is relied upon as security thereof, and in insuring he is not insuring the property but his
interest or lien thereon. His insurable interest is prima facie the value mortgaged and extends
only to the amount of the debt, not exceeding the value of the mortgaged property. Thus,
separate insurances covering different insurable interests may be obtained by the mortgagor and
the mortgagee.
Same; A policy or insurance contract is to be interpreted liberally in favor of the
insured and strictly against the company.It is a cardinal rule on insurance that a policy or
insurance contract is to be interpreted liberally in favor of the insured and strictly against the

company, the reason being, undoubtedly, to afford the greatest protection which the insured was
endeavoring to secure when he applied for insurance. It is also a cardinal principle of law that
forfeitures are not favored and that any construction which would result in the forfeiture of the
policy benefits for the person claiming thereunder, will be avoided, if it is possible to construe the
policy in a manner which would permit recovery, as, for example, by finding a waiver for such
forfeiture.
Same; Double Insurance; A double insurance exists where the same person is insured
by several insurers separately in respect of the same subject and interest.A double
insurance exists where the same person is insured by several insurers separately in respect of
the same subject and interest. As earlier stated, the insurable interests of a mortgagor and a
mortgagee on the mortgaged property are distinct and separate. Since the two policies of the
PFIC do not cover the same interest as that covered by the policy of the private respondent, no
double insurance exists. The non-disclosure then of the former policies was not fatal to the
petitioner's right to recover on the private respondent's policy.
Same; Same; The rationale behind the incorporation of "other insurance" clause in fire
policies is to prevent over-insurance and thus avert the perpetration of fraud.
Furthermore, by stating within Condition 3 itself that such condition shall not apply if the total
insurance in force at the time of loss does not exceed P200,000.00, the private respondent was
amenable to assume a co-insurer's liability up to a loss not exceeding P200,000.00. What it had
in mind was to discourage overinsurance. Indeed, the rationale behind the incorporation of
"other insurance" clause in fire policies is to prevent over-insurance and thus avert the
perpetration of fraud. When a property owner obtains insurance policies from two or more
insurers in a total amount that exceeds the property's value, the insured may have an
inducement to destroy the property for the purpose of collecting the insurance. The public as
well as the insurer is interested in preventing a situation in which a fire would be profitable to the
insured.
DAVIDE, JR., J.:
FACTS: The petitioner is the owner of Norman's Mart located in the public market of San
Francisco, Agusan del Sur. On 22 December 1989, he obtained from the private respondent fire
insurance policy No. F-146222 for P100,000.00. The period of the policy was from 22 December
1989 to 22 December 1990 and covered the following: "Stock-in-trade consisting principally of
dry goods such as RTW's for men and women wear and other usual to assured's business."
The petitioner declared in the policy under the subheading entitled CO-INSURANCE that
Mercantile Insurance Co., Inc. was the co-insurer for P50,000.00. From 1989 to 1990, the
petitioner had in his inventory stocks amounting to P392,130.50, itemized as follows:
Zenco Sales, Inc.
P 55,698.00
F. Legaspi Gen. Merchandise
86,432.50
Cebu Tesing Textiles
250,000.00 (on credit)
P392,130.50
The policy contained the following condition:
"3. The insured shall give notice to the Company of any insurance or insurances already effected,
or which may subsequently be effected, covering any of the property or properties consisting of
stocks in trade, goods in process and/or inventories only hereby insured, and unless such notice
be given and the particulars of such insurance or insurances be stated therein or endorsed in this
policy pursuant to Section 50 of the Insurance Code, by or on behalf of the Company before the
occurrence of any loss or damage, all benefits under this policy shall be deemed forfeited,

provided however, that this condition shall not apply when the total insurance or insurances in
force at the time of the loss or damage is not more than P200,000.00."
On 27 May 1990, fire of accidental origin broke out at around 7:30 p.m. at the public market of
San Francisco, Agusan del Sur. The petitioner's insured stocks-in-trade were completely
destroyed prompting him to file with the private respondent a claim under the policy. On 28
December 1990, the private respondent denied the claim because it found that at the time of the
loss the petitioner's stocks-in-trade were likewise covered by fire insurance policies No. GA-28146
and No. GA-28144, for P100,000.00 each, issued by the Cebu Branch of the Philippines First
Insurance Co., Inc. (hereinafter PFIC).3 These policies indicate that the insured was "Messrs.
Discount Mart (Mr. Armando Geagonia, Prop.)" with a mortgage clause reading:
"MORTGAGEE: Loss, if any, shall be payable to Messrs. Cebu Tesing Textiles, Cebu City as their
interest may appear subject to the terms of this policy. CO-INSURANCE DECLARED: P100,000.
Phils. First CEB/F-24758"4
The basis of the private respondent's denial was the petitioner's alleged violation of Condition 3
of the policy.
The petitioner then filed a complaint5 against the private respondent with the Insurance
Commission (Case No. 3340) for the recovery of P100,000.00 under fire insurance policy No.
F14622 and for attorney's fees and costs of litigation. He attached as Annex "M"6 thereof his
letter of 18 January 1991 which asked for the reconsideration of the denial. He admitted in the
said letter that at the time he obtained the private respondent's fire insurance policy he knew
that the two policies issued by the PFIC were already in existence; however, he had no
knowledge of the provision in the private respondent's policy requiring him to inform it of the
prior policies; this requirement was not mentioned to him by the private respondent's agent; and
had it been so mentioned, he would not have withheld such information. He further asserted that
the total of the amounts claimed under the three policies was below the actual value of his
stocks at the time of loss, which was P1,000,000.00.
In its answer, the private respondent specifically denied the allegations in the complaint and set
up as its principal defense the violation of Condition 3 of the policy.
G.R. No. 198588.July 11, 2012.
8. UNITED MERCHANTS CORPORATION, petitioner, vs. COUNTRY BANKERS INSURANCE
CORPORATION, respondent.
Insurance Law; In insurance cases, once an insured makes out a prima facie case in its
favor, the burden of evidence shifts to the insurer to controvert the insureds prima
facie case.Burden of proof is the duty of any party to present evidence to establish his claim
or defense by the amount of evidence required by law, which is preponderance of evidence in
civil cases. The party, whether plaintiff or defendant, who asserts the affirmative of the issue has
the burden of proof to obtain a favorable judgment. Particularly, in insurance cases, once an
insured makes out a prima facie case in its favor, the burden of evidence shifts to the insurer to
controvert the insureds prima facie case. In the present case, UMC established a prima facie
case against CBIC. CBIC does not dispute that UMCs stocks in trade were insured against fire
under the Insurance Policy and that the warehouse, where UMCs stocks in trade were stored,
was gutted by fire on 3 July 1996, within the duration of the fire insurance. However, since CBIC
alleged an excepted risk, then the burden of evidence shifted to CBIC to prove such exception.
Same; An insurer who seeks to defeat a claim because of an exception or limitation in
the policy has the burden of establishing that the loss comes within the purview of
the exception or limitation.An insurer who seeks to defeat a claim because of an exception
or limitation in the policy has the burden of establishing that the loss comes within the purview
of the exception or limitation. If loss is proved apparently within a contract of insurance, the
burden is upon the insurer to establish that the loss arose from a cause of loss which is excepted

or for which it is not liable, or from a cause which limits its liability. In the present case, CBIC
failed to discharge its primordial burden of establishing that the damage or loss was caused by
arson, a limitation in the policy.
Insurance Law; The submission of false invoices to the adjusters establishes a clear
case of fraud and misrepresentation which voids the insurers liability as per
condition of the policy.In Yu Ban Chuan v. Fieldmens Insurance, Co., Inc., 14 SCRA 491
(1965), the Court ruled that the submission of false invoices to the adjusters establishes a clear
case of fraud and misrepresentation which voids the insurers liability as per condition of the
policy. Their falsity is the best evidence of the fraudulent character of plaintiffs claim. In
Verendia v. Court of Appeals, 217 SCRA 417 (1993), where the insured presented a fraudulent
lease contract to support his claim for insurance benefits, the Court held that by its false
declaration, the insured forfeited all benefits under the policy provision similar to Condition No.
15 of the Insurance Policy in this case.
Same; It has long been settled that a false and material statement made with an
intent to deceive or defraud voids an insurance policy.It has long been settled that a
false and material statement made with an intent to deceive or defraud voids an insurance
policy. In Yu Cua v. South British Insurance Co., the claim was fourteen times bigger than the real
loss; in Go Lu v. Yorkshire Insurance Co., eight times; and in Tuason v. North China Insurance Co.,
six times. In the present case, the claim is twenty five times the actual claim proved.
Same; While it is a cardinal principle of insurance law that a contract of insurance is
to be construed liberally in favor of the insured and strictly against the insurer
company, contracts of insurance, like other contracts, are to be construed according
to the sense and meaning of the terms which the parties themselves have
used.While it is a cardinal principle of insurance law that a contract of insurance is to be
construed liberally in favor of the insured and strictly against the insurer company, contracts of
insurance, like other contracts, are to be construed according to the sense and meaning of the
terms which the parties themselves have used. If such terms are clear and unambiguous, they
must be taken and understood in their plain, ordinary and popular sense. Courts are not
permitted to make contracts for the parties; the function and duty of the courts is simply to
enforce and carry out the contracts actually made.
CARPIO,J.:
FACTS: Petitioner United Merchants Corporation (UMC) is engaged in the business of buying,
selling, and manufacturing Christmas lights. UMC leased a warehouse at 19-B Dagot Street, San
Jose Subdivision, Barrio Manresa, Quezon City, where UMC assembled and stored its products.
On 6 September 1995, UMCs General Manager Alfredo Tan insured UMCs stocks in trade of
Christmas lights against fire with defendant Country Bankers Insurance Corporation (CBIC) for
P15,000,000.00. The Fire Insurance Policy No. F-HO/95-576 (Insurance Policy) and Fire Invoice No.
12959A, valid until 6 September 1996, states:
AMOUNT OF INSURANCE:
FIFTEEN
MILLION PESOS
PHILIPPINE
CURRENCY
xxx
PROPERTY INSURED: On stocks in trade only, consisting of Christmas Lights, the properties of the
Assured or held by them in trust, on commissions, or on joint account with others and/or for
which they are responsible in the event of loss and/or damage during the currency of this policy,
whilst contained in the building of one lofty storey in height, constructed of concrete and/or
hollow blocks with portion of galvanized iron sheets, under galvanized iron rood, occupied as
Christmas lights storage..

On 7 May 1996, UMC and CBIC executed Endorsement F/96-154 and Fire Invoice No. 16583A to
form part of the Insurance Policy. Endorsement F/96-154 provides that UMCs stocks in trade
were insured against additional perils, to wit: typhoon, flood, ext. cover, and full earthquake.
The sum insured was also increased to P50,000,000.00 effective 7 May 1996 to 10 January 1997.
On 9 May 1996, CBIC issued Endorsement F/96-157 where the name of the assured was changed
from Alfredo Tan to UMC.
On 3 July 1996, a fire gutted the warehouse rented by UMC. CBIC designated CRM Adjustment
Corporation (CRM) to investigate and evaluate UMCs loss by reason of the fire. CBICs reinsurer,
Central Surety, likewise requested the National Bureau of Investigation (NBI) to conduct a parallel
investigation. On 6 July 1996, UMC, through CRM, submitted to CBIC its Sworn Statement of
Formal Claim, with proofs of its loss.
On 20 November 1996, UMC demanded for at least fifty percent (50%) payment of its claim from
CBIC. On 25 February 1997, UMC received CBICs letter, dated 10 January 1997, rejecting UMCs
claim due to breach of Condition No. 15 of the Insurance Policy. Condition No. 15 states:
If the claim be in any respect fraudulent, or if any false declaration be made or used in support
thereof, or if any fraudulent means or devices are used by the Insured or anyone acting in his
behalf to obtain any benefit under this Policy; or if the loss or damage be occasioned by the
willful act, or with the connivance of the Insured, all the benefits under this Policy shall be
forfeited.
On 19 February 1998, UMC filed a Complaint7 against CBIC with the RTC of Manila. UMC
anchored its insurance claim on the Insurance Policy, the Sworn Statement of Formal Claim
earlier submitted, and the Certification dated 24 July 1996 made by Deputy Fire Chief/Senior
Superintendent Bonifacio J. Garcia of the Bureau of Fire Protection. The Certification dated 24 July
1996 provides that:
This is to certify that according to available records of this office, on or about 6:10 P.M. of July 3,
1996, a fire broke out at United Merchants Corporation located at 19-B Dag[o]t Street, Brgy.
Manresa, Quezon City incurring an estimated damage of Fifty-Five Million Pesos (P55,000,000.00)
to the building and contents, while the reported insurance coverage amounted to Fifty Million
Pesos (P50,000,000.00) with Country Bankers Insurance Corporation.
The Bureau further certifies that no evidence was gathered to prove that the establishment was
willfully, feloniously and intentionally set on fire.
That the investigation of the fire incident is already closed being ACCIDENTAL in nature.
PREMIUM (SECTIONS 77-84)
G.R. No. 137172. April 4, 2001.
1. UCPB GENERAL INSURANCE CO., INC., petitioner, vs. MASAGANA TELAMART, INC.,
respondent.
Insurance; Premiums; Exceptions to the rule in Section 77 of the Insurance Code of
1978 that there be prepayment of premiums as a condition to the validity of the
insurance contract.It can be seen at once that Section 77 does not restate the portion of
Section 72 expressly permitting an agreement to extend the period to pay the premium. But are
there exceptions to Section 77? The answer is in the affirmative. The first exception is provided
by Section 77 itself, and that is, in case of a life or industrial life policy whenever the grace
period provision applies. The second is that covered by Section 78 of the Insurance Code, which
provides: SEC. 78. Any acknowledgment in a policy or contract of insurance of the receipt of
premium is conclusive evidence of its payment, so far as to make the policy binding,
notwithstanding any stipulation therein that it shall not be binding until premium is actually paid.
A third exception was laid down in Makati Tuscany Condominium Corporation vs. Court of
Appeals, wherein we ruled that Section 77 may not apply if the parties have agreed to the

payment in installments of the premium and partial payment has been made at the time of loss,
x x x Not only that. In Tuscany, we also quoted with approval the following pronouncement of the
Court of Appeals in its Resolution denying the motion for reconsideration of its decision: x x x By
the approval of the aforequoted findings and conclusion of the Court of Appeals, Tuscany has
provided a fourth exception to Section 77, namely, that the insurer may grant credit extension
for the payment of the premium. This simply means that if the insurer has granted the insured a
credit term for the payment of the premium and loss occurs before the expiration of the term,
recovery on the policy should be allowed even though the premium is paid after the loss but
within the credit term.
Same; Same; There is nothing in Section 77 which prohibits the parties in an
insurance contract to provide a credit term within which to pay the premiums.
Moreover, there is nothing in Section 77 which prohibits the parties in an insurance contract to
provide a credit term within which to pay the premiums. That agreement is not against the law,
morals, good customs, public order or public policy. The agreement binds the parties.
Same; Same; Estoppel; Where an insurer had consistently granted a 60- to 90-day
credit term for the payment of premiums despite its full awareness of Section 77, and
the assured had relied in good faith on such practice, estoppel bars it from taking
refuge under said Section.Finally in the instant case, it would be unjust and inequitable if
recovery on the policy would not be permitted against Petitioner, which had consistently panted
a 60- to 90-day credit term for the payment of premiums despite its full awareness of Section 77.
Estoppel bars it from taking refuge under said Section, since Respondent relied in good faith on
such practice. Estoppel then is the fifth exception to Section 77.
DAVIDE, JR., C.J.:
FACTS: Plaintiff [herein Respondent] obtained from defendant (herein Petitioner] five (5)
insurance policies (Exhibits A to E, Record, pp. 158-175) on its properties [in Pasay City and
Manila]. . . .
All five (5) policies reflect on their face the effectivity term: from 4:00 P.M. of 22 May 1991 to
4:00 P.M. of 22 May 1992. On June 13, 1992, plaintiffs properties located at 2410-2432 and
2442-2450 Taft Avenue, Pasay City were razed by fire. On July 13, 1992, plaintiff tendered, and
defendant accepted, five (5) Equitable Bank Managers Checks in the total amount of
P225,753.45 as renewal premium payments for which Official Receipt Direct Premium No. 62926
(Exhibit Q, Record, p. 191) was is sued by defendant. On July 14, 1992, Masagana made its
formal demand for indemnification for the burned insured properties. On the same day,
defendant returned the five (5) managers checks stating in its letter (Exhibit R/8, Record, p.
192) that it was rejecting Masaganas claim on the following grounds:
a) Said policies expired last May 22, 1992 and were not renewed for another term;
b) Defendant had put plaintiff and its alleged broker on notice of non-renewal earlier; and
c) The properties covered by the said policies were burned in a fire that took place last June 13,
1992, or before tender of premium payment.
Hence Masagana filed this case.
G.R. No. 119655. May 24, 1996.
2. SPS. ANTONIO A. TIBAY and VIOLETA R. TIBAY and OFELIA M. RORALDO, VICTORINA
M. RORALDO, VIRGILIO M. RORALDO, MYRNA M. RORALDO and ROSABELLA M.
RORALDO, petitioners, vs. COURT OF APPEALS and FORTUNE LIFE AND GENERAL
INSURANCE CO., INC., respondents.
Insurance; Words and Phrases; Insurance is a contract whereby one undertakes for a
consideration to indemnify another against loss, damage or liability arising from an

unknown or contingent event.Insurance is a contract whereby one undertakes for a


consideration to indemnify another against loss, damage or liability arising from an unknown or
contingent event. The consideration is the premium, which must be paid at the time and in the
way and manner specified in the policy, and if not so paid, the policy will lapse and be forfeited
by its own terms.
Same; Contracts; Where the parties expressly stipulated that the policy is not in force
until the premium has been fully paid, the payment of partial premium by the assured
should not be considered the payment required by the law and the stipulation of the
partiesrather, it must be taken in the concept of a deposit to be held in trust by the insurer
until such time that the full amount has been tendered and duly receipted for.Precisely, the
insurer and the insured expressly stipulated that (t)his policy including any renewal thereof
and/or any indorsement thereon is not in force until the premium has been fully paid to and duly
receipted by the Company x x x x and that this policy shall be deemed effective, valid and
binding upon the Company only when the premiums therefor have actually been paid in full and
duly acknowledged. Conformably with the aforesaid stipulations explicitly worded and taken in
conjunction with Sec. 77 of the Insurance Code the payment of partial premium by the assured in
this particular instance should not be considered the payment required by the law and the
stipulation of the parties. Rather, it must be taken in the concept of a deposit to be held in trust
by the insurer until such time that the full amount has been tendered and duly receipted for. In
other words, as expressly agreed upon in the contract, full payment must be made before the
risk occurs for the policy to be considered effective and in force.
Same; Same; The rule that contracts of insurance will be construed in favor of the
insured and most strongly against the insurer should not be permitted to have the
effect of making a plain agreement ambiguous and then construe it in favor of the
insured.Indeed, and far more importantly, the cardinal polestar in the construction of an
insurance contract is the intention of the parties as expressed in the policy. Courts have no other
function but to enforce the same. The rule that contracts of insurance will be construed in favor
of the insured and most strongly against the insurer should not be permitted to have the effect of
making a plain agreement ambiguous and then construe it in favor of the insured.
Same; Partial payment of premium even when accepted as a partial payment will not
keep the policy alive even for such fractional part of the year as the part payment
bears to the whole payment.Verily, it is elemental law that the payment of premium is
requisite to keep the policy of insurance in force. If the premium is not paid in the manner
prescribed in the policy as intended by the parties the policy is ineffective. Partial payment even
when accepted as a partial payment will not keep the policy alive even for such fractional part of
the year as the part payment bears to the whole payment.
Same; Same; It should be understood that the integrity of the legal reserve fund that
insurance companies are mandated by law to maintain cannot be secured if by judicial
fiat partial offerings of premiums were to be construed as a legal nexus between the
applicant and the insurer despite an express agreement to the contrary.In the desire
to safeguard the interest of the assured, it must not be ignored that the contract of insurance is
primarily a risk-distributing device, a mechanism by which all members of a group exposed to a
particular risk contribute premiums to an insurer. From these contributory funds are paid
whatever losses occur due to exposure to the peril insured against. Each party therefore takes a
risk: the insurer, that of being compelled upon the happening of the contingency to pay the
entire sum agreed upon, and the insured, that of parting with the amount required as premium,
without receiving anything therefor in case the contingency does not happen. To ensure payment
for these losses, the law mandates all insurance companies to maintain a legal reserve fund in
favor of those claiming under their policies. It should be understood that the integrity of this fund
cannot be secured and maintained if by judicial fiat partial offerings of premiums were to be
construed as a legal nexus between the applicant and the insurer despite an express agreement
to the contrary.

Same; Same; For as long as the current Insurance Code remains unchanged and
partial payment of premiums is not mentioned at all as among the exceptions
provided in Secs. 77 and 78, no policy of insurance can ever pretend to be efficacious
or effective until premium has been fully paid.Interpreting the contract of insurance
stringently against the insurer but liberally in favor of the insured despite clearly defined
obligations of the parties to the policy can be carried out to extremes that there is the danger
that we may, so to speak, kill the goose that lays the golden egg. We are well aware of
insurance companies falling into the despicable habit of collecting premiums promptly yet
resorting to all kinds of excuses to deny or delay payment of just insurance claims. But, in this
case, the law is manifestly on the side of the insurer. For as long as the current Insurance Code
remains unchanged and partial payment of premiums is not mentioned at all as among the
exceptions provided in Secs. 77 and 78, no policy of insurance can ever pretend to be efficacious
or effective until premium has been fully paid.
Same; Premium is the elixir vitae of the insurance business, and all actuarial
calculations and various tabulations of probabilities of losses under the risks insured
against are based on the sound hypothesis of prompt payment of premiums.And so it
must be. For it cannot be disputed that premium is the elixir vitae of the insurance business
because by law the insurer must maintain a legal reserve fund to meet its contingent obligations
to the public, hence, the imperative need for its prompt payment and full satisfaction. It must be
emphasized here that all actuarial calculations and various tabulations of probabilities of losses
under the risks insured against are based on the sound hypothesis of prompt payment of
premiums. Upon this bedrock insurance firms are enabled to offer the assurance of security to
the public at favorable rates.
BELLOSILLO, J.:
FACTS: May a fire insurance policy be valid, binding and enforceable upon mere partial payment
of premium?
On 22 January 1987 private respondent Fortune Life and General Insurance Co., Inc. (FORTUNE)
issued Fire Insurance Policy No. 136171 in favor of Violeta R. Tibay and/or Nicolas Roraldo on
their two-storey residential building located at 5855 Zobel Street, Makati City, together with all
their personal effects therein. The insurance was for P600,000.00 covering the period from 23
January 1987 to 23 January 1988. On 23 January 1987, of the total premium of P2,983.50,
petitioner Violeta Tibay only paid P600.00 thus leaving a considerable balance unpaid.
On 8 March 1987 the insured building was completely destroyed by fire. Two days later or on 10
March 1987 Violeta Tibay paid the balance of the premium. On the same day, she filed with
FORTUNE a claim on the fire insurance policy. Her claim was accordingly referred to its adjuster,
Goodwill Adjustment Services, Inc. (GASI), which immediately wrote Violeta requesting her to
furnish it with the necessary documents for the investigation and processing of her claim.
Petitioner forthwith complied. On 28 March 1987 she signed a non-waiver agreement with GASI
to the effect that any action taken by the companies or their representatives in investigating the
claim made by the claimant for his loss which occurred at 5855 Zobel Roxas, Makati on March 8,
1987, or in the investigating or ascertainment of the amount of actual cash value and loss, shall
not waive or invalidate any condition of the policies of such companies held by said claimant, nor
the rights of either or any of the parties to this agreement, and such action shall not be, or be
claimed to be, an admission of liability on the part of said companies or any of them.
In a letter dated 11 June 1987 FORTUNE denied the claim of Violeta for violation of Policy
Condition No. 2 and of Sec. 77 of the Insurance Code. Efforts to settle the case before the
Insurance Commission proved futile. On 3 March 1988 Violeta and the other petitioners sued
FORTUNE for damages in the amount of P600,000.00 representing the total coverage of the fire
insurance policy plus 12% interest per annum, P100,000.00 moral damages, and attorneys fees
equivalent to 20% of the total claim.

On 19 July 1990 the trial court ruled for petitioners and adjudged FORTUNE liable for the total
value of the insured building and personal properties in the amount of P600,000.00 plus interest
at the legal rate of 6% per annum.
No. L-28501. September 30, 1966
3. PEDRO ARCE, plaintiff-appellee, vs. THE CAPITAL INSURANCE & SURETY CO., INC.,
defendant-appellant.
Mercantile Law; Insurance; Unless premium is paid, insurance contract not effective;
Insurance company relieved of obligation to pay insurance proceeds under the policy
for insured's failure to pay premiums on the policy.It is obvious from both the Insurance
Act, as amended, and the stipulation of the parties that time is of the essence in respect of the
payment of the insurance premium so that if it is not paid the contract does not take effect
unless there is still another stipulation to the contrary. In the instant case, the INSURED was
given a grace period to pay the premium but the period having expired with no payment made,
he cannot insist that the COMPANY is nonetheless obligated to him.
ABAD SANTOS, J.:
In Civil Case No. 66466 of the Court of First Instance of Manila, the Capital Insurance and Surety
Co., Inc., (COMPANY) was ordered to pay Pedro Arce (INSURED) the proceeds of a fire insurance
policy. Not satisfied with the decision, the company appealed to this Court on questions of law.
The INSURED was the owner of a residential house in Tondo, Manila, which had been insured with
the COMPANY since 1961 under Fire Policy No. 24204. On November 27, 1965, the COMPANY sent
to the INSURED Renewal Certificate No. 47302 to cover the period December 5, 1965 to
December 5, 1966. The COMPANY also requested payment of the corresponding premium in the
amount of P38.10.
Anticipating that the premium could not be paid on time, the INSURED, thru his wife, promised to
pay it on January 4, 1966. The COMPANY accepted the promise but the premium was not paid on
January 4, 1966. On January 8, 1966, the house of the INSURED was totally destroyed by fire.
On January 10, 1966, INSURED'S wife presented a claim for indemnity to the COMPANY. She was
told that no indemnity was due because the premium on the policy was not paid. Nonetheless
the COMPANY tendered a check for P300.00 as financial aid which was received by the
INSURED'S daughter, Evelina R. Arce. The voucher for the check which Evelina signed stated that
it was "in full settlement (ex gratia) of the fire loss under Claim No. F-554 Policy No. F-24202."
Thereafter the INSURED and his wife went to the office of the COMPANY to have his signature on
the check identified preparatory to encashment. At that time the COMPANY reiterated that the
check was given "not as an obligation, but as a concession" because the renewal premium had
not been paid. The INSURED cashed the check but then sued the COMPANY on the policy.
The court a quo held that since the COMPANY could have demanded payment of the premium,
mutuality of obligation requires that it should also be liable on its policy. The court a quo also
held that the INSURED was not bound by the signature of Evelina on the check voucher because
he did not authorize her to sign the waiver.
G.R. No. 95546.November 6, 1992.
4. MAKATI TUSCANY CONDOMINIUM CORPORATION, petitioner,vs. THE COURT OF
APPEALS, AMERICAN HOME ASSURANCE CO., represented by American International
Underwriters (Phils.), Inc., respondent.
Insurance Law; Court holds that the subject policies are valid even if the premiums
were paid on installments.We hold that the subject policies are valid even if the premiums
were paid on installments. The records clearly show that petitioner and private respondent
intended subject insurance policies to be binding and effective notwithstanding the staggered

payment of the premiums. The initial insurance contract entered into in 1982 was renewed in
1983, then in 1984. In those three (3) years, the insurer accepted all the installment payments.
Such acceptance of payments speaks loudly of the insurers intention to honor the policies it
issued to petitioner. Certainly, basic principles of equity and fairness would not allow the insurer
to continue collecting and accepting the premiums, although paid on installments, and later deny
liability on the lame excuse that the premiums were not prepaid in full.
Same; Same; Where the risk is entire and the contract is indivisible, the insured is not
entitled to a refund of the premiums paid if the insurer was exposed to the risk
insured for any period however brief or momentary.It appearing from the peculiar
circumstances that the parties actually intended to make the three (3) insurance contracts valid,
effective and binding, petitioner may not be allowed to renege on its obligation to pay the
balance of the premium after the expiration of the whole term of the third policy (No. AH-CPP9210651) in March 1985. Moreover, as correctly observed by the appellate court, where the risk
is entire and the contract is indivisible, the insured is not entitled to a refund of the premiums
paid if the insurer was exposed to the risk insured for any period, however brief or momentary.
BELLOSILLO,J.:
FACTS: This case involves a purely legal question: whether payment by installment of the
premiums due on an insurance policy invalidates the contract of insurance, in view of Sec. 77 of
P.D. 612, otherwise known as the Insurance Code, as amended, which provides:
SECTION77.An insurer is entitled to the payment of the premium as soon as the thing is
exposed to the peril insured against Notwithstanding any agreement to the contrary, no policy or
contract of insurance issued by an insurance company is valid and binding unless and until the
premium thereof has been paid, except in the case of a life or an industrial life policy whenever
the grace period provision applies.
Sometime in early 1982, private respondent American Home Assurance Co. (A H A C),
represented by American International Underwriters (Phils.), Inc., issued in favor of petitioner
Makati Tuscany Condominium Corporation (TUSCANY) Insurance Policy No. AH-CPP-9210452 on
the latters building and premises, for a period beginning 1 March 1982 and ending 1 March
1983, with a total premium of P466,103.05. The premium was paid on installments on 12 March
1982, 20 May 1982, 21 June 1982 and 16 November 1982, all of which were accepted by private
respondent.
On 10 February 1983, private respondent issued to petitioner Insurance Policy No. AH-CPP9210596, which replaced and renewed the previous policy, for a term covering 1 March 1983 to 1
March 1984. The premium in the amount of P466,103.05 was again paid on installments on 13
April 1983, 13 July 1983, 3 August 1983, 9 September 1983, and 21 November 1983. All
payments were likewise accepted by private respondent.
On 20 January 1984, the policy was again renewed and private respondent issued to petitioner
Insurance Policy No. AH-CPP-9210651 for the period 1 March 1984 to 1 March 1985. On this
renewed policy, petitioner made two installment payments, both accepted by private
respondent, the first on 6 February 1984 for P52,000.00 and the second, on 6 June 1984 for
P100,000.00. Thereafter, petitioner refused to pay the balance of the premium.
Consequently, private respondent filed an action to recover the unpaid balance of P314,103.05
for Insurance Policy No. AH-CPP-9210651.
In its answer with counterclaim, petitioner admitted the issuance of Insurance Policy No. AH-CPP9210651. It explained that it discontinued the payment of premiums because the policy did not
contain a credit clause in its favor and the receipts for the installment payments covering the
policy for 1984-85 as well as the two (2) previous policies, stated the following reservations:
2.Acceptance of this payment shall not waive any of the company rights to deny liability on any
claim under the policy arising before such payments or after the expiration of the credit clause of
the policy; and
3.Subject to no loss prior to premium payment. If there be any loss such is not covered.

Petitioner further claimed that the policy was never binding and valid, and no risk attached to the
policy. It then pleaded a counterclaim for P152,000.00 for the premiums already paid for 198485, and in its answer with amended counterclaim, sought the refund of P924,206.10 representing
the premium payments for 1982-85.
After some incidents, petitioner and private respondent moved for summary judgment.
No. L-56718. January 17, 1985.
5. ACME SHOE RUBBER & PLASTIC CORPORATION, petitioner, vs. THE COURT OF
APPEALS and DOMESTIC INSURANCE COMPANY OF THE PHILIPPINES, respondents.
Insurance Law; Insurance policy was automatically cancelled upon failure to pay
premium within 90-day credit extension.Upon the facts, the evidence, and the law, we
sustain the Appellate Court. By the express terms of the Promissory Note signed by its President,
ACME was fully aware that the policy would be automatically cancelled on August 13, 1964, the
90th day from March 14, 1964, if it did not pay the premium before the former date. There is also
evidence to the effect that various reminders by the INSURER for payment remained unheeded
(Exhibit 10). Not having paid the 1964-1965 premium within the extension granted, and
pursuant to R.A. No. 3540, the policy was automatically cancelled and there was no insurance
coverage to speak of as of the date of the fire on October 13, 1964.
Same; R.A. 3540 which took effect on October 1, 1963 cannot be given retroactive effect as to
nullify the action taken by an insurance company in applying the payment made by the insured
on January 8, 1964 to its 1963-64 premium.Since Republic Act No. 3540 was approved only on
June 20, 1963 and was put into effect only beginning October 1, 1963, it could not retroactively
affect the renewal of the in surance policy on May 15, 1963, or prior to the Acts effective date.
ACMES premium payment of January 8, 1964, therefore, was properly applied to the 1963-1964
premium. The Trial Courts opinion that there was a clear agreement to grant ACME credit
extension for 1964-1965 is negated by ACMES Promissory Note binding itself to pay within
ninety days from the effective date of this policy, 15th May, 1964. . . . .the premium and
documentary stamps in the sum of P3,331.26 . . . .. Indubitably, the credit extension granted
ACME was only for 90 days.
Same; Same.If, in the past, ACME had been granted credit extensions, the Promissory Note it
had signed did away with such credit arrangement. Moreover, it was prior to the advent of
Republic Act No. 3540 when renewal receipts that the INSURER had issued did not contain the
Receipt of Payment and Credit Agreement clauses. By 1964, however, the situation had
changed by the passage of said Act by the express provision of which no policy could be valid
and binding unless and until the premium thereof had been paid.
MELENCIO-HERRERA, J.:
FACTS: Since 1946, petitioner ACME Shoe Rubber and Plastic Corporation (ACME, for brevity)
had been insuring yearly against fire its building, machines and general merchandise, located at
Caloocan City, with respondent Domestic Insurance Company of the Philippines (the INSURER, for
short). On May 14, 1962, ACME continued to insure its properties with the INSURER and was
issued Policy No. 24887 in the amount of P200,000.00 for the period May 15, 1962 up to May 15,
1963. On May 14, 1963, the INSURER issued Renewal Receipt No. 22989 to cover the period May
15, 1963 to May 15, 1964 (Exhibit D).
On January 8, 1964, ACME paid P3,331.26 as premium. The INSURER applied the payment as
renewal premium for theperiod May 15, 1963 to May 15, 1964.
On May 15, 1964, the INSURER issued Renewal Receipt No. 30127 (Exhibit E) for the renewal
premium of P3,331.26 for the period May 15, 1964 to May 15, 1965. Stamped on it was the
Note: Subject to Receipt of Payment Clause and Credit Agreement attached hereto and
forming part hereof.

The clauses mentioned, which were attached as riders to Renewal Receipt No. 30127,
respectively read as follows:
RECEIPT OF PAYMENT CLAUSE
IT IS HEREBY DECLARED AND AGREED that notwithstanding anything to the contrary contained
in the within policy, this insurance will be deemed valid and binding upon the Company only
when the premium and documentary stamps therefor have actually been paid in full and duly
acknowledged in an official receipt signed by an authorized official/representative of the
Company (Exhibit E-1)
CREDIT AGREEMENT
The premium corresponding to the first ninety days of the term of this policy or any renewal
thereof is hereby considered paid for the purpose only of making this Policy valid and binding
during said portion of the term. Thereafter, this Policy shall automatically become void and
ineffective (without prejudice to the obligation of the Insured to pay the corresponding short
period premium for the said 90 days) unless prior to the expiration of said period the Insured
shall have actually paid to the Company the total premium and the documentary stamps
stipulated in this Policy. (Exhibit E-2)
On May 26, 1964, ACME, through its President, signed the following
PROMISSORY NOTE
18th May, 1964
Received RR #30127 to be applied on Policy No. 24887 for which I/we promise to pay DOMESTIC
INSURANCE CO. OF THE PHILIPPINES or order, within ninety days from the effective date of this
policy, 15th May. 1964, the premium and documentary stamps in the sum of P3,331.26. Should
I/we fail to pay this promissory note when due, I/we agree that the said policy should stand
automatically cancelled, without further notice by the Company or election on my/our part, and
I/we shall then be liable to pay only the short period premium corresponding to 90 days.
____________________________________
ACME SHOE RUBBER & PLASTIC CORP.
(Signed)
ACMEs properties were completely destroyed by fire on October 13, 1964. ACME filed its
insurance claim but the INSURER disclaimed liability on the ground that as of the date of loss, the
properties burned were not covered by insurance.
On March 20, 1965, ACME sued on the policy before the Court of First Instance of Rizal Branch
XII, Caloocan City, for the collection of the insurance proceeds and for damages in the form of
lost profits by reason of the delay in payment.
[No. L-2910. June 29, 1951.]
6. THE MANUFACTURERS LIFE INSURANCE Co., plaintiff and appellant, vs. BIBIANO L.
MEER, in the capacity as Collector of Internal Revenue, defendant and appellee.
1.LIFE INSURANCE; CASH SURRENDER VALUE.Cash surrender value "as applied to a life
insurance policy, is the amount of money the company agrees to pay to the holder of the policy if
he surrenders it and releases his claims upon it. The more premiums the insured has paid the
greater will be the surrender value; but the surrender value is always a lesser sum than the total
amount of premiums paid." The cash value or cash surrender value is therefore an amount which
the insurance company holds in trust for the insured to be delivered to him upon demand, and is
a liability of the company to the insured.
2.ID.; PREMIUM PAID ON AUTOMATIC LOAN HELD SUBJECT TO TAX.As the insurer agreed
to consider the premium paid on the strength of the automatic loan, which is taken out of the

cash surrender value, the premium is therefore paid by means of a note or credit or other
substitute for money, and tax is due thereon under section 255 of the National Internal Revenue
Code as amended.
3.ID.; TAXATION; COMPANIES ENGAGED IN BUSINESS IN THE PHILIPPINES, LlABLE TO
TAX; PAYMENT OF PREMIUMS IN A FOREIGN COUNTRY, DOES NOT EXEMPT.The
issuance company claims that as the advances of premiums were made in Toronto, such
premiums are deemed to have been paid therenot in the Philippinesand therefore those
payments are not subject to local taxation. Held: The loans are made to policy-holders in the
Philippines, who in turn pay therewith the premiums to the insurer through the Manila Branch,
and are therefore taxable locally.
4.INSURANCE COMPANY; CLOSING OF BRANCH OFFICE IN THE PHILIPPINES DURING THE WAR, DlD
NOT TERMINATE OPERATIONS OF THE COMPANY.Although the insurer was not open for new
business because its Manila office was closed, yet if it was collecting premiums on its
outstanding policies, incurring the risks and/or enjoying the benefits consequent thereto, it was
operating in this country.
BENGZON, J.:
FACTS: The plaintiff, the Manufacturers Life Insurance Company in a corporation duly organized
in Canada with head office at Toronto. It is duly registered and licensed to engage in life
insurance business in the Philippines, and maintains a branch office in Manila. It was engaged in
such business in the Philippines for more than five years before and including the year 1941, But
due to the exigencies of the war it closed the branch office at Manila during 1942 up to
September 1945.
In the course of its operations before the war, plaintiff issued a number of life insurance policies
in the Philippines containing stipulations referred to as nonforfeiture clauses, as follows:
" '8. Automatic Premium Loan.This Policy shall not lapse for non-payment of any premium after
it has been three full years in force, if, at the due date of such premium, the Cash Value of this
Policy and of any bonus additions and dividends left on accumulation (after deducting any
indebtedness to the Company and the interest accrued thereon) shall exceed the amount of said
premium. In which event the company will, without further request, treat the premium then due
as paid, and the amount of such premium, with interest from its actual due date at six per cent
per annum, compounded yearly, and one per cent, compounded yearly, for expenses, shall be a
first lien on this Policy in the Company's favour in priority to the claim of any assignee or any
other person. The accumulated lien may at any time, while the Policy is in force, be paid in whole
or in part.
'When the premium falls due and is not paid in cash within the month's grace, if the Cash Value
of this policy and of any bonus additions and dividends left on accumulation (after deducting any
accumulated indebtedness) be less than the premium then due, the Company will, without
further requests, continue this insurance in force for a period * * *.
'10. Cash and Paid-Up Insurance Values.At the end of the third policy year or thereafter, upon
the legal surrender of this Policy to the Company while there is no default in premium payments
or within two months after the due date of the premium in default, the Company will (1) grant a
cash value as specified in Column (A) increased by the cash value of any bonus additions and
dividends left on accumulation, which have been alloted to this Policy, less all indebtedness to
the Company on this Policy on the date of such surrender, or (2) endorse this Policy as a NonParticipating Paid-up Policy for the amount as specified in Column (B) of the Table of Guaranteed
Values * * *.
'11. Extended Insurance.After the premiums for three or more full years have been paid
hereunder in cash, if any subsequent premium is not paid when due, and there is no
indebtedness to the Company, on the written request of the Insured * * *."

From January 1, 1942 to December 31, 1946 for failure of the insured under the above policies to
pay the corresponding premiums for one or more years, the plaintiff's head office at Toronto,
applied the provisions of the automatic premium loan clauses; and the net amount of premiums
so advanced or loaned totalled P1,069,254.98. On this sum the defendant Collector of Internal
Revenue assessed P17,917.12which plaintiff paid supra protest. The assessment was made
pursuant to section 255 of the National Internal Revenue Code as amended, which partly
provides:
"SEC. 255. Taxes on insurance premiums.There shall be collected from every person, company,
or corporation (except purely cooperative companies or associations) doing insurance business
of any sort in the Philippines a tax of one per centum of the total premiums collected * * *
whether such premiums are paid in money, notes, credits, or any substitute for money but
premiums refunded within six months after payment on account of rejection of risk or returned
for other reason to person insured shall not be included in the taxable receipts * * *."
It is the plaintiff's contention that when it made premium loans or premium advances, as above
stated, by virtue of the non-forfeiture clauses, it did not collect premiums within the meaning of
the above sections of the law, and therefore it is not amenable to the tax therein provided.
G.R. No. 165585.November 20, 2013.
7. GOVERNMENT SERVICE INSURANCE SYSTEM, petitioner, vs. PRUDENTIAL
GUARANTEE AND ASSURANCE, INC., DEVELOPMENT BANK OF THE PHILIPPINES, and
LAND BANK OF THE PHILIPPINES, respondents.
Same; Same; Same; Government Service Insurance System (GSIS); The funds and
assets of Government Service Insurance System (GSIS) may after the resolution of the
appeal and barring any provisional injunction thereto be subject to execution, attachment,
garnishment or levy since the exemption under Section 39 of RA 8291 does not operate to deny
private entities from properly enforcing their contractual claims against GSIS.While an
execution pending appeal should not lie in view of the above-discussed reasons, it must be noted
that the funds and assets of GSIS may after the resolution of the appeal and barring any
provisional injunction thereto be subject to execution, attachment, garnishment or levy since
the exemption under Section 39 of RA 8291 does not operate to deny private entities from
properly enforcing their contractual claims against GSIS. This has been established in the case of
Rubia wherein the Court held as follows: [T]he declared policy of the State in Section 39 of the
GSIS Charter granting GSIS an exemption from tax, lien, attachment, levy, execution, and other
legal processes should be read together with the grant of power to the GSIS to invest its excess
funds under Section 36 of the same Act. Under Section 36, the GSIS is granted the ancillary
power to invest in business and other ventures for the benefit of the employees, by using its
excess funds for investment purposes. In the exercise of such function and power, the GSIS is
allowed to assume a character similar to a private corporation. Thus, it may sue and be sued, as
also explicitly granted by its charter. Needless to say, where proper, under Section 36, the GSIS
may be held liable for the contracts it has entered into in the course of its business investments.
For GSIS cannot claim a special immunity from liability in regard to its business ventures under
said Section. Nor can it deny contracting parties, in our view, the right of redress and the
enforcement of a claim, particularly as it arises from a purely contractual relationship of a private
character between an individual and the GSIS.
PERLAS-BERNABE,J.:
FACTS: Assailed in these consolidated petitions for review on certiorari1 are separate issuances
of the Court of Appeals (CA) in relation to the complaint for sum of money filed by Prudential
Guarantee and Assurance, Inc. (PGAI) against the Government Service Insurance System (GSIS)
before the Regional Trial Court of Makati City, Branch 149 (RTC), docketed as Civil Case No. 011634.

In particular, the petition in G.R. No. 165585 assails the Decision2 dated May 26, 2004 and
Resolution3 dated October 6, 2004 of the CA in CA-G.R. SP No. 69289 which affirmed the Order4
dated February 14, 2002, as well as the Order,5 Notices of Garnishment,6 and Writ of Execution,7
all dated February 19, 2002, issued by the RTC authorizing execution pending appeal.
On the other hand, the petition in G.R. No. 176982 assails the Decision8 dated October 30, 2006
and Resolution9 dated March 12, 2007 of the CA in CA-G.R. CV No. 73965 which dismissed the
appeal filed by GSIS, affirming with modification the Order10 dated January 11, 2002 of the RTC
rendering judgment on the pleadings.
Sometime in March 1999, the National Electrification Administration (NEA) entered into a
Memorandum of Agreement11 (MOA) with GSIS insuring all real and personal properties
mortgaged to it by electrical cooperatives under an Industrial All Risks Policy (IAR policy).12 The
total sum insured under the IAR policy was P16,731,141,166.80, out of which, 95% or
P15,894,584,108.40 was reinsured by GSIS with PGAI for a period of one year or from March 5,
1999 to March 5, 2000.13 As reflected in Reinsurance Request Note No. 99-15014 (reinsurance
cover) and the Reinsurance Binder15 dated April 21, 1999 (reinsurance binder), GSIS agreed to
pay PGAI reinsurance premiums in the amount of P32,885,894.52 per quarter or a total of
P131,543,578.08.16 While GSIS remitted to PGAI the reinsurance premiums for the first three
quarters, it, however, failed to pay the fourth and last reinsurance premium due on December 5,
1999 despite demands.
This prompted PGAI to file, on November 15, 2001, a Complaint17 for sum of money (complaint)
against GSIS before the RTC, docketed as Civil Case No. 01-1634.
In its complaint, PGAI alleged, among others, that: (a) after it had issued the IAR policy, it further
reinsured the risks covered under the said reinsurance with reputable reinsurers worldwide such
as Lloyds of London, Copenhagen Re, Cigna Singapore, CCR, Generali, and Arig;18 (b) the first
three reinsurance premiums were paid to PGAI by GSIS and, in the same vein, NEA paid the first
three reinsurance premiums due to GSIS;19 (c) GSIS failed to pay PGAI the fourth and last
reinsurance premium due on December 5, 1999;20 (d) the IAR policy remained in full force and
effect for the entire insurable period and, in fact, the losses/damages on various risks reinsured
by PGAI were paid and accordingly settled by it;21 (e) PGAI is under continuous pressure from its
reinsurers in the international market to settle the matter;22 and (f) GSIS acknowledged its
obligation to pay the last reinsurance premium as it, in turn, demanded from NEA the fourth and
last reinsurance premium.
In its Answer,24 GSIS admitted, among others, that: (a) its request for reinsurance cover was
accepted by PGAI in a reinsurance binder;25 (b) it remitted to PGAI the first three reinsurance
premiums which were paid by NEA;26 and (c) it failed to remit the fourth and last reinsurance
premium to PGAI.
This prompted PGAI to file, on November 15, 2001, a Complaint17 for sum of money (complaint)
against GSIS before the RTC, docketed as Civil Case No. 01-1634.
In its complaint, PGAI alleged, among others, that: (a) after it had issued the IAR policy, it further
reinsured the risks covered under the said reinsurance with reputable reinsurers worldwide such
as Lloyds of London, Copenhagen Re, Cigna Singapore, CCR, Generali, and Arig;18 (b) the first
three reinsurance premiums were paid to PGAI by GSIS and, in the same vein, NEA paid the first
three reinsurance premiums due to GSIS;19 (c) GSIS failed to pay PGAI the fourth and last
reinsurance premium due on December 5, 1999;20 (d) the IAR policy remained in full force and
effect for the entire insurable period and, in fact, the losses/damages on various risks reinsured
by PGAI were paid and accordingly settled by it;21 (e) PGAI is under continuous pressure from its
reinsurers in the international market to settle the matter;22 and (f) GSIS acknowledged its
obligation to pay the last reinsurance premium as it, in turn, demanded from NEA the fourth and
last reinsurance premium.

In its Answer,24 GSIS admitted, among others, that: (a) its request for reinsurance cover was
accepted by PGAI in a reinsurance binder;25 (b) it remitted to PGAI the first three reinsurance
premiums which were paid by NEA;26 and (c) it failed to remit the fourth and last reinsurance
premium to PGAI (Rules). GSIS opposed36 the foregoing motion by reiterating the allegations and
defenses in its Answer.
On January 11, 2002, the RTC issued an Order37 (January 11, 2002 Order) granting PGAIs Motion
for Judgment on the Pleadings. It observed that the admissions of GSIS that it paid the first three
quarterly reinsurance premiums to PGAI affirmed the validity of the contract of reinsurance
between them. As such, GSIS cannot now renege on its obligation to remit the last and remaining
quarterly reinsurance premium.38 It further pointed out that while it is true that the payment of
the premium is a requisite for the validity of an insurance contract as provided under Section 77
of Presidential Decree No. (PD) 612,39 otherwise known as The Insurance Code, it was held in
Makati Tuscany Condominium Corp. v. CA40 (Makati Tuscany) that insurance policies are valid
even if the premiums were paid in installments, as in this case.41 Thus, in view of the foregoing,
the RTC ordered GSIS to pay PGAI the last quarter reinsurance premium in the sum of
P32,885,894.52, including interests amounting to P6,519,515.91 as of July 31, 2000 until full
payment, attorneys fees, and costs of suit.42 Dissatisfied, GSIS filed a notice of appeal.
LOSS AND NOTICE OF LOSS (SECTIONS 85-94)
G.R. No. 136914. January 25, 2002.
1. COUNTRY BANKERS INSURANCE CORPORATION, petitioner, vs. LIANGA BAY AND
COMMUNITY MULTI-PURPOSE COOPERATIVE, INC., respondent.
Same; Insurance; If a proof is made of a loss apparently within a contract of
insurance, the burden is upon the insurer to prove that the loss arose from a cause of
loss which is excepted or for which it is not liable, or from a cause which limits its
liability.Where a risk is excepted by the terms of a policy which insures against other perils or
hazards, loss from such a risk constitutes a defense which the insurer may urge, since it has not
assumed that risk, and from this it follows that an insurer seeking to defeat a claim because of
an exception or limitation in the policy has the burden of proving that the loss comes within the
purview of the exception or limitation set up. If a proof is made of a loss apparently within a
contract of insurance, the burden is upon the insurer to prove that the loss arose from a cause of
loss which is excepted or for which it is not liable, or from a cause which limits its liability. Stated
elsewise, since the petitioner in this case is defending on the ground of non-coverage and relying
upon an exemption or exception clause in the fire insurance policy, it has the burden of proving
the facts upon which such excepted risk is based, by a preponderance of evidence. But petitioner
failed to do so.
Insurance; Damages; Interest Rates; The insurance claim in this case is evidently not
a forbearance of money, goods or credit, and thus the interest rate should be fixed at
six percent (6%) computed from the date of filing of the complaint.In the said case of
Eastern Shipping, the Court further observed that a forbearance in the context of the usury law
is a contractual obligation of lender or creditor to refrain, during a given period of time, from
requiring the borrower or debtor to repay a loan or debt then due and payable. Considering the
foregoing, the insurance claim in this case is evidently not a forbearance of money, goods or
credit, and thus the interest rate should be as it is hereby fixed at six percent (6%) computed
from the date of filing of the complaint.
Same; Same; Exemplary damages are imposed not to enrich one party or impoverish
another but to serve as a deterrent against or as a negative incentive to curb socially
deleterious actions.Concerning the award of exemplary damages for Fifty Thousand Pesos
(P50,000.00), we likewise find no legal and valid basis for granting the same. Article 2229 of the

New Civil Code provides that exemplary damages may be imposed by way of example or
correction for the public good. Exemplary damages are imposed not to enrich one party or
impoverish another but to serve as a deterrent against or as a negative incentive to curb socially
deleterious actions. They are designed to permit the courts to mould behavior that has socially
deleterious consequences, and its imposition is required by public policy to suppress the wanton
acts of an offender. However, it cannot be recovered as a matter of right. It is based entirely on
the discretion of the court. We find no cogent and valid reason to award the same in the case at
bar.
DE LEON, JR., J.:
FACTS: The petitioner is a domestic corporation principally engaged in the insurance business
wherein it undertakes, for a consideration, to indemnify another against loss, damage or liability
from an unknown or contingent event including fire while the respondent is a duly registered
cooperative judicially declared insolvent and represented by the elected assignee, Cornelio
Jamero.
It appears that sometime in 1989, the petitioner and the respondent entered into a contract of
fire insurance. Under Fire Insurance Policy No. F-1397, the petitioner insured the respondents
stocks-in-trade against fire loss, damage or liability during the period starting from June 20, 1989
at 4:00 p.m. to June 20, 1990 at 4:00 p.m., for the sum of Two Hundred Thousand Pesos
(P200,000.00).
On July 1, 1989, at or about 12:40 a.m., the respondents building located at Barangay Diatagon,
Lianga, Surigao del Sur was gutted by fire and reduced to ashes, resulting in the total loss of the
respondents stocks-in-trade, pieces of furniture and fixtures, equipments and records.
Due to the loss, the respondent filed an insurance claim with the petitioner under its Fire
Insurance Policy No. F-1397, submitting: (a) the Spot Report of Pfc. Arturo V. Juarbal, INP
Investigator, dated July 1, 1989; (b) the Sworn Statement of Jose Lomocso; and (c) the Sworn
Statement of Ernesto Urbiztondo.
The petitioner, however, denied the insurance claim on the ground that, based on the submitted
documents, the building was set on fire by two (2) NPA rebels who wanted to obtain canned
goods, rice and medicines as provisions for their comrades in the forest, and that such loss was
an excepted risk under paragraph No. 6 of the policy conditions of Fire Insurance Policy No. F1397, which provides:
This insurance does not cover any loss or damage occasioned by or through or in consequence,
directly or indirectly, of any of the following occurrences, namely:
xxx
xxx
xxx
(d) Mutiny, riot, military or popular uprising, insurrection, rebellion, revolution, military or
usurped power.
Any loss or damage happening during the existence of abnormal conditions (whether physical or
otherwise) which are occasioned by or through or in consequence, directly or indirectly, of any of
said occurrences shall be deemed to be loss or damage which is not covered by this insurance,
except to the extent that the Insured shall prove that such loss or damage happened
independently of the existence of such abnormal conditions.
Finding the denial of its claim unacceptable, the respondent then instituted in the trial court the
complaint for recovery of loss, damage or liability against petitioner. The petitioner answered
the complaint and reiterated the ground it earlier cited to deny the insurance claim, that is, that
the loss was due to NPA rebels, an excepted risk under the fire insurance policy.
G.R. No. 137775. March 31, 2005.
FGU INSURANCE CORPORATION, petitioner, vs. THE COURT OF APPEALS, SAN MIGUEL
CORPORATION, and ESTATE OF ANG GUI, represented by LUCIO, JULIAN, and JAIME, all
surnamed ANG, and CO TO, respondents.

Civil Law; Obligations; Force Majeure; By definition, caso fortuito or force majeure are
extraordinary events not foreseeable or avoidable, events that could not be foreseen,
or which though foreseen, were inevitable; It is not enough that the event should not
have been foreseen or anticipated, as is commonly believed but it must be one
impossible to foresee or to avoid.Caso fortuito or force majeure (which in law are identical
insofar as they exempt an obligor from liability) by definition, are extraordinary events not
foreseeable or avoidable, events that could not be foreseen, or which though foreseen, were
inevitable. It is therefore not enough that the event should not have been foreseen or
anticipated, as is commonly believed but it must be one impossible to foresee or to avoid.
Same; Same; Same; To be exempted from responsibility, the natural disaster should have been
the proximate and only cause of the loss.While the loss of the cargoes was admittedly caused
by the typhoon Sisang, a natural disaster, ANCO could not escape liability to respondent SMC.
The records clearly show the failure of petitioners representatives to exercise the extraordinary
degree of diligence mandated by law. To be exempted from responsibility, the natural disaster
should have been the proximate and only cause of the loss. There must have been no
contributory negligence on the part of the common carrier.
Same; Same; Same; Insurance Law; It is a basic rule in insurance that the
carelessness and negligence of the insured or his agents constitute no defense on the
part of the insurer; The rule presupposes that the loss occurred due to the causes
which could not have been prevented by the insured despite the exercise of due
diligence.One of the purposes for taking out insurance is to protect the insured against the
consequences of his own negligence and that of his agents. Thus, it is a basic rule in insurance
that the carelessness and negligence of the insured or his agents constitute no defense on the
part of the insurer. This rule however presupposes that the loss has occurred due to causes which
could not have been prevented by the insured, despite the exercise of due diligence.
Same; Same; Same; Same; When the evidence show that the insureds negligence or
recklessness is so gross as to be sufficient to constitute a willful act, the insurer must
be exonerated.The question now is whether there is a certain degree of negligence on the
part of the insured or his agents that will deprive him the right to recover under the insurance
contract. We say there is. However, to what extent such negligence must go in order to
exonerate the insurer from liability must be evaluated in light of the circumstances surrounding
each case. When evidence show that the insureds negligence or recklessness is so gross as to
be sufficient to constitute a willful act, the insurer must be exonerated.
CHICO-NAZARIO, J.:
FACTS: Evidence shows that Anco Enterprises Company (ANCO), a partnership between Ang Gui
and Co To, was engaged in the shipping business. It owned the M/T ANCO tugboat and the D/B
Lucio barge which were operated as common carriers. Since the D/B Lucio had no engine of its
own, it could not maneuver by itself and had to be towed by a tugboat for it to move from one
place to another.
On 23 September 1979, San Miguel Corporation (SMC) shipped from Mandaue City, Cebu, on
board the D/B Lucio, for towage by M/T ANCO, the following cargoes:
Bill of Lading No.
Shipment
Destination
1
25,000 cases Pale Pilsen
Estancia, Iloilo
350 cases Cerveza Negra
Estancia, Iloilo
2
15,000 cases Pale Pilsen

San Jose, Antique


200 cases Cerveza Negra
San Jose, Antique
The consignee for the cargoes covered by Bill of Lading No. 1 was SMCs Beer Marketing Division
(BMD)-Estancia Beer Sales Office, Estancia, Iloilo, while the consignee for the car goes covered
by Bill of Lading No. 2 was SMCs BMD-San Jose Beer Sales Office, San Jose, Antique.
The D/B Lucio was towed by the M/T ANCO all the way from Mandaue City to San Jose, Antique.
The vessels arrived at San Jose, Antique, at about one oclock in the afternoon of 30 September
1979. The tugboat M/T ANCO left the barge immediately after reaching San Jose, Antique.
When the barge and tugboat arrived at San Jose, Antique, in the afternoon of 30 September
1979, the clouds over the area were dark and the waves were already big. The arrastre workers
unloading the cargoes of SMC on board the D/B Lucio began to complain about their difficulty in
unloading the cargoes. SMCs District Sales Supervisor, Fernando Macabuag, requested ANCOs
representative to transfer the barge to a safer place because the vessel might not be able to
withstand the big waves.
ANCOs representative did not heed the request because he was confident that the barge could
withstand the waves. This, notwithstanding the fact that at that time, only the M/T ANCO was left
at the wharf of San Jose, Antique, as all other vessels already left the wharf to seek shelter. With
the waves growing bigger and bigger, only Ten Thousand Seven Hundred Ninety (10,790) cases
of beer were discharged into the custody of the arrastre operator.
At about ten to eleven oclock in the evening of 01 October 1979, the crew of D/B Lucio
abandoned the vessel because the barges rope attached to the wharf was cut off by the big
waves. At around midnight, the barge run aground and was broken and the cargoes of beer in
the barge were swept away.
As a result, ANCO failed to deliver to SMCs consignee Twenty-Nine Thousand Two Hundred Ten
(29,210) cases of Pale Pilsen and Five Hundred Fifty (550) cases of Cerveza Negra. The value per
case of Pale Pilsen was Forty-Five Pesos and Twenty Centavos (P45.20). The value of a case of
Cerveza Negra was Forty-Seven Pesos and Ten Centavos (P47.10), hence, SMCs claim against
ANCO amounted to One Million Three Hundred Forty-Six Thousand One Hundred Ninety-Seven
Pesos (P1,346,197.00).
As a consequence of the incident, SMC filed a complaint for Breach of Contract of Carriage and
Damages against ANCO for the amount of One Million Three Hundred Forty-Six Thousand One
Hundred Ninety-Seven Pesos (P1,346,197.00) plus interest, litigation expenses and Twenty-Five
Percent (25%) of the total claim as attorneys fees.
Upon Ang Guis death, ANCO, as a partnership, was dissolved hence, on 26 January 1993, SMC
filed a second amended complaint which was admitted by the Court impleading the surviving
partner, Co To and the Estate of Ang Gui represented by Lucio, Julian and Jaime, all surnamed
Ang. The substituted defendants adopted the original answer with counterclaim of ANCO since
the substantial allegations of the original complaint and the amended complaint are practically
the same.
ANCO admitted that the cases of beer Pale Pilsen and Cerveza Negra mentioned in the complaint
were indeed loaded on the vessel belonging to ANCO. It claimed however that it had an
agreement with SMC that ANCO would not be liable for any losses or damages resulting to the
cargoes by reason of fortuitous event. Since the cases of beer Pale Pilsen and Cerveza Negra
were lost by reason of a storm, a fortuitous event which battered and sunk the vessel in which
they were loaded, they should not be held liable. ANCO further asserted that there was an

agreement between them and SMC to insure the cargoes in order to recover indemnity in case of
loss. Pursuant to that agreement, the cargoes to the extent of Twenty Thousand (20,000) cases
was insured with FGU Insurance Corporation (FGU) for the total amount of Eight Hundred FiftyEight Thousand Five Hundred Pesos (P858,500.00) per Marine Insurance Policy No. 29591.
Subsequently, ANCO, with leave of court, filed a Third-Party Complaint against FGU, alleging that
before the vessel of ANCO left for San Jose, Antique with the cargoes owned by SMC, the cargoes,
to the extent of Twenty Thousand (20,000) cases, were insured with FGU for a total amount of
Eight Hundred Fifty-Eight Thousand Five Hundred Pesos (P858,500.00) under Marine Insurance
Policy No. 29591. ANCO further alleged that on or about 02 October 1979, by reason of very
strong winds and heavy waves brought about by a passing typhoon, the vessel run aground near
the vicinity of San Jose, Antique, as a result of which, the vessel was totally wrecked and its
cargoes owned by SMC were lost and/or destroyed. According to ANCO, the loss of said cargoes
occurred as a result of risks insured against in the insurance policy and during the existence and
lifetime of said insurance policy. ANCO went on to assert that in the remote possibility that the
court will order ANCO to pay SMCs claim, the third-party defendant corporation should be held
liable to indemnify or reimburse ANCO whatever amounts, or damages, it may be required to pay
to SMC.
G.R. No. 92383. July 17, 1992.*
4. SUN INSURANCE OFFICE, LTD., petitioner, vs. THE HON. COURT OF APPEALS and
NERISSA LIM, respondents.
Insurance Law; Definition of accident.An accident is an event which happens without any
human agency or, if happening through human agency, an event which, under the
circumstances, is unusual to and not expected by the person to whom it happens. It has also
been defined as an injury which happens by reason of some violence or casualty to the insured
without his design, consent, or voluntary cooperation.
Same; Same; Court is convinced that the incident that resulted in Lims death was
indeed an accident.In light of these definitions, the Court is convinced that the incident that
resulted in Lims death was indeed an accident. The petitioner, invoking the case of De la Cruz v.
Capital Insurance, says that there is no accident when a deliberate act is performed unless
some additional, unexpected, independent and unforeseen happening occurs which produces or
brings about their injury or death. There was such a happening. This was the firing of the gun,
which was the additional unexpected and independent and unforeseen occurrence that led to the
insured persons death.
Same; Same; Suicide and willful exposure to needless peril are in pari materia
because they both signify a disregard for ones life.It should be noted at the outset that
suicide and willful exposure to needless peril are in pari materia because they both signify a
disregard for ones life. The only difference is in degree, as suicide imports a positive act of
ending such life whereas the second act indicates a reckless risking of it that is almost suicidal in
intent.
Same; Contract; There is nothing in the policy that relieves the insurer of the
responsibility to pay the indemnity agreed upon if the insured is shown to have
contributed to his own accident.Lim was unquestionably negligent and that negligence
cost him his own life. But it should not prevent his widow from recovering from the insurance
policy he obtained precisely against accident. There is nothing in the policy that relieves the
insurer of the responsibility to pay the indemnity agreed upon if the insured is shown to have
contributed to his own accident. Indeed, most accidents are caused by negligence. There are
only four exceptions expressly made in the contract to relieve the insurer from liability, and none
of these exceptions is applicable in the case at bar.

Same; Same; As a rule, insurance contracts are supposed to be interpreted liberally in


favor of the assured.It bears noting that insurance contracts are as a rule supposed to be
interpreted liberally in favor of the assured. There is no reason to deviate from this rule,
especially in view of the circumstances of this case as above analyzed.
Civil Law; Damages; The Supreme Court holds that the award of moral and exemplary
damages and of attorneys fees is unjust and so must be disapproved.On the second
assigned error, however, the Court must rule in favor of the petitioner. The basic issue raised in
this case is, as the petitioner correctly observed, one of first impression. It is evident that the
petitioner was acting in good faith when it resisted the private respondents claim on the ground
that the death of the insured was covered by the exception. The issue was indeed debatable and
was clearly not raised only for the purpose of evading a legitimate obligation. We hold therefore
that the award of moral and exemplary damages and of attorneys fees is unjust and so must be
disapproved.
CRUZ, J.:
FACTS: The petitioner issued Personal Accident Policy No. 05687 to Felix Lim, Jr. with a face
value of P200,000.00. Two months later, he was dead with a bullet wound in his head. As
beneficiary, his wife Nerissa Lim sought payment on the policy but her claim was rejected. The
petitioner agreed that there was no suicide. It argued, however, that there was no accident
either.
Pilar Nalagon, Lims secretary, was the only eyewitness to his death. It happened on October 6,
1982, at about 10 oclock in the evening, after his mothers birthday party. According to Nalagon,
Lim was in a happy mood (but not drunk) and was playing with his handgun, from which he had
previously removed the magazine. As she watched television, he stood in front of her and
pointed the gun at her. She pushed it aside and said it might be loaded. He assured her it was
not and then pointed it to his temple. The next moment there was an explosion and Lim slumped
to the floor. He was dead before he fell.
The widow sued the petitioner in the Regional Trial Court of Zamboanga City and was sustained.2
The petitioner was sentenced to pay her P200,000.00, representing the face value of the policy,
with interest at the legal rate; P10,000.00 as moral damages; P5,000.00 as exemplary damages;
P5,000.00 as actual and compensatory damages; and P5,000.00 as attorneys fees, plus the
costs of the suit. This decision was affirmed on appeal, and the motion for reconsideration was
denied.3 The petitioner then came to this Court to fault the Court of Appeals for approving the
payment of the claim and the award of damages.
The term accident has been defined as follows:
The words accident and accidental have never acquired any technical signification in law,
and when used in an insurance contract are to be construed and considered according to the
ordinary understanding and common usage and speech of people generally. In substance, the
courts are practically agreed that the words accident and accidental mean that which
happens by chance or fortuitously, without intention or design, and which is unexpected,
unusual, and unforeseen. The definition that has usually been adopted by the courts is that an
accident is an event that takes place without ones foresight or expectationan event that
proceeds from an unknown cause, or is an unusual effect of a known case, and therefore not
expected.
An accident is an event which happens without any human agency or, if happening through
human agency, an event which, under the circumstances, is unusual to and not expected by the
person to whom it happens. It has also been defined as an injury which happens by reason of
some violence or casualty to the insured without his design, consent, or voluntary co-operation.
In light of these definitions, the Court is convinced that the incident that resulted in Lims death
was indeed an accident. The petitioner, invoking the case of De la Cruz v. Capital Insurance,6

says that there is no accident when a deliberate act is performed unless some additional,
unexpected, independent and unforeseen happening occurs which produces or brings about their
injury or death. There was such a happening. This was the firing of the gun, which was the
additional unexpected and independent and unforeseen occurrence that led to the insured
persons death.
The petitioner also cites one of the four exceptions provided for in the insurance contract and
contends that the private petitioners claim is barred by such provision. It is there stated:
Exceptions
The company shall not be liable in respect of
1. Bodily injury
x
x
x
b. consequent upon
i) The insured person attempting to commit suicide or willfully exposing himself to needless
peril except in an attempt to save human life.
To repeat, the parties agree that Lim did not commit suicide. Nevertheless, the petitioner
contends that the insured willfully exposed himself to needless peril and thus removed himself
from the coverage of the insurance policy.
5. No. L-38613. February 25, 1982.
PACIFIC TIMBER EXPORT CORPORATION, petitioner, vs. THE HONORABLE COURT OF
APPEALS and WORKMENS INSURANCE COMPANY, INC., respondents.
-Refer to case no. 6 under Policy
GR NO. L-67835, OCTOBER 12, 1987
6. MALAYAN INSURANCE CO V. ARNALDO
G.R. No. 82036. May 22, 1997.
7. TRAVELLERS INSURANCE & SURETY CORPORATION, petitioner, vs. HON. COURT OF
APPEALS and VICENTE MENDOZA, respondents.
Insurance; The prescriptive period to bring suit in court under an insurance policy,
begins to run from the date of the insurers rejection of the claim filed by the insured,
the beneficiary or any person claiming under an insurance contract.We have certainly
ruled with consis tency that the pres criptive period to bring suit in court under an insurance
policy, begins to run from the date of the insurers rejection of the claim filed by the insured, the
beneficiary or any person claiming under an insurance contract. This ruling is premised upon the
compliance by the pers ons suing under an insurance contract, with the indispensable
requirement of having filed the written claim mandated by Section 384 of the Insurance Code
before and after its amendment. Absent such written claim filed by the person suing under an
insurance contract, no cause of action accrues under such insurance contract, considering that it
is the rejection of that claim that triggers the running of the one-year prescriptive period to bring
suit in court, and there can be no opportunity for the insurer to even reject a claim if none has
been filed in the first place, as in the instant case.
Same; Respondent appellate court committed reversible error in finding petitioner
liable under an insurance contract the existence of which had not at all been proven
in court.When petitioner asseverates, thus, that no written claim was filed by private
respondent and rejected by petitioner, and private respondent does not dispute such
asseveration through a denial in his pleadings, we are constrained to rule that respondent
appellate court committed reversible error in finding petitioner liable under an insurance contract
the existence of which had not at all been proven in court. Even if there were such a contract,
private respondents cause of action can not prevail because he failed to file the written claim
mandated by Section 384 of the Insurance Code. He is deemed, under this legal provision, to
have waived his rights as against petitioner-insurer.

HERMOSISIMA, JR., J.:


FACTS: At about 5:30 oclock in the morning of July 20, 1980, a 78-year old woman by the name
of Feliza Vineza de Mendoza was on her way to hear mass at the Tayuman Cathedral. While
walking along Tayuman corner Gregorio Perfecto Streets, she was bumped by a taxi that was
running fast. Several persons witnes sed the accident, among whom were Rolando Marvilla,
Ernesto Lopez and Eulogio Tabalno. After the bum ping, the old woman was seen sprawled on the
pavement. Right away, the good Samaritan that he was, Marvilla ran towards the old woman and
held her on his lap to inquire from her what had happened, but obviously she was already in
shock and could not talk. At this moment, a private jeep stopped. With the driver of that vehicle,
the two helped board the old woman on the jeep and brought her to the Mary Johnston Hospital
in Tondo.
x x x Ernesto Lopez, a driver of a passenger jeepney plying along Tayuman Street from Pritil,
Tondo, to Rizal Avenue and vice-versa, also witnessed the incident. It was on his return trip from
Rizal Avenue when Lopez saw the plaintiff and his brother who were crying near the scene of the
accident. Upon learning that the two were the sons of the old woman, Lopez told them what had
happened. The Mendoza brothers were then able to trace their mother at the Mary Johnston
Hospital where they were advised by the attending physician that they should bring the patient
to the National Orthopedic Hospital becaus e of her fractured bones . Instead, the victim was
brought to the U.S.T. Hospital where she expired at 9:00 oclock that same morning. Death was
caused by traumatic shock as a result of the severe injuries she sustained x x x x.
x x x The evidence shows that at the moment the victim was bumped by the vehicle, the latter
was running fast, so much so that because of the strong impact the old woman was thrown away
and she fell on the pavement. x x x In truth, in that related criminal case against defendant
Dumlao x x x the trial court found as a fact that therein accused was driving the subject taxicab
in a careless, reckless and imprudent manner and at a speed greater than what was reasonable
and proper without taking the necessary precaution to avoid accident to persons x x x con s
iderin g the condition of the traffic at the place at the time aforementioned x x x. Moreover, the
driver fled from the scene of the accident and without rendering assistance to the victim. x x x
x x x Three (3) witnesses who were at the scene at the time identified the taxi involved, though
not necessarily the driver thereof. Marvilla saw a lone taxi speeding away just after the bumping
which, when it passed by him, said witness noticed to be a Lady Love Taxi with Plate No. 438,
painted maroon, with baggage bar attached on the baggage compartment and with an antenae
[sic] attached at the right rear s ide. The same descriptions were revealed by Ernesto Lopez, who
further described the taxi to have x x x reflectorized decorations on the edges of the glass at the
back. x x x A third witness in the person of Eulogio Tabalno x x x made similar descriptions
although, because of the fas t speed of the taxi, he was only able to detect the last digit of the
plate number which is 8.
x x x [T]he police proceeded to the garage of Lady Love Taxi and then and there they took
possession of such a taxi and later impounded it in the impounding area of the agency
concerned. x x x [T]he eyewitnesses x x x were unanimous in pointing to that Lady Love Taxi with
Plate No. 438, obviously the vehicle involved herein. x x x During the investigation, defendant
Armando Abellon, the registered owner of Lady Love Taxi bearing No. 438-HA Pilipinas Taxi 1980,
certified to the fact that the vehicle was driven last July 20, 1980 by one Rodrigo Dumlao x x x x
x x It was on the basis of this affidavit of the registered owner that caused the police to
apprehend Rodrigo Dumlao, and consequently to have him prosecuted and eventually convicted
of the offense x x x. x x x [S]aid Dumlao absconded in that criminal case, s pecially at the time of
the promulgation of the judgment therein so much so that he is now a fugitive from justice.

You might also like